Jump to content

Wikipedia:Reference desk/Science: Difference between revisions

From Wikipedia, the free encyclopedia
Content deleted Content added
Line 819: Line 819:


:Short and not-in-depth answer: You have to take into account the magnetic fields in space - homogeneous magnetic change only the direction of the velocity vector (except for the Abraham-Lorentz force of course), but inhomogeneous fields also cause a change in kinetic energy. [[User:Icek|Icek]] ([[User talk:Icek|talk]]) 15:09, 23 December 2008 (UTC)
:Short and not-in-depth answer: You have to take into account the magnetic fields in space - homogeneous magnetic change only the direction of the velocity vector (except for the Abraham-Lorentz force of course), but inhomogeneous fields also cause a change in kinetic energy. [[User:Icek|Icek]] ([[User talk:Icek|talk]]) 15:09, 23 December 2008 (UTC)
::Hmm - magnetic fields never change the potential energy a charged particle, regardless of the inhomogeneity. I think what the OP is looking for is [[Coulomb scattering]]. An ion traveling through a "cloud of ions" (see [[plasma|plasma (physics)]]) interacts with the background ions via the coulomb interaction, thus transferring some of its kinetic energy to the background ions. This leads to an exponential decay of momentum and energy of the incident ion. I won't comment on the rest of the plans you don't have. --[[User:Bmk|Bmk]] ([[User talk:Bmk|talk]]) 05:35, 24 December 2008 (UTC)
::Hmm - magnetic fields never change the total kinetic energy a charged particle, regardless of the inhomogeneity. I think what the OP is looking for is [[Coulomb scattering]]. An ion traveling through a "cloud of ions" (see [[plasma|plasma (physics)]]) interacts with the background ions via the coulomb interaction, thus transferring some of its kinetic energy to the background ions. This leads to an exponential decay of momentum and energy of the incident ion. I won't comment on the rest of the plans you don't have. --[[User:Bmk|Bmk]] ([[User talk:Bmk|talk]]) 05:35, 24 December 2008 (UTC)


== What happens to sexual desire after penectomy? ==
== What happens to sexual desire after penectomy? ==

Revision as of 05:38, 24 December 2008

Welcome to the science section
of the Wikipedia reference desk.
Select a section:
Want a faster answer?

Main page: Help searching Wikipedia

   

How can I get my question answered?

  • Select the section of the desk that best fits the general topic of your question (see the navigation column to the right).
  • Post your question to only one section, providing a short header that gives the topic of your question.
  • Type '~~~~' (that is, four tilde characters) at the end – this signs and dates your contribution so we know who wrote what and when.
  • Don't post personal contact information – it will be removed. Any answers will be provided here.
  • Please be as specific as possible, and include all relevant context – the usefulness of answers may depend on the context.
  • Note:
    • We don't answer (and may remove) questions that require medical diagnosis or legal advice.
    • We don't answer requests for opinions, predictions or debate.
    • We don't do your homework for you, though we'll help you past the stuck point.
    • We don't conduct original research or provide a free source of ideas, but we'll help you find information you need.



How do I answer a question?

Main page: Wikipedia:Reference desk/Guidelines

  • The best answers address the question directly, and back up facts with wikilinks and links to sources. Do not edit others' comments and do not give any medical or legal advice.
See also:


December 16

Density (follow on from the above)

So say i made something out of this Osmium stuff, how much more would it weigh than say some tin, or some plastic, or iron, or other stuff. I mean to say...if I had say a chunk the size of a golf-ball would I be able to pick it up? I know that's a bit of a simple question but i'm not sure what 22.6 g/cm3 is like in comparison to say everyday option (essentially i'm looking for a weight-based equivilent of those "this ship could fit 5 empire state buildings end-to-end in its hull" style claims). 194.221.133.226 (talk) 13:19, 16 December 2008 (UTC)[reply]

According to density#Densities of various materials, that's about twice as heavy as lead. Pretty heavy, but you could certainly pick up a golf ball of it. Algebraist 13:42, 16 December 2008 (UTC)[reply]
EC:Well, lead, as in bullets, has a density of 11.34 g/cm3, or about half that of osmium. StuRat (talk) 13:42, 16 December 2008 (UTC)[reply]
A sphere the size of a golf ball would have a volume of 40.67cm3. So, that'd work out to about 919.1g so, about two pounds. About twenty normal golf balls.
(For comparison, a lead golf ball would only weigh as much as ten normal golf balls.) APL (talk) 13:45, 16 December 2008 (UTC)[reply]
To put it in terms more illustrative to a drunkard like me: a fifth of bourbon weighs 750 grams (1.6 lbs), while a fifth of osmium would weigh 17,000 grams (37 lbs)! --Sean 14:14, 16 December 2008 (UTC)[reply]
Quite the heavy drinker, eh? What mixer would one use? DMacks (talk) 14:42, 16 December 2008 (UTC)[reply]
My official unit of weight is a 1l Tetra Brick of milk, which is close enough to 1kg or 2lb for getting an intuitive grasp. Quite by coincident, its about the same weight as the Osmium golf ball. If you prefer soft drinks, 2 golf balls is a 2l bottle of coke.... --Stephan Schulz (talk) 15:32, 16 December 2008 (UTC)[reply]
I had touble visualising a pound (I've always wallowed in the metric system) - so I went around the house weighing things. My sneaker weighs exactly a pound. SteveBaker (talk) 02:31, 17 December 2008 (UTC)[reply]
There's the answer then : A Osmium golf ball would weigh as much as both of SteveBaker's shoes. APL (talk) 04:57, 17 December 2008 (UTC)[reply]
Visualising weights and densities requires a bit of mental preparation. The metric system is a HUGE help in that regard. The way to think about metric weights is that a 10 cm x 10 cm x 10 cm cube of water is 1 liter (an alternative name for a liter is a cubic-decimeter) - and one liter of water weighs a kilogram. That's why 1 liter of milk weighs 1kg and 2 liters of coke weighs 2kg - because milk, coke and water are pretty much the same density. The density of water is 1kg/liter - but handily, a 'cc' (cubic centimeter) is 1/1000th of a liter and a gram is 1/1000th of a kilogram. So 1g/cc and 1kg/liter are the same density. You can go up a step and visualise that too - a metric ton (tonne) is 1000kg or 1 megagram - that is the weight of a 100cm x 100cm x 100cm cube of water - a 1m x 1m x 1m cube. So a 1 meter ice cube would weigh a tonne. So now you can think 1g/cc, 1kg/liter and 1tonne/cubic-meter are all the same density - and now you can visualise the gram, kilogram and tonne as 1cm, 10cm and 100cm cubes of water - roughly, something the size of a sugar cube, a tetrapack of milk or a large refigerator (to stick with a 'kitcheny' theme!). So Osmium is 22g/cc, 22kg/liter or 22tonnes/cubicmeter - lead is 11g/cc, 11kg/liter or 11tonnes/cubicmeter, iron is 6g/cc, 6kg/liter or 6tonnes/cubicmeter. Gasses are harder to visualise. Air is just 0.0012g/cc which is impossible to visualise - but we can go with 1.2g/liter, 1.2kg/cubicmeter and mentally compare the weight of a tetrapack of water to the weight of the air inside a large, empty refrigerator and be roughly right. SteveBaker (talk) 18:43, 16 December 2008 (UTC)[reply]
Steve, Steve, Steve... did you just say that ice has the same density as water? A m3 of ice will weigh less than a m3 of liquid water. I'm so shocked that I'm not even going to get on your back about kilograms not being a measure of weight! Matt Deres (talk) 17:43, 17 December 2008 (UTC)[reply]
We're looking for ROUGH ways to visualise weight - not PRECISE ways to measure it - the difference between the density of ice and water is well within my error bars here! A liter of liquid water doesn't weigh a kilogram either - it depends on temperature and in any case, the standard definition of a kilogram is the mass of some block of metal in a museum someplace. Also, (rightly or wrongly) we all describe the weight of things in kilograms all the time - if you don't believe that, you're going to have a very hard time making your way in the world! There is a time to be pedantic and a time to help the OP...this is the latter. SteveBaker (talk) 19:50, 17 December 2008 (UTC)[reply]
Dude, you're talking to him about g/cc densities and megagrams, but I'm being pedantic? ;-) You're 2.54 centimetering into hypocrisy on that one. And if there's one thing that gets 5.44N'ed into you on these desks, it's that you can always make room for pedantry! Matt Deres (talk) 20:55, 17 December 2008 (UTC)[reply]
How is measuring densities in g/cc pedantic? How would you measure them? --Tango (talk) 21:26, 17 December 2008 (UTC)[reply]
It would depend on what it was. I don't think of cc being used anywhere outside of the medical and auto spheres, which is probably why Steve is familiar with it. I've been raised in metric, and I use litres (and mL) for all general volume uses of that kind of scale. Obviously, there's nothing wrong with it (I was more interested in giving Steve a hard time than anything, after all), but it's not a unit I would tend to use. If I say something like "Gimme 100 cc's of chocolate milk!" surely that brings up an image of getting an intravenous injection even without the "Stat!" at the end? Matt Deres (talk) 22:44, 17 December 2008 (UTC)[reply]
Either that or the name of an especially pathetic motorbike (cf "Gimme 800cc of Harley between my legs!") The reason I used 'cc' was because cm<sup>3</sup> (or worse: 10<sup>-6</sup>[[meter|m]]<sup>3</sup>) is harder to type. Also, in my defense, I was educated right at the start of the time when kids were taught the metric system in British schools - and the precise details of whether you should say 'cc' or 'cm3' or 'ml' were still being thought through. It wasn't "SI-units" or "mks" (meter-kilogram-second) - it was some fuzzy idea of "The Metric System" or perhaps "cgs" (centimeter-gram-second) - none of which are exactly the same thing! It's taken me ages to stop thinking about "dynes" and "ergs" - but "cc" does creep in once in a while! But (as I said before) the meaning is clear and in informal prose such as the RD, it really is OK to use them interchangeably. At least we don't have to worry about pounds and ounces and fluid ounces(British) versus fluid ounces(US)...yuck! SteveBaker (talk) 21:07, 18 December 2008 (UTC)[reply]

A critical dictionary of psychology?

Looking through the APA Dictionary of Psychology (first=latest edition, 2006, ISBN 1-59147-380-2), I see that it treats psychoanalysis, analytic psychology, "neuro-linguistic programming", and even scientology in all seriousness. I mean, these are treated as living subfields of psychology/psychotherapy/etc, not as well-intentioned historical missteps (or worse). While the entry for "pseudoscience" does give scientology as an example, it doesn't give any of the others, and the entry for "scientology" doesn't link to pseudoscience. All of this strikes me as an extraordinary -- but no, I mustn't speechify. There are of course plenty of books that debunk this or that pseudoscience or pseudotherapy, but is there a fairly up-to-date general-purpose dictionary of the putative science of psychology that's discerning (whether by evaluation or selectivity) and recommendable? Tama1988 (talk) 11:09, 16 December 2008 (UTC)[reply]

But what about Scientology and psychiatry? Scientologists are strongly opposed to psychology/iatry.-- MacAddct1984 (talk &#149; contribs) 15:12, 16 December 2008 (UTC)[reply]
All definitions seem just like something you'd expect in a dictionary. They aren't treated as living subfields of psychology. They are merely described. This dictionary also does not take a stance towards animism, sociobiology, Christian science, or evolutionary psychology. The only entry that I have found that explicitly takes a stance is social Darwinism.--droptone (talk) 16:03, 16 December 2008 (UTC)[reply]
This is the crucial difference between a DICTIONARY and an ENCYCLOPEDIA. A dictionary defines what the word means - an encyclopedia provides facts and explanation relating to that meaning. Hence, here at WikiPEDIA, we talk about Astrology and explain that it's a pseudoscience - but over at WikTIONARY we simply say that it's "The study of the movements and relative positions of celestial bodies and their supposed influence on human affairs.". Dictionaries are not there to explain - merely to define. You shouldn't read any kind of acceptance of these ridiculous thing into their presence in the dictionary. SteveBaker (talk) 18:06, 16 December 2008 (UTC)[reply]
Then why would they call social Darwinism "a discredited theory ..."?--droptone (talk) 21:40, 16 December 2008 (UTC)[reply]
It's a good question, actually. Social Darwinism can mean many, many things, some of which are discredited, some of which are just seen as not in vogue, some of which are considered pretty standard. Much the same can be said about psychoanalysis. What do they say about, say, "eugenics"? --98.217.8.46 (talk) 01:46, 17 December 2008 (UTC)[reply]
"a social and political philosophy, based loosely on the evolutionary theory of Charles Darwin...The eugenic position is groundless and scientifically naive, in that many conditions associated with disability or disorder, such as syndromes that increase risk of mental retardation, are inherited recessively and occur unpredictably". It defines g as hypothetical, they raise objections in the definition of IQ but dismisses them, chi is defined without objections, acupuncture is defined while saying that scientists are unable to explain how it works (with the presumption that it does), homeopathy is defined without objection, reflexology is also defined without objection, as is reiki.--droptone (talk) 13:23, 17 December 2008 (UTC)[reply]

Thank you for your responses and my apologies for my absence.

The APA Dictionary announces that it represents a snapshot of the lexicon of psychology as it exists at the beginning of the 21st century. [...] Although selective in scope, it seeks not to limit the vocabulary but, rather, to delineate the current and evolving understanding of the language used within the field. That last sentence seems bizarre, implying as it does that language has been used "within the field" with a limited understanding of what that language meant; I'll guess at some copyediting error here. Surely this is the language that is used (and perhaps also obsolete language that is still quoted and discussed) for the current and evolving understanding of the field. There's no hint that it's also a dictionary of what I imagine would be called ethnopsychology, vernacular understandings and misunderstandings of the area treated by psychology.

The only comparable book that I possess is Stuart Sutherland's International Dictionary of Psychology (2nd ed., 1995), which I inexpertly think is very good but which is showing its age. This doesn't bother with "neuro-linguistic programming" or "scientology", or with any that I bothered to look up of the depressing list (homeopathy, reflexology, etc.) that Droptone gives above. It does cover the wobbly Freudian and Jungian edifices, and it does so politely; but Sutherland discreetly inserts such phrases as "said to be" in these contexts, so the reader isn't lulled too easily into taking it all too seriously. As for seriousness, Sutherland is a witty writer where he can be (the two prefaces make me laugh), but the comments on this book at amazon.com unfairly highlight the Johnsonian joke entry for "Love", which is very atypical.

I suppose what I'm looking for is something like an updated and expanded 3rd ed. of The International Dictionary of Psychology. Tama1988 (talk) 02:40, 18 December 2008 (UTC)[reply]

the method to reduce alcohol or menthol scent

I need to know, How to reduce alcohol or menthol scent but still keep the cool function?

may answer to my e-mail ***@*** —Preceding unsigned comment added by Pattytai (talkcontribs) 14:55, 16 December 2008 (UTC)[reply]

E-mail address removed, as per ref desk rules (see the top of this page). Mostly to protect you from spam etc. Aeluwas (talk) 15:13, 16 December 2008 (UTC)[reply]
Alcohol, when applied to the skin, cools via evaporation: the same mechanism that allows sweating to cool the body. I am guessing alcohol feels cooler because it vaporizes much quicker taking heat away at a higher rate even at room temperature. Menthol creates a cooling sensation via a different mechanism; it binds to receptors on the skin which fools the brain into thinking that patch of skin is cool (like inverse capsaicin). Alcohol must evaporate in order to cool, so the only way to reduce the smell is to mask it with other, nicer smells. Menthol on the other hand just needs to be in contact with the skin so preventing it from evaporating (i.e. becoming smellable) is one solution, along with masking it with other smells. Mixing the menthol with wax or petroleum jelly may help a little... but it will still be smellable anyway. 152.16.15.23 (talk) 16:19, 16 December 2008 (UTC)[reply]

Flat screen moving volumetric display with full vertical and horizontal parallax?

Has anyone ever invented or have a design for a "perfect" large, high definition, (nearly) flat screen volumetric display with 180 degree vertical and horizontal parallax, and proper occlusion of overlapping light and dark areas? And if no one has, what would be the best way for a lone inventor to exploit or promote the design? —Preceding unsigned comment added by Trevor Loughlin (talkcontribs) 15:18, 16 December 2008 (UTC)[reply]

I think the article Volumetric display and its links gives an account of the state of the art. Can't say I've heard about anything resembling what you describe, but holograms do it well for static pictures in black and white. If you have a design that you want to promote, this is probably not the right place to ask. But there are forums on the net as well as personal accounts by inventors that discuss how to proceed from an idea to a finished product. EverGreg (talk) 15:36, 16 December 2008 (UTC)[reply]
You might try building a working prototype and using it as a demo to get investors and form your own business selling volumetric displays. I'm sure this isn't easy, but it probably has the highest potential pay-off for inventors. APL (talk) 17:25, 16 December 2008 (UTC)[reply]
There are three additional criteria to specify:
  1. Can you view it without any kind of special eyewear or headwear?
  2. Is it capable of supporting multiple simultaneous viewers?
  3. Can it do appropriate hidden surface occlusion?
If the answer to any of those three questions is "No" then there are several solutions. The only systems that I'm aware of that can cope with multiple viewers and does not require special user-mounted equipment and can do hidden surfaces - are holograms. Contrary to what EverGreg says - I've seen (with my own eyes) moving, interactive, color holograms generated by computer. The system was a half million dollars worth of Silicon Graphics workstation with some very custom hardware involving ultrasonics and all sorts of other mysterious things - but it could display a small (like 5 centimeter) low resolution color hologram of a cube that was spinning - being updated a couple of times a second. I suspect that it only had horizontal parallax - but in most applications, that's good enough.
So in answer to whether a "perfect" solution exists - certainly not. Nor do I expect we'll ever see one that isn't a hologram simply because if you don't know where your viewers are and they don't have some kind of modified vision or are in some other manner restricted - then your display has to generate the complete waveform of the light that would come from a real object - and that's a hologram.
Not one of the systems out there (including the hologram) is able to exclude light coming from behind the object - so at best, you get the R2D2 "Help me Obi-Wan Kinobi you're my only hope" type of translucent hologram - not the StarTrek "Holodeck" kind that look 100% real. It's really hard to imagine how a system could do that...even in principle.
I agree that if you can build a working prototype (and get your patent registered) that would be the ideal first step. Once you have those two things - just announce it in as many places as you can think of (Slashdot and Digg would be good places!) and the news will spread like wildfire. If it really does work as you seem to be implying - there will be no shortage of people beating a path to your door. However, you must forgive me for being skeptical - there have been VERY many attempts at solving this in the past and I doubt very much that you've cracked it either. If you have not built a working prototype then I'm afraid that it's overwhelmingly likely that there is something that you have not thought about.
SteveBaker (talk) 17:53, 16 December 2008 (UTC)[reply]
Lenticular displays are making progress, and they meet Steve's three bullet points, but they only have parallax in one dimension, so they don't answer the question either. APL (talk) 19:32, 16 December 2008 (UTC)[reply]
Lenticular displays are a great theoretical solution - but building them with enough resolution to give you stereopsis and arbitary view direction requires insanely high-res displays. In the limit, they ARE holograms because if you can get the resolution up high enough to do arbitary viewer smoothely, you have enough resolution to render a hologram directly and you can dump the lenticular lens layer. SteveBaker (talk) 19:55, 16 December 2008 (UTC)[reply]
Incidentally, haven't you already invented a 'Retrocausal information transfer' device? You could probably use that to make money on the stock market, then use those earnings to finance your 3d display company.
See you at SIGGRAPH! APL (talk) 19:43, 16 December 2008 (UTC)[reply]
Ah - thanks for the reminder...OK - nothing to see here - move along! SteveBaker (talk) 19:55, 16 December 2008 (UTC)[reply]

My idea meets all three criteria. As for retrocausal data transfer, I can now predict (0-3) decimal at a couple of percent above expectations, instead of just one bit at fractions of a percent greater than chance, so I could finance it, but getting a team together to sort out the practicalities is more of a challenge. I will send Steve Baker (who seems to know more about optics than me) an avi video of my design to see if it is practical and completely original-or so expensive or open to challenge that I may as well give the idea away on this forum, bearing in mind the legal minefield the patent process is for even a well financed private individual these days.Trevor Loughlin (talk) 05:49, 17 December 2008 (UTC)[reply]

No - please don't send it to me. I also invent things - I have patents on display technologies - and I don't want you claiming that I stole something from you. First patent - THEN show to people. SteveBaker (talk) 19:42, 17 December 2008 (UTC)[reply]
Well since you can get the finances I don't think assemblying a team will be a great challenge. Sure you may not be able to convince Nobel prize winners to join you or even the top students (well unless your evidence and ideas are very strong perhaps) but provided you have resonable evidence and theories and the cash to pay them, I'm sure you can easily entice some okay scientists, engineers and the like to join your team. Obviously they're only going to work for you if you can guarantee a resonable salary for the time they are employed, you can't expect them to work on the promise of future riches alone Nil Einne (talk) 12:47, 18 December 2008 (UTC)[reply]

Most powerful force on earth

What is it? —Preceding unsigned comment added by 79.67.56.238 (talk) 23:46, 16 December 2008 (UTC)[reply]

That's rather difficult to define, really. Do you want something sustainable, or is something that's extremely powerful for very short periods enough to qualify? Does an external force applied to the whole Earth qualify? If so, the Sun's gravity (ie. the Earth's weight) is almost certainly it at 3.5x1022N, I can't think of anything else comparable. --Tango (talk) 23:56, 16 December 2008 (UTC)[reply]
Clarification: forces (transitory or continuous) created solely on and by the planet we call earth; and not due to any external solar system/universal effects. —Preceding unsigned comment added by 79.67.56.238 (talk) 00:05, 17 December 2008 (UTC)[reply]
Ok. There must be some pretty major forces at work during an earthquake - making large amounts of the Earth's crust move around would require a lot of force. That would be my guess. (Well, actually, your definition would allow the Earth's gravity on the Sun, which is exactly the same as the other way around, but I'm guessing you didn't mean to include that!) --Tango (talk) 00:17, 17 December 2008 (UTC)[reply]
Hmmm - F=Ma - so we either need a very big mass - or a very big accelleration. The earth/sun gravitational force involves some pretty spectacular masses - but the accelerations aren't that big. Maybe we should be looking at smaller masses with bigger accelerations. But whatever it is, it's not going to be able to knock the earth out of it's orbit - so it'll have to be a very brief acceleration. SteveBaker (talk) 02:26, 17 December 2008 (UTC)[reply]
UM, I call bullshit on "accelerations aren;t that big" in the Earth-Sun system. The Earth is turning around a huge circle at a fantastic rate of speed; curved motion represents an acceleration, so there are some rather profound accelerations in the system... --Jayron32.talk.contribs 05:55, 17 December 2008 (UTC)[reply]
Centripetal acceleration is For T=1 year and R=1 AU, google tells me that is 0.006 ms-2. That's a pretty tiny acceleration. --Tango (talk) 13:16, 17 December 2008 (UTC)[reply]
Yup, I get the same. That's 0 to 60 mph in about 75 minutes. Gandalf61 (talk) 13:23, 17 December 2008 (UTC)[reply]
SteveBaker's F=ma is great - it mathematicizes the question. But the original poster asked about the most powerful force, so we might consider P = dW/dt (power is the rate of change of energy). So we might expand or restate the "big mass or big acceleration" to also include "lots of energy or very short amounts of time" - so, might the Strong Nuclear Force, with some of its transient interactions on the order of planck time scales, have extremely large bursts of momentary power? Nimur (talk) 15:35, 17 December 2008 (UTC)[reply]

What about what's going on in the core of the earth? Surely that's the answer?Rfwoolf (talk) 02:28, 17 December 2008 (UTC)[reply]

If you're okay with a very brief condition, I'd think that the detonation of a hydrogen bomb would create some pretty stupendous forces at its onset. As a side note, I know it isn't technically force, but Diamond anvil cells can achieve pressures more than one million times normal atmospheric pressure. -RunningOnBrains 06:14, 17 December 2008 (UTC)[reply]

Further clarification: I meant natural forces; not man or machine made ones.--GreenSpigot (talk) 14:58, 17 December 2008 (UTC)[reply]

The forces of flowing water: water tears down mountains to dust, over and over again. --VanBurenen (talk) 15:02, 17 December 2008 (UTC)[reply]
It's a nebulous question, so I'll throw in Krakatoa, wave power, and wind power, although the latter two are of course caused by extra-planetary forces. --Sean 15:25, 17 December 2008 (UTC)[reply]

The Strong Force! at about 10^39 times stronger than gravity, not to be dismissed. —Preceding unsigned comment added by 92.3.10.246 (talk) 00:59, 19 December 2008 (UTC)[reply]

probably not the answer you were looking for but: a good lie. "Not all the armies in all the world can stop an idea whose time has come..." —Preceding unsigned comment added by 94.27.221.237 (talk) 14:15, 19 December 2008 (UTC)[reply]

if so earth it self is the strongest force on earth--אזרח תמים (talk) 16:56, 22 December 2008 (UTC)[reply]

Newton's Third: What about the rest of the universe acting on earth?


December 17

The Vagina and an imperforate hymen

I can't seem to get clarity on this - if you have an imperforate hymen, then fluids would not be able to escape from the vagina, so most articles that touch on this simply say "so a procedure is needed" - but it doesn't explain the consequences of what would happen if a procedure was not performed...
In other words:
If you have an imperforate hymen would the fluids get reabsorbed into the blood stream, would the menstrual cycle just discontinue, or would the whole thing burst?
Rfwoolf (talk) 01:44, 17 December 2008 (UTC)[reply]

I think I found the answer here: "The fluid gets backed up inside the vagina and causes a condition called hematocolpos -- a big wad of blood". I assume that when this happens the menstrual cycle stops, but I do wonder if there's any re-absorption into the system or what, and assuming the menstrual cycle did not discontinue it would probably eventually burst. Interesting... Rfwoolf (talk) 01:59, 17 December 2008 (UTC)[reply]
I believe that symptoms and signs would occur long before any 'bursting' occurred. It would be very likely that pain due to the haematocolpos would be felt which would draw attention to the problem. 86.4.182.202 (talk) 08:54, 18 December 2008 (UTC)[reply]

When discussing the most important organ, care must be taken to spell everything correctly: "hæmatocolpos".

Does human hair change color while decomposing in water?

I know that hair decomposes very slowly under normal circumstances. But, can loss or change of pigmentation occur during say, 6 months of death and decomposition? If so, how much difference would it make, if any, if the body was submerged in fresh, warm water? The body in question is one making headlines in Florida currently, so was in a somewhat tropical climate. Thanks so much. MoeJade (talk) 02:10, 17 December 2008 (UTC)[reply]

Sunlight (if it was present) causes depigmentation. That's all I know. Rfwoolf (talk) 02:24, 17 December 2008 (UTC)[reply]
That stone-age guy they found mummified in deep snow in the Italian alps had hair that retained it's color over maybe 100,000 years...so depigmentation doesn't happen in the dark under ice.
Human hair wigs seem to last for many decades (at least) - so air and sunlight isn't really enough. I suppose submersion could make a big difference - but that doesn't seem likely to me. SteveBaker (talk) 03:12, 17 December 2008 (UTC)[reply]

See New York Medical Journal, 1870, p.412, [1] which cites the observations of a Dr. Hauptmann, that a man who had dark brown hair during life, was found to have red hair when dug up after 20 years in the ground. A Prof. Sononschein said that dark hair becomes red after long burial due to an acid reaction. When it is claimed hair "retained its color" we can't ascertain the truth of the statement without knowing the original color. The Hauptmann finding was cited in "A Treatise on Diseases of the Hair"(1912) by Jackson, p. 71 [2]. Hair which has been dyed can certainly change color if the dye washes out or reacts chemically with the water [3]. Old and credulous sources say the hair color can change post-mortem, but their reliabliity is called into question by the repeated claim that hair and fingernails continue growingto great lengths after death [4]. This is even seen in encyclopedias and medical books from the first 2 decades of the 20th century.Publications of the Body Farm [5] should provide a modern answer to the question of the effect of submersion after death on hair color, but I did not find the specfic information. "Advances in Forensic Taphonomy" (2001) p 123 says"Hair color is usually modified as a result of burial and early decomposition" Melanin (dark hair) becomes lighter, and hair of those buried a long time (archeological samples) tends to make it "ginger" or reddish. P383 says wool (which is hair of a sort) turns "honey brown" in "waterlogged anoxic conditions." Edison (talk) 16:02, 17 December 2008 (UTC)[reply]

Liquid light

Can high density photons turn into liquids or solids ? Can they be consumed ? 69.157.229.14 (talk) 03:04, 17 December 2008 (UTC)[reply]

No, they can't be turned into liquids or solids. They can be absorbed though - when you shine a light on a black surface - the surface is "consuming" the photons - making the surface a little warmer. SteveBaker (talk) 03:18, 17 December 2008 (UTC)[reply]

What about electrons ? This is a could question, meaning with various futuristic technologies ? 69.157.229.14 (talk) 03:26, 17 December 2008 (UTC)[reply]

Electrons can also be absorbed by liquids or solids. Just think of the old computer monitors and tvs. Their absorbtion, just like light, also makes things warmer. Sjschen (talk) 03:41, 17 December 2008 (UTC)[reply]
The only elementary particle that could be a liquid or solid as far as I know is a neutron. See neutron star. --98.217.8.46 (talk) 04:15, 17 December 2008 (UTC)[reply]
That is of course, stretching the definition of "liquid" and "solid" beyond the bounds of creditibility. The standard three-phase model of matter (solid-liquid-gas) presupposes atomic matter at standard room conditions. Once you start to mess with those conditions (i.e. non-atomic or subatomic particles) or raise the temperature to very high levels, the three-phase model breaks down. --Jayron32.talk.contribs 05:51, 17 December 2008 (UTC)[reply]
I'm utterly out of my depth in this subject, but I discovered by Googling "photonium" that there is a posited entity - I don't know if you could call it matter - that might result from virtual electron/positron pairs if you could persuade them to remain stable (see here for a serious example). Make sure you put on your tinfoil hat before searching for this stuff, however, because some of it is fringe science or science fiction. --Heron (talk) 13:25, 17 December 2008 (UTC)[reply]
Just because a concept has a name and google hits in no way makes it credible. Do a Google search for 'teleport' and you get 6.1 million hits - does that make it real/possible? No. SteveBaker (talk) 19:37, 17 December 2008 (UTC)[reply]
I know. That's why I provided a link to a (speculative) statement by a professional physicist. He could still be wrong, of course. --Heron (talk) 20:35, 17 December 2008 (UTC)[reply]
Isn't a stable virtual particle something of a contradiction in terms? --Tango (talk) 14:13, 17 December 2008 (UTC)[reply]
I probably misunderstood the page I linked to. In some places it talks about creating virtual e+e- pairs and in other places it seems to imply that they are real particles. I don't know how they change from virtual to real, if indeed they do, and neither do I know whether the 'photonium' of which he speaks is composed of these particles or is just the field in which they sit. I was hoping that one of you boffins could sort it out for me, but you don't seem to be biting. :) --Heron (talk) 20:35, 17 December 2008 (UTC)[reply]
Yes photons can turn into liquids in a sense. Two photons can be made to collide, producing elementary particles which may or may not then form part of a liquid. Most likely they'd be a plasma. Photons themselves cannot form a liquid because the particles that form a liquid need to be able to bump into each other. Photons can't bump into each other. EverGreg (talk) 14:35, 17 December 2008 (UTC)[reply]
That's just saying "photons can turn into things that in turn can form liquids and solids" - the photons themselves can't do that. SteveBaker (talk) 19:37, 17 December 2008 (UTC)[reply]
I think you've just refuted my speculation above, Steve. I'll stop arguing now. -Heron (talk) 20:40, 17 December 2008 (UTC)[reply]
Photons can bump into each other (redlink?! try the literature), so a photon gas could be interacting, but they can't have attractive interactions, so they can't be condensed matter. --Tardis (talk) 19:11, 18 December 2008 (UTC)[reply]
An electron and positron in "orbit" around each other form an "atom" referred to as positronium, which has been generated in the lab. Some versions of it can actually stick around for a fraction of a second before decaying, too. Confusing Manifestation(Say hi!) 22:09, 17 December 2008 (UTC)[reply]

Surely light can't form a plasma? A Bose-Einstein Condensate seems more likely than any of the more conventional forms of matter. I don't know if it's possible, but at least you'd be heading along the right lines. 81.174.226.229 (talk) 14:32, 18 December 2008 (UTC)[reply]

speed of sound

Assume that sound is traveling through a medium of x density.

How does the speed of sound tend as x approaches 0? —Preceding unsigned comment added by 24.251.110.126 (talk) 04:04, 17 December 2008 (UTC)[reply]

I don't know the answer off-the-cuff, but reading the speed of sound article may help. --Scray (talk) 04:13, 17 December 2008 (UTC)[reply]
The speed of sound when x=0 is 0, so isn't that the answer? --99.237.96.81 (talk) 04:29, 17 December 2008 (UTC)[reply]
We weren't asked what the speed of sound is at x=0, but how it behaves as x approaches 0. This is a slightly different question, and has to do with Limit (mathematics), specifically Limit of a function. Mathematically speaking, the limit of a function as x->a does not have to be the same as the value of the function when x=a. For example the limit of the function f(x) = (3x-6)/(x-2) as x approaches 2 is 3, even though the value of f(2) is undefined. -- 128.104.112.113 (talk) 19:07, 17 December 2008 (UTC)[reply]

The speed of sound is inversely proportional to the square root of the density, so as density approaches 0, speed approaches infinity. Of course the medium cannot be changed in this way without changing the stiffness, so your result will differ. Graeme Bartlett (talk) 04:54, 17 December 2008 (UTC)[reply]

Also, as the density approaches 0 (think of very diffuse gas, approaching vacuum conditions), the material will no longer sustain a sound wave, and the pressure and energy wavefront will lose coherence and dissipate. Nimur (talk) 15:39, 17 December 2008 (UTC)[reply]

plants

because plants breath co2 like we breath o2, would adding co2 to a plant in.....(lets say a box)...make it grow faster? —Preceding unsigned comment added by 76.14.124.175 (talk) 04:21, 17 December 2008 (UTC)[reply]

Yes, and in fact glasshouses often have CO2 added in the form of decomposing organic matter to make the plants grow quicker. Graeme Bartlett (talk) 04:44, 17 December 2008 (UTC)[reply]
(edit conflict) A news release from Stanford University dated December of 2002 indicates that it's not that simple. Yes, it seems that increasing CO2 increases growth, all else being equal, but in combination with increased temperature, water, or soil nitrogen, an elevated ambient level of CO2 can actually retard growth. --Milkbreath (talk) 04:50, 17 December 2008 (UTC)[reply]
Yes, but only if lack of CO2 is the limiting factor holding back growth. If lack of water, sunlight, or minerals are the issue, then more CO2 won't help until those other shortages are addresed. StuRat (talk) 15:01, 17 December 2008 (UTC)[reply]
Also keep in mind that plants only use CO2 during photosynthesis. They produce CO2 during regular Cellular respiration. Increased CO2 wouldn't be useful at night, for example. Some plants have evolved special carbon fixation cycles that allow the photosynthesis to use CO2 produced by respiration. (See CAM photosynthesis and C4 carbon fixation)-- JSBillings 16:11, 17 December 2008 (UTC)[reply]
Certainly it can help - but as JSBillings says - plants only consume CO2 during daylight - in the dark, they actually PRODUCE CO2 - just like us animals. Another issue that's come up as a part of the global warming debate is that it seems that quite often the weeds that compete with our crop plants do even better with increases CO2 than the plants we're trying to encourage - so I guess in some situations, it might result in a shift in the 'balance of power' between competing species that could be counter-productive. SteveBaker (talk) 19:28, 17 December 2008 (UTC)[reply]

Indian apple

I've been trying to find out the scientific name for the spiny fruit shown in this picture (bottom right side, on the table). In Tonga, it's known as the "Indian apple", but I can't find any references explaining what the scientific name is, or if it has any other names. Anyone know the answer? Thanks! ···日本穣? · Talk to Nihonjoe 05:54, 17 December 2008 (UTC)[reply]

:To me, your pic looks like Papaya, but "Indian apple" gets you Pomegranate which is more rounded and a different colour. Still unclear, Julia Rossi (talk) 09:40, 17 December 2008 (UTC)[reply]

Oh, bottom right on the table is a "star fruit" or Carambola... as per the handwritten name tag, : ) Julia Rossi (talk) 09:42, 17 December 2008 (UTC)[reply]
Yeah, I read that tag, but it's not by the fruit in question (as you noticed below). Thanks, though. :) ···日本穣? · Talk to Nihonjoe 03:47, 18 December 2008 (UTC)[reply]
I think it is a Annona muricata, which is a type of custard apple. (another picture). Apparently, it is called apele ‘initia in Tonga, which could have been vulgarized into "Indian apple" (speculating here!) Abecedare (talk) 10:07, 17 December 2008 (UTC)[reply]
That's the fruit. I think you are probably right about the "Indian apple" part. Thanks! ···日本穣? · Talk to Nihonjoe 03:47, 18 December 2008 (UTC)[reply]
Julia, I don't think you were seeing the whole image. --Sean 15:30, 17 December 2008 (UTC)[reply]
Arg, you're right, Sean – there's nothing like having the whole picture. Annona M. it is. :-P Julia Rossi (talk) 21:02, 17 December 2008 (UTC)[reply]
You might be interested in the excellent image zoom extension to Firefox. --Sean 15:12, 18 December 2008 (UTC)[reply]
Thanks for the tip. Just upgraded firefox and it looks to have a built in pic zoom using the text-shrink keys (apple -). That's better! 8-) Julia Rossi (talk) 08:42, 20 December 2008 (UTC)[reply]

Hiroshima Peace Memorial

I just wanted to be clear on the reason that the building that became the Hiroshima Peace Memorial survived the A-bomb as well as it did.

The story I was given was:

  • The building was almost directly below the hypocenter.
  • Therefore the shockwaves that hit the structure were traveling almost vertically.
  • Therefore its horizontal structures (e.g. ceilings) were largely destroyed while its vertical structures (e.g. walls) were largely preserved.

Is this correct? Joaq99 (talk) 11:20, 17 December 2008 (UTC)[reply]

That sounds reasonably correct, but keep in mind that only a small portion of the building actually "survived as well as it did" (compare before and after). It's also worth noting that reinforced structures can do okay depending on the conditions of the shock wave. If it had been a wooden house that had been right underneath, it would have been smashed flat, no problem. --98.217.8.46 (talk) 14:48, 17 December 2008 (UTC)[reply]


The dome was 150m from the hypocenter and the bomb was detonated 600m above ground. This gives a blast 15 degrees off vertical. But the article links to this page: [[6]] It states that the dome and surrounding ruins was part of some larger complex, so even close to the hypocenter, walls were not generally spared. You'd have to look at more ruins to say if they'r statistically more likely to withstand at the hypocenter, but remember that a nuclear explosion also creates a strong "backdraft" feeding the rising mushroom stem. In films from nuclear test explosions, it can be seen that this backdraft is almost equally devastating on buildings and it probably didn't have the exact same direction as the blast. So my answear would be no, the explanation you got is not quite good enough. EverGreg (talk) 14:51, 17 December 2008 (UTC)[reply]
The explanation seems a bit ad hoc. Was it based on any computer simulations of how the blast would affect the structure, based on the actual plans? Edison (talk) 15:32, 17 December 2008 (UTC)[reply]
Could be an example of survivor bias here. Some building has to be the closest surviving building to the hypocenter; people want to believe this is due to something other than pure chance (or, at least, the complex and unpredictable interaction of many different factors), so they invent various post hoc explanations. Gandalf61 (talk) 13:56, 18 December 2008 (UTC)[reply]
That building is by no means the only one to survive in similar condition (see this, this, and this) - many modern iron, steel, and concrete buildings "survived" (in that they remained recognisable husks). The reason that so few buildings are evident in the post-bomb photographs is because, like the centres of most Japanese cities of the era, most buildings were constructed of wood. By this late in the war the centres of most cities had been gutted by bombing or firebombing (and most of their stock of wooden buildings consumed); Hiroshima and Nagasaki were spared at the behest of the Conant committee (the manhattan project's political sponsors) which had explicitly asked Curtis LeMay to refrain from bombing either city (see Atomic bombings of Hiroshima and Nagasaki#Choice of targets). So we can attribute the extremity of the devastation in both cities to the incendiary effect of both bombs. The memorial building, and those few like it, survived firstly because they were more modern and more solidly constructed (so they withstood the pressures) and secondly because they were constructed of materials that didn't burn. 87.114.130.249 (talk) 18:52, 18 December 2008 (UTC)[reply]

What is the difference between a male seed and a female seed?

Please note, I found this question at CAT:CSD/What is the differance between a male seed abd a female seed?. I've deleted the page, but decided to AGF and redirect the questioning to here. Below is the text from that deleted article -Andrew c [talk] 15:38, 17 December 2008 (UTC)[reply]

Does anyone know what the differance between a male seed in an orange and a female seed in an orange? please write something about it on wikipedia please I need to know because I am trying to grow an Orange tree so please help if you can and i will only take answers that are the most relible to me so please send in thanks alot from your fellow Wikipedia user.


Susan E. Summers User:Dumplin97

Believe it or not, we have a pretty good article on this. See Plant sexuality. Good luck! --Jayron32.talk.contribs 21:06, 17 December 2008 (UTC)[reply]
There's no such thing as a "female" or "male" seed. bibliomaniac15 21:25, 17 December 2008 (UTC)[reply]
Are you sure that's that true? Some Dioecious plants (Holly, for example) have distinct male and female plants - surely they grew from distinctly male or female seeds? Certainly SOME plants (Monoecious and Hermaphrodite) have just one 'sex' that has both male and female flowers - in THOSE cases, there is presumably no sexual difference evident in the seeds. (But I could be wrong - I'm not an expert!) SteveBaker (talk) 22:00, 17 December 2008 (UTC)[reply]
Anyone care to guess what the top results are when you search for male seeds or female seeds? In that case, there is no apparent way to tell the seeds apart simply by looking at them (can't say I'm surprised. Actually I'd be more surprised if the opposite is true) Nil Einne (talk) 13:09, 18 December 2008 (UTC)[reply]
That doesn't sound all that different from animal development, where physical differentiation between the sexes actually comes quite late to the game. It can be several weeks in humans before physical differences exist. --Jayron32.talk.contribs 19:24, 18 December 2008 (UTC)[reply]
Indeed - I'd be very surprised if you could tell the difference between a boy holly-tree seed and a girl holly tree seed without doing some kind of genetic testing on them. However, I believe that a genetic difference must exist inside the seed for these Dioecious plants - it just doesn't manifest itself as a difference in the appearance of the seeds. SteveBaker (talk) 20:43, 18 December 2008 (UTC)[reply]

Researching Invention for child's project

My daughter has to create an invention that will help humanity etc. She wanted to create some kind of shield that people could buy to add to their existing outdoor lighting to block the light so that sea turtles would not be drawn to it, yet allow the lighting to still shine on the deck etc where people would only see it. Has this been already done? I have searched and found lights that come with special lighting for sea turtles but have not been able to find out if an actual shield for garden or light posts have been invented.

Thank you for your help.

Dee Ingham —Preceding unsigned comment added by 76.97.57.168 (talk) 17:20, 17 December 2008 (UTC)[reply]

Use something like this to determine what frequencies of light the sea turtle can see. Then, get a filter to block whatever frequencies the sea turtle sees. The light will be very dim on those frequencies, but bright in all others. -- kainaw 17:46, 17 December 2008 (UTC)[reply]
You might also investigate the various efforts made by astronomers to avoid light pollution. The International Dark Sky organization (http://www.darksky.org/) might also be a source of relevant information. Click on the 'approved fixtures' button to see a list of companies that make outdoor lighting that is less polluting. There are many things people can do to help - certainly employing reflectors behind the bulb to direct the light only in the direction it's needed. Using MUCH dimmer bulbs (our eyes will adapt perfectly well to seeing by candle-light - we don't need multiple 100W bulbs). Use light absorbing (ie flat-black) paint in areas where the light strikes but is not needed - avoid reflections in water surfaces, etc. SteveBaker (talk) 19:22, 17 December 2008 (UTC)[reply]
Typical office lighting where I live is ceiling fixtures consisting of a box with two or four fluorescent tubes inside, and a sheet of translucent plastic that keeps you from seeing the bare tubes but lets the light out freely. However, there is an alternative version where the plastic sheet is replaced with something that only lets light out in a narrow range of directions, the light is concentrated downward on the area below the fixture and if you look at the fixture from another part of the room it does not look bright. As far as office lighting is concerned I personally hate this effect (along with the notion that it is all right to use less light overall), but it sounds exactly like what you want for this purpose. Unfortunately I don't know what these fittings are called or even exactly how they work: my impression is that the sheet is constructed from of a large number of small lenses, each maybe 1 or 2 cm across. --Anonymous, 21:44 UTC, December 17, 2008.

Disseminated Intravascular Coagulation Question

Is there any connection between DIC and long past surgeries?

The case I'm concerned with most likely got started with vasculitis (thank you, Wiki community for the information by the way) but another party was told the DIC resulted from a surgery 20 years ago. The surgery was a vascular transplant (femoral artery was replaced with anothe vessle from the patient). We can find information on the causes of DIC but nothing yet on association between DIC and past surgeries. Thank you for any help in understanding this. - Jill —Preceding unsigned comment added by 71.193.169.115 (talk) 17:48, 17 December 2008 (UTC)[reply]

One strong risk factor for DIC is major trauma, infection or sepsis. Surgery has the possibility to cause all of these but only you will know if it applies to your case or not. The supposed pathophysiology of this is the release of inflammatory or infected tissue into the blood circulation triggers the cytokine network and coagulation pathway. So in conclusion, there could be a link, but I'm pushing the boat out a bit far speculating whether this is the cause or not. —Cyclonenim (talk · contribs · email) 19:37, 17 December 2008 (UTC)[reply]
There's no significant association between DIC and a history of long-past surgery. One might argue that DIC can be a reaction to a foreign body surgically implanted into someone in the distant past, but the surgery you mention above doesn't seem to fall into that class of operation (it used the patient's own tissue rather than, say, a dacron vascular graft.). - Nunh-huh 19:42, 17 December 2008 (UTC)[reply]
Agree with Nunh-huh - Draeco (talk) 01:07, 18 December 2008 (UTC)[reply]

Helium latex Balloon Questions

How much does a standard (in the US) latex balloon weigh, what is a safe volume to fill it up to, and if I fill it up to that volume with the type of helium typically used at dollar stores that sells balloons, what would be the amount of weight that it could lift (or at least cancel out so that it would just float) before it falls?

Any good guesses?

A few of the things we need to know to answer this question is-

1) what is the average volume of a latex balloon in the US, and

2) how much weight does helium offset.

Scofield Boy (talkcontribs)y 19:40, 17 December 2008 (UTC)[reply]

2 is easy: just check the relative densities of helium and air. At STP, you get about 1.1 g/L of lift in dry air. Since volume increases more quickly than surface area, larger balloons will have more lifting power. However, I've got no idea what an average latex balloon's volume is (or even what a latex balloon is). Do you mean the basic inflate-via-lung-power balloons used at birthday parties and the like? — Lomn 20:45, 17 December 2008 (UTC)[reply]
Yeh, that's what I mean. The rubber-like ones. Scofield Boy 21:27, 17 December 2008 (UTC) —Preceding unsigned comment added by Scofield Boy (talkcontribs)
It may not be as simple as Lomn indicates. I've read that "the type of helium typically used at dollar stores that sells balloons" is not pure helium but is diluted with air, because this is cheaper and provides sufficient lift to keep the balloon off the ground (for some hours until the gas leaks out, that is). However, I don't have a reliable source to cite for this. --Anonymous, 21:48 UTC, December 17, 2008.
Also, Lomn's estimate for the lifting power is incorrect. The pressure of the gas inside the balloon is higher than the ambient air pressure because it has to counteract the inward force of the rubber. One reason that the mylar foil balloons work so well is that the balloon itself doesn't have to be stretched much - so lower pressure helium (and therefore, LESS helium) may be used - both saving money and increasing lift
SteveBaker (talk), 17 December 2008 (UTC)
Lomn's estimate is plenty good enough for back-of-the-envelope work, though. The excess pressure inside a typical latex balloon is only about ten percent over ambient atmospheric pressure: [7]. Ten percent higher helium pressure makes for about a 1.5% decrease in lifting power per unit volume — barely noticeable. (Since Lomn only gave his answer to two significant figures, the effect of elevated pressure would probably be lost in the rounding....)
That's not to say that there aren't other benefits to the mylar balloons. The balloon material is typically lighter, giving more apparent lifting power. I believe that metallized mylar is less permeable than latex is to helium gas, giving the balloon a longer floating lifetime. Since the helium isn't under excess pressure, you do still save about 10% on your filling costs, and the balloon doesn't make a scary noise if it bursts. Finally, a lot of people have a natural latex allergy while mylar is hypoallergenic. TenOfAllTrades(talk) 23:09, 17 December 2008 (UTC)[reply]
From personal experience as a kid, a freshly-filled birthday-party balloon has a net lift of between 1 and 4 grams, decreasing to 0 over the course of about 48 hours. --Carnildo (talk) 22:51, 17 December 2008 (UTC)[reply]
Lift is a force and should be measured in newtons (or pounds), not in grams. The statement "a balloon has a net life of between 1 and 4 grams" is incorrect. That balloon has enough lift to maintain a 1 to 4 gram object at equilibrium (or to accelerate it upward at some approximately fixed acceleration). Nimur (talk) 16:23, 18 December 2008 (UTC)[reply]
Thanks for that: that knowledge might come in handy. But the technicalities really don't matter much to me. Carnildo statisfyed my question. I was mostly interested in how practical it would be to lift a certain weight wigh a balloon, and his personal experience was excactly what I was looking for. Scofield Boy (talkcontribs) 19:25, 19 December 2008 (UTC)[reply]
Pounds and grams are both mass units in strict usage (contrary to what some of us were taught in school) and both are commonly used as force units (because here on the Earth, in many practical contexts the distinction is not important and it's easier to work with them). --Anonymous, 23:15 UTC, December 18, 2008.

"Good" Chemical

My teacher has assigned our class to write a letter to a company thanking them for using a "good" chemical. Normally I'd rant about how this is forcing a political opinion on someone, but you have to pick your battles. I'm having difficulties finding a suitable example. Does anyone else know of something?Tuesday42 (talk) 20:05, 17 December 2008 (UTC)[reply]

Everything is a chemical, and an argument could be made for any one of them. What makes a chemical "good" in your opinion? What is something you find useful in your life? What is a time you have chosen one chemical product over another, and why? DMacks (talk) 20:10, 17 December 2008 (UTC)[reply]
I once read a newspaper article lambasting tinned peaches (or somesuch) for having added chemicals - the one the columnist picked out for especial opprobium was ascorbic acid. DuncanHill (talk) 20:16, 17 December 2008 (UTC)[reply]
Personally, I would write to my local water authority. Dihydrogen monoxide is a pretty useful chemical, even if it is highly addictive! – ClockworkSoul 20:21, 17 December 2008 (UTC)[reply]
There are various additives that have value. Calcium in orange juice. Iron in breakfast cereal. Fluoride in water. Iodine in salt. etc.
That would meet the requirement without seeming like a gimmick.
Even additives that people complain about usually serve a useful purpose and could arguably be "good". Commercial bread lasts a lot longer in your pantry than freshly baked bread. You could thank them for adding preservatives.
That may be the point of the exercise, anyway : To get you to understand that you're not living in a Captain Planet cartoon. Nobody uses a chemical just to be evil, they use it because it serves a useful purpose. So just about any substance ever used by any company ever would qualify. It's all in how you frame it. APL (talk) 20:59, 17 December 2008 (UTC)[reply]
DMack's is right, you need to define "good" before you can get anywhere. Do you mean useful? Environmentally friendly? Abundant? Cheap to make? Pretty looking? Nice smelling? Once you've done that, you need to decide what you're comparing it to, since any reasonable definition of "good" will be relative. You need to find a company which is using a "good" chemical as opposed to a "bad" chemical which they could have used. --Tango (talk) 21:02, 17 December 2008 (UTC)[reply]
All of these additives are "good" when considered from one perspective or another - even the melamine that was added to baby formula in China (killing who-knows-how-many kids along the way) - was "good" from the point of view of the evil bastards who added it - it increased the protein content measurement reading and was cheaper than actual protein. From THEIR warped perspective, it was a good additive or else they wouldn't have done it. No matter what chemical you pick, it's going to be good for some people and bad for others. Consider vitamin E to heat treated milk to replace the natural vitamin that got trashed when the milk was pasteurized - that's "A Good Thing" for the health of the people who consume milk - but it must increase production cost somewhere down the line - so that vitamin supplement is pushing up the cost of the milk. People who already get plenty of vitamin E in their diet might well regard the cost of that addition to be "A Bad Thing". So - you may calm your moral outrage with the knowledge that every single one of the chemicals that are added will be good for some set of people and bad for others...so knowing that, you can just pick one and write the letter. SteveBaker (talk) 21:48, 17 December 2008 (UTC)[reply]
I dissagree with that fluoride in drinking water is a good thing. A chemical that is only supposed to help your teeth shouldn't have to travel through your bloodstream to get there. New studies have suggested adverse health effects. ~AH1(TCU) 21:57, 17 December 2008 (UTC)[reply]
Old studies have suggested that too. Which is why we have a whole Water fluoridation article with all sorts of cites:) Any chemical can be problematic, any can be well-intentioned but have what (some, few, many, most, all) (have, do, will someday) deem to be unacceptibly high drawbacks compared to the advantages and available alternatives. Your rationale is either poorly explained or poorly considered among similar routes of administration: "shouldn't have to" only becomes "therefore shouldn't do it" if there's a better way to accomplish the goal. Otherwise people wouldn't swallow antibiotics for localized infections. DMacks (talk) 22:36, 17 December 2008 (UTC)[reply]
Bah. I just knew someone was going to say that. That's sort of my point. Even the additives that are complained about, and may have side effects, can still be considered "good" because they serve a purpose. So, an easy way to complete the assignment was to pick an additive, more or less at random, look up what it's for, and thank whoever put it there for putting it there. Assignment done.
(Of course, Steve took this one step farther and talked about chemicals that kill babies for profit. But no one argued that. Either I'm not very good at communicating my point, or fluoride evokes automatic knee-jerk replies. Or likely both. ) APL (talk) 01:41, 18 December 2008 (UTC)[reply]
DDT has saved millions of lives, and yet water has killed millions more. I'm sure you know which one is a "good" chemical :-D -RunningOnBrains 22:28, 17 December 2008 (UTC)[reply]
You could write to the Stolichnaya folks and thank them for using ethanol. That's what I'd do. Deor (talk) 22:31, 17 December 2008 (UTC)[reply]
... as opposed to using methanol ... 93.132.182.39 (talk) 07:49, 18 December 2008 (UTC)[reply]
I notice that a lot of the answers above equate 'chemical' (in the context of food) with 'additive'. Everything in your food is a chemical or a mixture thereof. Write a nice letter to a manufacturer of sugar for giving us sucrose. Send a note to a table salt company acknowledging their delivery of sodium chloride. Congratulate your local bread company for their brilliant use of polysaccharides, gliadin, and glutenin. All those 'natural' foodstuffs? They're full of evil, dirty 'chemicals' like fructose, or palmitic acid and hydroxytyrosol.
Moving beyond the world of foods, my sarcastic side urges you to thank your local fertility clinic for their efforts with deoxyribonucleic acid. TenOfAllTrades(talk) 23:20, 17 December 2008 (UTC)[reply]
In formal science, it is true that any substance that's made of atoms is a 'chemical'. However, many English dictionaries offer an alternative, less formal definition which is a better fit for what our OP is clearly asking. Wiktionary (for example) provides three definitions:
chemical
  1. Any specific chemical element or compound.
  2. An artificial chemical.
    "I color my hair with henna, not chemicals."
  3. An addictive drug.
We have to assume that this assignment implies the second definition.
Personally - I love the elegant definition in the American Heritage Science Dictionary:
  • A substance having a specific molecular composition, obtained by or used in a chemical process.
The implication being that it's a chemical if it has a simple, pure composition - so pure, distilled, water is a chemical - but sea water isn't. That's a much better fit to the way most of us use the word.
SteveBaker (talk) 23:34, 17 December 2008 (UTC)[reply]
I was more-or-less imagining a strict reading of the first definition with my answer. (ie: Sugar is a "chemical", but Flour is a substance made up of a multitude of chemicals.) My examples were additives, because it would be weird (to me) to thank a company for some particular chemical unless they were using it specifically and purposely. APL (talk) 01:34, 18 December 2008 (UTC)[reply]

Other chemicals to be thankful for : Caffeine, Ethanol. APL (talk) 01:43, 18 December 2008 (UTC)[reply]

I apologize for not specifying what I was asking for. The assignment wasn't quite as ambiguous, it wanted a chemical that a company was using for something that most other companies in the same field weren't. As for the political opinion aside, it isn't really evident when you look at the assignment out of context, but this teacher... let's just say I'm looking forward to the end of this term.Tuesday42 (talk) 03:57, 18 December 2008 (UTC)[reply]

So, to clarify ... you're looking for an example of a company voluntarily switching from some dangerous chemical to a safer chemical that serves the same function even if the safer one is higher cost? APL (talk) 06:19, 18 December 2008 (UTC)[reply]
How about this? I should caution you, however, that since this article was published the FDA has given its official approval so I'm sure other competitors will jump on the bandwagon. 216.239.234.196 (talk) 13:45, 18 December 2008 (UTC)[reply]
Have you checked the websites for big chem companies like Dow, Exxon, Oxy or BASF? I'm sure they have numerous press releases touting how they wuv the planet and how they are doing good? Have you seen the "green" Exxon PR commercials lately? --70.167.58.6 (talk) 20:20, 18 December 2008 (UTC)[reply]
Yeah - "We've found a way to continue to fill the air with Carbon Dioxide even after the oil runs out. Yay! Gas-to-liquid technology! (Oh - and could you please buy some of our stock because we aren't doing so good now that the price of oil has tanked. K'Thnks!)" ...argh! SteveBaker (talk) 20:36, 18 December 2008 (UTC)[reply]

Does anyone else think it's stupid that every definition for the word "chemical" uses the word chemical? Mac Davis (talk) 21:12, 18 December 2008 (UTC)[reply]

Yeah, that pretty much sucks. The condensed OED, here defines it as "noun a distinct compound or substance, especially one which has been artificially prepared or purified." There ya go... --Jayron32.talk.contribs 00:56, 19 December 2008 (UTC)[reply]

Evolution of the liver

Are there any clues as to the original function of the liver? How it could have evolved? What changes in the digestive system would have led to the first primitive liver, and what evolutionary advantages the most primitive adaptations to the digestive system on the way to a liver would have have been given the host creature? Jooler (talk) 21:20, 17 December 2008 (UTC)[reply]

I can't find any references so I'm just going to go on speculation. I think it's likely that in some animal in the past, certain cells started to produce enzymes which broke down certain substances and helped the creature gain an evolutionary advantage by being able to eat something which others could not, because they could not break down that substance. This probably advanced further and further until many enzymes could break down many substances. —Cyclonenim (talk · contribs · email) 22:25, 17 December 2008 (UTC)[reply]
The wording makes it sound like a homework question. We don't do your homework, but we do try to help with specifics and hints. I'm not a biologist, but here's how I would approach this: Look at our articles on the more primitive animal phyla. Based on that, what simple organs with some functions similar to liver functions arise? After you have found some obvious ones, go "back" to "more primitive" phyla to see if you can find homologus organs. You can probably create a plausible evolutionary path. -Arch dude (talk) 01:07, 18 December 2008 (UTC)[reply]
I don't get the homework vibe; what red flags do you see? —Tamfang (talk) 20:02, 28 December 2008 (UTC)[reply]
The vertebrate liver evolved from the hepatic diverticulum of the gut of an amphioxus-like ancestor during early chordate evolution. The amphioxus hepatic diverticulum has been found to synthesize several substances that are similarly synthesized in the vertebrate liver. - Nunh-huh 01:20, 18 December 2008 (UTC)[reply]

@Arch dude. It's not homework. Its a very a long time since I left school. Jooler (talk) 15:20, 19 December 2008 (UTC)[reply]

Copurification, Pull-down assays, and co-immunoprecipitation

What is the relationship of these three terms? My best guess so far is that copurification = pull-down assays, while co-immunoprecipitation is a particular type of copurification/pull-down assay (as well as a type of immunoprecipitation). Is that at all accurate? Unfortunately copurification is currently a red link, and pull-down assay just redirects to immunoprecipitation even though it is mentioned nowhere in the article. Kaldari (talk) 23:20, 17 December 2008 (UTC)[reply]

Looks like your red has magically turned to blue. Isn't it great when a question has positive consequences? --Scray (talk) 12:18, 18 December 2008 (UTC)[reply]

Unfortunately, the link doesn't really address the OP's question. A "pull-down assay" is a generic term for a biochemical protein purification in which a target protein is isolated from a soup of other proteins (usually some type of cell lysate or extract).

"Immunoprecipitation" implies that a specific antibody against the target protein is used to achieve the "pull-down".

"Co-immunoprecipitation" is when you look for other proteins (or in the case of chromatin immunoprecipitation, a piece of DNA) that also purify along with your targeted protein, thus implying a physical interaction between the two entities.

There are also non-antibody means of achieving a "co-purification" including:

  • His-tag, which can be purified with nickel-coated beads
  • GST-tag, which can be purified with glutathione-coated beads
  • Maltose binding protein, which can be purified with maltose-coated beads

Fusion proteins can be engineered to contain the preferred tag -- this is then expressed in bacteria, yeast or cell culture to be used in a "pull-down" experiment to see what else binds to the fusion protein. ---Medical geneticist (talk) 00:28, 19 December 2008 (UTC)[reply]

Thanks. That is very helpful information. Kaldari (talk) 20:26, 19 December 2008 (UTC)[reply]


December 18

Hi guys, an anon added a message to the Brazilian copperfish article saying it was a hoax, and I'm certainly finding it hard to find evidence that it isn't. Any of you guys have a big book of south american fish or something? --fvw* 01:25, 18 December 2008 (UTC)[reply]

I can't find it on FishBase, which is usually pretty good for piscine problems. DuncanHill (talk) 01:46, 18 December 2008 (UTC)[reply]
Good enough for me, AfDed it. Thanks! --fvw* 02:34, 18 December 2008 (UTC)[reply]
The article has no references to check. There are exactly TWO Google hits for the fish (either in English or Portugese) - this Wikipedia page and this odd page www.mydearvalentine.com/valentine-vacations/rio-%20de-janeiro.html (which is blacklisted here on Wikipedia) that uses almost the same odd phrasing:
MyDearValantine.com: "If you are in a mood for fishing then catch a plethora of the famous Brazilian copper fish which are in abundance in the river and makes fishing a popular activity in Brazil."
Our article: "In Rio de Janeiro, one can expect to catch a plethora of famous Brazilian Copperfish."
But that's IT. The fish is not mentioned at on any of the many [8] fishing web sites devoted to the Rio de Janerio area), no hits for the supposed discoverer, no hits for the Genera it's supposed to be a member of. It's supposed to be moderately closely related to the Piranah (according to the classification on that page) - yet the fish in the photo looks nothing like a Piranah. The sole author of the article has been accused of vandalism half a dozen times and has shown no previous interest in working on articles about fish. Given the political turmoil and tension between the urban and rural parts of Brazil in 1936 - it seems a doubtful destination for a field trip by "Nature enthusiasts". The language of the article is peppered with phrases like "Believe it or not" - which (along with the zero references and the utter non-notability of the subject) would be enough to justify deletion even if it wasn't a hoax. Yeah - take it down. SteveBaker (talk) 04:18, 18 December 2008 (UTC)[reply]
We need some sort of "This article was deleted with help from the RefDesk" thingy, like the one for articles we helped improve! DuncanHill (talk) 04:21, 18 December 2008 (UTC)[reply]
For the sake of the record, the (now closed) AfD is at Wikipedia:Articles for deletion/Brazilian copperfish. DuncanHill (talk) 04:27, 18 December 2008 (UTC)[reply]
Actually - I was kinda annoyed at how quickly it vanished - I wanted to see from the edit history whether this user had collaborators or sock-puppets and perhaps thereby track down any other hoaxes from the same source that might have gone unnoticed. The connection to the 'MyDearValantine.com' site is particularly interesting. I also found a company CALLED "Copperfish" who promote web sites by bumping their Google search results...it seemed possible that this hoax article was somehow related to their activities. I was happily doing my Sherlock Holmes thing - when all of the evidence was suddenly evaporated! Oh well...next time maybe. SteveBaker (talk) 20:31, 18 December 2008 (UTC)[reply]
Maybe a friendly admin would email you a copy of the edit history of the article? DuncanHill (talk) 20:33, 18 December 2008 (UTC)[reply]
Looks like almost all of the article text came from User:Vinegar. Kaldari (talk) 23:09, 18 December 2008 (UTC)[reply]
A kindly passing admin gave me a copy of the article - I'll go and play detective for a while. SteveBaker (talk) 04:14, 19 December 2008 (UTC)[reply]
So would you say, on the whole, that the article is (was) fishy? —Tamfang (talk) 18:58, 28 December 2008 (UTC)[reply]
Perhaps it is not a hoax because many fish exist in the Amazon rainforest that are often not documented in western fish encyclopedias such as FishBase and some have not even been scientifically classified as species, also i found one travel website that mentions the Brazilian copperfish as the local delicacy of the city Rio de Janeiro and an additional website that says the Brazilian copperfish is the official food of the Brazilian national team. --Apollonius 1236 (talk) 01:18, 26 December 2008 (UTC)[reply]
  • There's another person on wikipedia who knows the person who made it (I merely know someone who knew the creator). In any case, read the AfD log - it's pretty obvious it's a hoax. --V2Blast (talk) 07:34, 25 January 2009 (UTC)[reply]

cheap lead testing

After reading a previous question about lighting and turtles, I had an idea for a student's school science experiment and I was wondering if it is possible... Every year, there are recalls for toys that have lead paint. It is possible to spend a lot of money to send every toy you have to a lab for testing, but what about home testing? Using only products easily purchased at a common grocery or department store, is it possible to effectively test for lead? Right now, I'm going through the standard cleaners and looking for one that has a chemical that will change colors, produce smoke, or smell real bad if mixed with lead. -- kainaw 13:54, 18 December 2008 (UTC)[reply]

This Consumer Product Safety Commission report says that the home lead test kits you can buy are unreliable, and their manufacturers don't have the "made with common ingredients" constraint that you do, so I imagine the answer is "no". --Sean 15:21, 18 December 2008 (UTC)[reply]
What's really disturbing about that report is that the vast majority of errors are false negatives (50% of the samples that were positive by reference test were missed). A qualitative screening test should always be designed to err on the side of caution, since it can be followed with a more reliable test for a "confirmed result." Given that a consumer is unlikely to even consistently perform the test correctly, they're even more suspect. One point of methodology that may be of interest. The kits given are most likely swab tests. Destructive testing (i.e. grind up the toy) is more likely to give thorough results, though it has obvious problems if you wanted to play with said toy... SDY (talk) 19:32, 18 December 2008 (UTC)[reply]

Could high energy alcohol be used as a jet fuel?

I'm particularly asking about 2-Ethylhexanol. Sure with proper jet engine redesign / modifications. 2-Ethylhexanol should contain more energy per kilogram then jet fuel/gasoline (failed to find numbers using Google, but looks like it). And it even denser. And in comparetion to 9+ carbon alcohols 2-Ethylhexanol have low melting/freezing point. Even lower then jet fuel. Cost of production is irrelevant to this question - just imagine that we can get 2-Ethylhexanol at same or a bit lower price then jet fuel. What would prevent use of it as a fuel for airliners? Vitall (talk) 16:17, 18 December 2008 (UTC)[reply]

I don't know about that particular fuel, but here are some other things to consider for a jet fuel in general:
1) It must have a high boiling point so it doesn't boil off before using it. This is a deficiency in liquid hydrogen.
2) It must not thicken up at cold temps. This can be a deficiency in bio-diesel fuels.
3) In addition to having a high energy density per mass, it also needs a high energy density per volume.
4) The ignition temp needs to be within a certain range. StuRat (talk) 00:17, 19 December 2008 (UTC)[reply]
Safety is another issue - for passenger airliners, they have to add gelling agents to prevent fuel spills from causing major problems. Boiling at low pressures - causing carb icing in prop planes. How safe is it to store, transport, pump. There are just an enormous number of practical problems.

1) Boiling point of 2-Ethylhexanol is 183-185°C. Even higher then Etanol, and FAR CRY from hydrogen(obviously). Boiling point for real life Jet fuel JP-5, MIL-T-5624M, AVCAT is 156-293°C. Essentially the same.

2) As I mentioned before, freezing/melting point of 2-Ethylhexanol are lower then most of jet fuels(-76°C versus -40°C). Liquids tend to thicken up when they come close freezing point.

3) That very strange point. If energy per mass comparable or close and one substance(2-Ethylhexanol) is denser then another(jet fuel), aren't that void your point?

4) Flash point for JP-5: 60°C. "2-Ethylhexanol is a combustible liquid above 60°C.". Essentially same. And there are jet fuels with different flashing points.

Increasing safety by adding gelling agent most likely won't be needed. So called kinematic viscosity of high carbon alcohols are high. But main question - what the problem with adding different agents, if they needed to be added? Why it is not possible to add them, just like we add them in case of ordinary jet fuel? Some very special chemical properties of 2-Ethylhexanol? Or you think something wrong with alcohols? What?

"How safe is it to store, transport, pump. You think with current technology it not possible to safely store transport and pump 2-Ethylhexanol? but industry produce it thousands of tons annually. You think they did not store or transport ethylhexanol??? Or do they do it in a non safe way? What make you think that???? Vitall (talk) 10:23, 19 December 2008 (UTC)[reply]

1) Boiling point of 184°C is a long way from 293°C, so it sounds like your fuel may not work for those applications which result in the most heating of the fuel tanks, such as long, high altitude, high speed flights, at least not without an additive to increase the boiling point. It may still be appropriate for lower, slower, shorter, flights, however.
2) As for viscosity, it's not solely a function of how close you are to the freezing point. Some fuels, like bio-diesel, become unacceptably thick at temps well above the freezing temp.
3) I don't understand what you're saying. My argument is that you must not only keep the plane's weight down, you also need to fit the fuel into the existing tanks to get the same range, regardless of weight. (Although the fuel weight does obviously have an effect on range, too.) So, for example, you couldn't use uncompressed hydrogen gas, regardless of it's energy-to-mass ratio, because it's energy-to-volume ratio is too low.
5) (New item) It must not damage the materials it will contact. For example, ethanol tends to damage rubber. This isn't insurmountable, as all rubber seals can be replaced with silicone, but it is a concern.
6) (New item) It must burn cleanly, not producing tar that will pollute the environment and gum up the engines.
7) (New item) It shouldn't be water-soluble, as that allows large fuels spills on the ground to pollute the water table. The most infamous case of this was the gasoline additive, MTBE.
As for safety, the other poster didn't say that your fuel is unsafe, only that this is a general concern for all fuels. I would add that, in practice, this means that the fuel must not be highly volatile (readily evaporate). If so, any small spill becomes an explosion hazard. Ideally, you should be able to toss a lit match on a puddle of the fuel without it igniting. Only when sprayed through a nozzle should it become flammable. Toxicity is also a concern, as workers are likely to get the fuel on their skin or inhale fumes occasionally. StuRat (talk) 16:22, 19 December 2008 (UTC)[reply]
1) 184°C higher then 156°C. JP-5 is in range 156-293°C. Second Google link for "JP-5 boiling point" claim "Initial point: 182°C". 184°C is almost same as 182°C. And JP-5 is not ONLY jet fuel out there.
2) There are additions that could increase lubricity if needed. And in fact they were used in some blends of jet fuels. Plus there is jelling agents, as someone mentioned above. So adjusting that property should not be a problem(if it need to be adjusted of course).
3) That exact point I have clearly addressed in initial question.
5) That is a problem of ethanol, not hi-carbon alcohols. Anyway, I wasn't in any way suggesting to use existing equipment/engines/etc.
6) During burns alcohols produce CO2 and H2O.
7) 2-Ethylhexanol non soluble in water.
Current jet fuels are complex mixture of hydrocarbons that are toxic by definition. And that include dermatotoxicity. I'm not sure what substance more toxic in relation to each other. And about lit match - that totally untrue for current jet fuels. The only one I could recall had such property, was highly specialized fuel for SR-71(Blackbird). Obviously not in use anymore.
Overall, this looks like general discussion of what jet fuel could/should be. Not exactly my question about 2-Ethylhexanol. Vitall (talk) 23:01, 19 December 2008 (UTC)[reply]
Well, since you know all the properties of that substance, you can judge for yourself whether it meets the criteria for a jet fuel which we describe.
6) Note that many fuels theoretically burn to produce nothing but carbon dioxide and water. However, in the real world they produce complex hydrocarbons and/or carbon monoxide, due to incomplete combustion. StuRat (talk)

I don't know ALL properties, and that is why I'm asking. My primarily source is Wikipedia(different articles) and secondary Google search. I still hope someone with knowledge of 2-Ethylhexanol will step in. Vitall (talk) 17:15, 22 December 2008 (UTC)[reply]

fluorimeter/turbimieter

In the context of measuring the turbidity (haziness) of a liquid, is there any important difference between a dedicated turbimeter and a fluorimeter set to the same emission and excitation wavelength? ike9898 (talk) 16:31, 18 December 2008 (UTC)[reply]

Exoplanet detections in next decade

How precise will the orbital elements and physical characteristics of exoplanets will it be determined in next decade from telescopes or ultra-precise astronomical instruments, such as Terrestrial Planet Finder, Darwin, New Worlds Observer, SIM PlanetQuest, and Overwhelmingly Large Telescope?

Will future telescopes and instruments will determine inclination, true mass, radius, density, surface gravity, temperature, appearance, features, composition, detecting whether it has moons or rings, and others for almost all known exoplanets as of the next decade? Knowing the true mass of exoplanets is important about whether some candidate exoplanets are actaully exoplanets or a brown dwarfs. Knowing the appearance of extrasolar planets is important about how will extrasolar planets be look like, along with moons and rings if detected. BlueEarth (talk | contribs) 18:47, 18 December 2008 (UTC)[reply]

I don't know the answer to your question - but I think the primary goal is to capture enough light from an exoplanet to do spectrographic analysis of the atmosphere. The idea being that we'd have a chance to detect life by finding some 'signature' molecules in the atmosphere. Another goal is simply to make a gross count the numbers of planets of different kinds so we'd know how unique (or non-unique) the earth is. Knowing the other data is interesting too - but beyond simply hoarding information - I'm not sure how it moves science forwards. SteveBaker (talk) 01:06, 19 December 2008 (UTC)[reply]

Expansion of the Universe and Perpetual Motion.

(This is a follow-on from an earlier question which is about to scroll off the top of the desk without getting properly answered.)

User:Tango and I seem to have invented a perpetual motion machine - which is annoying.

Find someplace with a nice hard vacuum and no nearby stars or planets to mess things up. Take two large masses (I'm thinking 'bowling balls') and fling them off in opposite directions at a velocity that's just a little less than the escape velocity due to their mutual gravitation. They fly off in opposite directions and (because they aren't all THAT heavy - and they have to move very slowly to avoid hitting their mutual escape velocity) it takes a l-o-n-g time for them to slow down and start falling back towards each other. During this time, the expansion of the universe is gradually increasing the distance between them - so that on the way back towards the starting point, they have to travel a tiny bit further than they did on the way out. This means that the total momentum when they eventually collide is a little more than the momentum we had to give them to launch them off in the first place. Since that extra momentum could be harvested by various means, we get a tiny bit of energy for free...from nowhere.

(We can make a proper perpetual motion machine out of this by using rubber bowling balls which bounce off of each other and head back out again at just the right speed...I'm going to boldly assert that the extra energy we pick up from the expansion of the universe is set up to be exactly enough to overcome annoying things like drag from the interstellar hydrogen and the energy lost in the collision).

Could someone please help us figure out why it can't work? (Or failing that - tell me where the "OFF" switch is - all of these bouncing rubber balls are REALLY distracting?)

SteveBaker (talk) 21:56, 18 December 2008 (UTC)[reply]

The expansion of the universe does NOT add momentum to the balls. Momentum is universally conserved even in an expanding universe. Thus, your make an incorrect assumption that they will return to the same point with the same energy as they left. They will have less energy at collision than you sent them off with because the increasing distance between them means that there will be a corresponding decrease in net gravitational attraction (as integrated over the time of travel) than would exist in a "nonexpanding" universe. The increase in distance should be exactly offset by the decreasing gravitational attraction due to that distance increase, and we return to a nice, zero-sum universe again. QED. --Jayron32.talk.contribs 22:15, 18 December 2008 (UTC)[reply]
I thought about that - it's OK to boldly assert that momentum is conserved - but how exactly? The universe expands - and to avoid the ball having so much momentum - it has to...what? If I drop a stationary ball from 10 meters - it'll hit the ground faster than one I drop from one meter. I don't see a way to deny that. So if we consider the upward trip of a ball thrown into the air as merely a time-reversal of the downward journey - and we know that the upward leg was a shorter distance than the downward leg because of the expansion of the universe during the upwards and downwards trips. Sure we can ASSERT that it must all sort itself out - and I'm sure it does - I just don't see how. SteveBaker (talk) 00:56, 19 December 2008 (UTC)[reply]
Ok. Look at it this way. Why do the balls slow down and start moving back in again? Gravity, right? What is the strength of gravity dependent on? The distance between the objects. Right? So what happens if the distance between the objects increases because of the expansion of the universe? Then the strength of the gravitional force bringing them back together decreases by the same factor. Therefore, the acceleration of the balls towards each other will decrease by exactly the same factor as the rate of expansion of the universe. Thus, they will still be moving at the same speed at collision (assuming of course a spherical cow, erm, a perfect vacuum) as they were at the start of the experiment. Which means the universe is still a zero sum game. All is well, and we can sleep at night... --Jayron32.talk.contribs 01:18, 19 December 2008 (UTC)[reply]
Please ignore everything that Jayron has said here, as I would go to far as saying it completely lacks any truth. If you consider that the distance has expanded, and that the masses remained the same then the acceleration will be exactly the same at any given distance between the two objects, but the integral (which relates to the kinetic energy of the two particles) will be slightly larger due to the increment increase is the distance. Also on another note, when you increase the distance between two masses by some factor, the acceleration does not decrease by the same factor as gravity is an inverse square law. Philc 0780 01:29, 19 December 2008 (UTC)
Let's try this a third time.
  • Lets consider each effect in isolation. Acceleration refers to an objects change of motion such that its position (distance from starting point) varies with the square of time; that is each successive arbitrary unit of time causes the distance traveled in that time to increase by a constant amount over that time. So if it is accelerating at 1 m*s-2, after one second, the object has moved 1 meter. After two seconds, it has moved 3 meters, after three meters it has moved 6 meters, etc. etc. Using a little calculus, we find that the time to cover a given displacement decreases with the square root of that displacement. If we increase the distance between our bowling balls, their time to cover that increased distance will be relative to the square root of the distance we added.
  • Now, lets consider the effect of gravity. As Phil has already astutely noted, gravitational force (which is the source of our acceleration) is an inverse square effect, thus adding some arbitrary distance between the bowling balls will decrease the force of gravity between those balls the square of the distance we add between them.
  • If we consider these two effects on the velocity of balls at our hypothetical "zero" point (i.e. that point we pushed them apart from), lets compare the situation to the "non-expanding" universe. In that situation, the balls should arrive back at the collision point moving the same speed we initially imparted on them. Any recoil energy they get from the collision will be identical to that, so our situation (ignoring friction, and assuming they are moving in the same dimension) will be a perfect harmonic oscilation, with the two balls always moving the same velocity at the instant before the collision point. So what happens in the "expanding" universe. We already determined that the added distance (lets call it x) has decreased the gravitational force by a factor of x2 and that the additional distance traveled will mean that we are accelerating by increasing the amount of time required to cover that distance by a factor of x1/2. Last time I checked, the square root of a square was no change at all, so the balls "act" like no distance has been added. They will strike each other with the same velocity as if the added distance had not been added at all, and we have our nice little harmonic oscilator back again. The universe is safe from perpetual motion, and we are back at our zero sum... --Jayron32.talk.contribs 04:46, 19 December 2008 (UTC)[reply]
Your first point depends on the assumption of uniform acceleration (and even then your numerical examples are wrong). Neither that assumption, nor the conclusion that time follows x1/2 is correct. In fact, the time is proportional to x3/2 for an inverse square force. Dragons flight (talk) 06:03, 19 December 2008 (UTC)[reply]
Are you measuring the distance between the objects as a fraction of the size of the universe? I so this is a very odd thing to do and is clearly the source of the problems. And if you aren't, then why do you assert that its expansion will have affected the distance between the balls. And you are and you assert that it does, by what measure has the distance increased? (or to say, what thing would not have been affected by this expansion) and so the entire reference frame is stretched equally, and so within the frame no stretch at all has occurred. Philc 0780 01:25, 19 December 2008 (UTC)
Why do I assert that the distance between the balls has increased? Because that's what happens to galaxies - they move away from us...why doesn't a bowling ball do the same thing? (That's a rhetorical question - of course it does the same thing - but by a TINY amount - so we don't notice.) SteveBaker (talk) 02:21, 19 December 2008 (UTC)[reply]
No, they don't move apart. See below. Dragons flight (talk) 05:50, 19 December 2008 (UTC)[reply]
Steve, I think you are essentially asking whether conservation of energy holds, given the the expansion of the universe. I believe your original interpretation (perpetual motion) is tantamount to saying to "energy conservation is inconsistent with an expanding universe." I don't believe that is the standard and generally accepted point of view that most cosmologists hold, but this is most definitely a major un-answered question. See accelerating universe, which clearly needs some expert attention. It is my opinion that there is a great deal of uncertainty regarding some of the astronomical observations upon which some of the basic theoretical claims are made (for example, it is my opinion that quantitative rates of acceleration of the universe expansion are awfully complex derivations from fairly sparse and noisy sets of observational data). Others who specialize in cosmology may disagree with my assessment. Nonetheless, there is some theoretical uncertainty about just which physical "laws" (such as energy conservation) are really universally applicable. Nimur (talk) 04:54, 19 December 2008 (UTC)[reply]
If the local space is empty, then the distance between the balls does not expand. Hubble flow describes a solution for the passive evolution of approximately uniform matter under the action of general relativity. However it does not say that the distance between every pair of objects under all possible conditions must expand. In the future, galaxies will be moving away from each other, because they are moving away from each other now, and because they were moving away from each other in the past. The expansion of the distances between galaxies is a natural consequence of their dynamics and occurs in the absence of external forces (strictly speaking I am ignoring dark energy). However, if you take a uniform, isotropic, and expanding universe, and then cut a spherical hole in it to create a region of truly empty space, it follows from general relativity that the expansion occurring outside that hole has no impact on the matter within it. (This is essentially GR's version of the shell theorem.) In particular, you can put a galaxy, solar system, or even a pair of bowling balls in that region of "empty space" and they will not experience Hubble flow. The assumption of locally empty space is incompatible with the assumption of Hubble flow. The alternative is that you are embedded in a medium experiencing Hubble flow and hence necessarily have an opportunity to interact gravitationally with the mass comprising that medium, which implies they you can exchange energy and momentum with it. Dragons flight (talk) 05:50, 19 December 2008 (UTC)[reply]
That would certainly solve the problem. It's not something I've heard before, though - how does that tally with the idea that it isn't the matter moving outward but rather the space between them getting bigger? The space between two galaxies is essentially empty, so how come the galaxies move apart? --Tango (talk) 14:55, 20 December 2008 (UTC)[reply]
They move apart because they have a relative velocity. There's only one kind of relative motion in general relativity, and it's responsible for both the separation of superclusters and the separation of rubber bowling balls. They were given an initial "push" (by inflation or SteveBaker) and they keep going because of inertia. It's not necessarily wrong to give the two cases different names, but it's like the difference between a tadpole and a frog: the one can be continuously deformed into the other, and any terminological cutoff is going to be arbitrary. -- BenRG (talk) 07:50, 21 December 2008 (UTC)[reply]
But that's movement *through* space, rather than space expanding, I thought the whole point was that it's space expanding, not matter within space moving. --Tango (talk) 01:05, 24 December 2008 (UTC)[reply]

If you visualise the gravitational field/potential energy contours around the bowling balls it might make things a lot easier. The bowling ball/s start off with an initial velocity which takes them up to a certain contour before they fall back. If they are always given the same initial velocity, they will always reach the same contour (which might now be further out, if an expanding universe is to be believed), but the total kinetic energy gained is dependent only on the potential energy difference between the starting energy contour and the furthest contour reached, and those two shells will remain the same even if the geometry has stretched. In other words, whether the universe expands or not is irrelevant, and gravitational fields remain conservative. Rotational (talk) 08:56, 24 December 2008 (UTC)[reply]

But the potential energy is a simple function of distance, so if the universe expands the potential energy should increase. --Tango (talk) 23:06, 25 December 2008 (UTC)[reply]

If the geometry remains static, then the potential energy would be a simple function of distance, but you are postulating an expanding universe, so that gravity is steadily diluted and as a result the contour intervals are constantly increasing. The balls have further to go before they stop, since the gravitational gradient is flatter. To use a model, you've lowered the inclined plane slightly allowing the ball to go further, but the final height it reaches above the starting plane is going to be the same. ciao Rotational (talk) 14:27, 26 December 2008 (UTC)[reply]

Mathematical Modeling

Regardless of the final solution, I'm curious as to how one would go about calculating the time evolution of such a system. Assuming a symmetrical system of two masses m, each moving away from a stationary reference gnat at position x=0 with velocity v, it was relatively straightforward to calculate for the non-expanding case. Using Newton's second law and Newton's law of gravitation, the time evolution of the position of one of the masses should follow giving us the second-order ordinary differential equation , with . However, I have no clue on how I would even start to incorporate the metric expansion of space into such a calculation. I surmise this is because I'm trying to use the tools of classical mechanics to work on a relativistic problem. How would someone versed in general relativity approach calculating this problem? -- 128.104.112.113 (talk) 16:15, 19 December 2008 (UTC)[reply]

I should think that Newtonian Mechanics would not be able to model such a system as one of the features of newtonian mechanics is the conservation of energy, something which this hypothetical system attempts to violate. —Preceding unsigned comment added by 92.3.10.246 (talk) 13:44, 20 December 2008 (UTC)[reply]
There is a Newtonian version of big bang cosmology and you can derive a version of the Friedmann equations within it (see here). You can probably get some insight into the original question from this model. Two test particles of negligible mass moving collinearly (no orbital angular momentum) in a Newtonian big bang will satisfy
where r(t) is the distance between the particles and ρ(t) is the density of the dust (ordinary and dark matter). If you also include a 1/r2 force between the particles (hereafter "balls") then you get
where m is the total mass of the balls. Unfortunately I don't think this can be explicitly solved even for simple choices of ρ(t) (like ρ(t) = 0), but you can get a sense for the solutions by starting at the point of maximum separation (where r'(t) = 0) and extending forward and backward in time. If ρ(t) = 0 then the equation is time-reversal symmetric, so the future and past halves of the trajectory look the same. That means that in an empty de Sitter universe (which is roughly what we're destined for in the future according to ΛCDM), the two rubber balls will neither gain nor lose speed with each bounce. In an expanding universe with nonzero density, ρ(t) is larger for the past half of the trajectory and the attractive force is correspondingly larger, so the balls will collide sooner and with greater speed in the past direction. In other words, the balls lose speed with each bounce in this scenario. They would gain speed in a contracting universe. So where does that energy come from (or go to)? I don't think there's any great mystery here. The balls are simply getting a kind of gravity assist from the dust. They will in turn (Newton's third law) disturb the motion of the dust, which I ignored in my model. If I hadn't ignored it, the apparent violation of the conservation laws would presumably disappear. -- BenRG (talk) 07:18, 21 December 2008 (UTC)[reply]

December 19

Getting a Shine On

Why do the planets of the solar system, seen from earth, ahine like incandescent stars? Also, why does the earth's moon, seen from earth, shine rather like a fluorescent lamp? The astronauts who walked on the moon did not see the surface shining like that. Things are supposed to get dimmer as we get farther away. Instead, the planets and the moon get brighter. – GlowWorm —Preceding unsigned comment added by 98.17.46.132 (talk) 00:12, 19 December 2008 (UTC)[reply]

The light is simply reflected sunlight - the planets and moons (including Earth and our Moon) reflect sunlight - and against a dark sky - they look bright. I disagree that the astronauts walking on the moon didn't see the surface shining - they certainly did. That's how they could see the ground around them! When you stand in your back yard - looking at the grass - what you are seeing is light reflected off the surface of the earth (your lawn)...when you stand on the moon - it's the same deal. When you're in orbit around the moon? Same deal. When you are on the earth looking up at the moon? Same deal.
The business of things getting dimmer as they get further away is a bit more complicated. Light from a square meter of the surface of the moon reflects an amount of light into your eyes that's inversely proportional to the square of the distance you are from it. When you get twice as far away - you see four times less light from that square meter. BUT (and this is the important thing) as you get further away, each square millimeter of your retina is being lit up by more square meters of the moon - because perspective is making the moon look smaller. These two effects PRECISELY cancel out. That's why the grass on a distant hillside looks just as bright as the grass on the lawn you are standing on.
Now (and this is where it gets complicated) - this effect where the light from each square meter gets less and less - but more and more square meters pile up to push light into your eye - only lasts until the object starts to get very small. When it's small enough, all of the light from the object is reaching your eye already - now, as you get further away from it, the light from each square meter gets less - but you're already percieving all of the square meters already - so now the object starts to get dimmer as it gets further away.
The moon is so close that the 'cancelling out' effect is still working - so the full moon looks just as bright from here on Earth as it does when you're standing on it. (Well - except that our atmosphere blocks some of the light).
I'm not explaining this as well as I'd like to - but I hope you can follow what I'm saying. SteveBaker (talk) 00:36, 19 December 2008 (UTC)[reply]
(edit conflict, great explaination, SteveBaker)Hi. The reason is that the planets and moon reflect light from the sun. For example, the moon's surface is as dark as ashphalt. In fact, it only reflects about 8% of the sun's light. As you get closer to the Moon, its surface magnitude becomes higher, meaning the "density" of the brightness is less as it is more spread out. Looking down at an asphalt road on Earth on a clear day does not look bright, but if the entire Earth was covered in asphalt, then you may be able to see the reflection as ALL of the asphalt is now visible. Almost any surface will have some degree of reflection, and since the moon and planets are large objects, being able to see the entire object will make it appear brighter. However, if you were looking at the moon from say Mars, it would appear much dimmer than from Earth as it is farther away. See also albedo. I'm sure someone with a better explaination will be soon to come along, however. Hope this helps. Thanks. ~AH1(TCU) 00:41, 19 December 2008 (UTC)[reply]
(EC with SteveBaker and AH). Get in a really dark room, with no light at all. Hold a white ball infront of a black piece of paper. Do you see it? No? That's because the light is not on. Now, turn on the light. See the ball? Why? Cuz the light from the room is bouncing off of it. Now, the solar system is a REALLY BIG ROOM and the moon and the planets are still just balls, and the sun is a REALLY BIG LIGHTBULB. Same deal. --Jayron32.talk.contribs 00:43, 19 December 2008 (UTC)[reply]
Also, field of view plays into this as well. If you are standing on the moon looking straight down at the ground, you see only about two degrees wide clearly (a degree is a way of measuring how big something looks, a fist held at arm's length looks about 10 degrees wide). However, your periphial vision allows you to see about 160 degrees horizontally and about 100 degrees vertically. When you're standing on the moon, not only are you only able to see a small patch of the Moon if you're looking around, you can only see an even smaller patch at any one time. (If it's night on the moon, you would have a much harder time seeing, unless the Earth is lighting your sky [see Earthshine ]. You wouldn't be able to see the nighttime portion of the moon during the day and just barely at night.) Since from the Earth, you can see the entire moon with the naked eye without having to look around, you see the entire moon reflecting light into your eyes. ~AH1(TCU) 00:49, 19 December 2008 (UTC)[reply]
With regard to grass on a distant hillside looking just as bright as the grass at a person's feet, that is talking about two different things. The grass seen on a distant hillside has a far greater area than the grass seen at a person's feet when the angular span is the same for both views. A bigger object reflects more light. In contrast, an object of a given size seen from a distance will be less bright than the same object seen close up. When the same object is at a distance, less light energy is received by the eye. GlowWorm —Preceding unsigned comment added by 98.17.46.132 (talk) 06:05, 19 December 2008 (UTC)[reply]
We have an excellent article on albedo - the extent to which an object diffusely reflects light from the Sun. It says that the average albedo of the Moon is around 7%. This is much lower than the albedos of fresh snow (80%-90%), desert sand (40%) or green grass (25%), so an astronaut walking on the Moon will perceive the surface as rather dull. Now if the Moon's albedo were much higher than this, say about 30%, then it would reflect 30% of the sunlight falling on it. As the Moon is almost exactly the same angular size as the Sun, this would make moonlight from a full Moon on a clear night 30% as bright as direct sunlight - so a moonlit night would be as bright as a dull overcast day (clouds reflect about 70% of incident sunlight). This is clearly not the case, so a low albedo figure for the Moon is reasonable. The only reason the Moon appears to be especially bright in the night sky is by contrast against a dark background. Gandalf61 (talk) 14:52, 19 December 2008 (UTC)[reply]
Here is my explanation of this phenomenon. Everyone has seen a diagram of how light emanates from a point source. The diagram shows lines radiating fanwise from the point. Actually, of course, the lines are imaginary. But four of the lines, adjacent in 3 dimensions, will strike an imaginary spherical shell at any given distance from the energy source. The shell is centered on the energy source. The 4 lines and shell portion form a pyramid with a rounded base. As distance from the source increases, the total light energy encompassed by those 4 lines is spread over a larger base. So the energy decreases on each unit of area on the base (such as one square millimeter). Also as distance increases, within each additional increment of distance the pyramid comes closer to being a tube with a square cross-section. Therefore at a great enough distance, as distance increases further, there is very little change in light intensity on the aforesaid square millimeter. This can also be thought of in terms of a cone becoming more like a round pipe. As distance increases, the area of the image decreases at a slower rate than total light intensity decreases. (The image is the base of the pyramid or cone). Therefore an image becomes brighter as well as smaller. (An image is made up of a great many points issuing or reflecting light. The light from each point follows the principle described above.) Eventually, at a great enough distance, a crossover point is reached at which the rate of decrease in light intensity is insufficient to increase the brightness. The image then becomes dimmer and dimmer as distance increases. — GlowWorm —Preceding unsigned comment added by 98.17.46.132 (talk) 16:04, 19 December 2008 (UTC)[reply]
P.S. I just made some changes near the end of the above explanation, then changed it back again. It's tricky to think about. I'm still not happy with the explanation. – GlowWorm —Preceding unsigned comment added by 98.17.46.132 (talk) 18:27, 19 December 2008 (UTC)[reply]

The sky is falling! Positive or negative feedback mechanism?

Hi. Apparently, as the troposphere warms as a result of global warming and greenhouse gases, the upper atmosphere, ie. the stratosphere to the ionosphere, is cooling rapidly. This is causing the upper atmosphere to shrink and move toward the ground. Since the extra air would be pressing on the troposphere, the part of the atmosphere that affects our weather, would this be a positive or negative feedback, or neither? Would it be a positive feedback since the extra atmosphere would increase the pressure in the troposphere and thus increase its heat content and also trap in the existing greenhouse gases, or would it be a negative feedback by absorbing the Sun's heat before it reaches the troposphere and by absorbing tropospheric heat as it attempts to radiate into space? In a comment on wunderground.com, it appears a 17C cooling has occured in the ionosphere, affecting satellites as the atmospheric pressure there has decreased by 2-3%. Could this be related to the gigantic chunks of ice falling from a clear sky reported by Discover magazine? Could this also be why the air temperature over parts of Antarctica have apparently cooled? Does the cooling of the stratosphere also decrease stratospheric ozone? Thanks. ~AH1(TCU) 01:26, 19 December 2008 (UTC)[reply]

Gas contracts when it is cooled, but a given volume does not become heavier. So a cooler stratosphere does not press more heavily on the troposphere. The stratosphere just becomes thinner. —Preceding unsigned comment added by 98.17.46.132 (talk) 02:47, 19 December 2008 (UTC)[reply]
Worse still - the troposphere is getting warmer - it's going to expand - surely that's going to push the upper atmosphere higher?
So what is this mechanism that's making the upper atmosphere cool? Is it because CO2 is trapping the sunlight in the lower atmosphere - reducing the amount of sunlight travelling back out through the upper atmosphere? SteveBaker (talk) 04:10, 19 December 2008 (UTC)[reply]
I was at the American Geophysical Union Fall Meeting 2008, and (being among other things, a researcher of ionospheric physics) I happened to spend a good amount of time in the Space Weather and Climate Modeling sessions. There are hundreds of thermodynamic models for global climate. Some only model ocean temperatures. Some model solar flares. Some model electric field interactions between low-altitude and high-altitude winds. Some generate statistical aggregates of empirical observations. A search for "troposphere climate feedback" in this fall's meeting abstracts generated over forty results - that is, forty new-as-of-this-Fall scientific approaches to that specific problem ([9], for example). The point I am trying to make is that it is the research scientist's choice to model the effect you describe as positive or negative feedback and there is not currently much scientific consensus (let alone, predictions or model-to-data matching). Nimur (talk) 05:10, 19 December 2008 (UTC)[reply]
:-), also at AGU. Dragons flight (talk) 06:06, 19 December 2008 (UTC)[reply]
If the climate had positive feedback it would quickly spiral out of control. Considering that it didn't, I think we can safely say the feedback is generally negative. Of course, since there is vastly more than one degree of freedom, positive and negative feedback is a gross oversimplification, but at least we can say that in the last five billion years, the climate never got so bad it wouldn't go back. — DanielLC 17:42, 19 December 2008 (UTC)[reply]
It is a common misunderstanding that a positive feedback implies a runaway feedback. A positive feedback chain can be understood iteratively as x1 = x0 + a*x0, x2 = x1 + a*x1 = x0 + a*x0 + a2*x0, ... If a is less than 1, the geometric series , and one has a finite and controlled positive feedback. It is only when the response to a perturbation is larger than the perturbation itself that one has a runaway feedback, i.e. a > 1. The climate system has a great many positive and negative feedback, and in fact many of the most important are positive feedbacks, but obviously those feedbacks aren't large enough to cause the climate system to break down. Dragons flight (talk) 18:03, 19 December 2008 (UTC)[reply]

how to reduce alcohol or menthol smell but keep the cool function

I need to know how to reduce the alcohol/ menthol smell but keep the cool function.

By answer to my e-mail : email removed—Preceding unsigned comment added by 124.121.5.186 (talk) 04:18, 19 December 2008 (UTC)[reply]

Drink smoothies.--Shantavira|feed me 08:58, 19 December 2008 (UTC)[reply]
You got an answer when you asked this a few days ago: Wikipedia:Reference_desk/Science#the_method_to_reduce_alcohol_or_menthol_scent. We aren't going to send it to your email. ~ mazca t|c 14:35, 19 December 2008 (UTC)[reply]

IMMUNOLOGY

Which is the most potent antigen presenting cell(APC) for naive T cells? Which is the best antigen presenting cell? —Preceding unsigned comment added by 117.196.65.28 (talk) 07:42, 19 December 2008 (UTC)[reply]

Our antigen-presenting cell article addresses this question directly (regarding the most potent APC for naive T cells). For your second question, "best" is not specific enough to answer directly because there is no single characteristic to define "best". --Scray (talk) 11:45, 19 December 2008 (UTC)[reply]

Conjoined twins longevity question

Why do conjoined twins have short lives? 124.180.116.201 (talk) 08:43, 19 December 2008 (UTC)[reply]

Did you read conjoined twins? Many of them lead long and fulfilling lives.--Shantavira|feed me 09:00, 19 December 2008 (UTC)[reply]
Well, yes - but an awful lot of them die at birth or within a very short time afterwards. The average longevity has to be a lot worse than for normal twins. The "why" of that is likely to be different depending on how they are conjoined - but very often internal organs are shared or highly distorted or even compressed against bones that "shouldn't be there". SteveBaker (talk) 22:49, 19 December 2008 (UTC)[reply]

Demineralised water

Today I bought some demineralised water to top up my car battery. It said on the bottle, 'not for human consumption'. Now, I don't plan on drinking it, but if I did surely it would be fine wouldn't it? —Preceding unsigned comment added by 58.165.239.250 (talk) 11:39, 19 December 2008 (UTC)[reply]

Yes. What they most likely meant is that their production facility did not applied and not certified by food safety/quality control organs/bureaucrats. Vitall (talk) 11:50, 19 December 2008 (UTC)[reply]
Distilled water is safe to drink, but if you only drink distilled water, you can die of thirst. --131.188.3.20 (talk) 13:34, 19 December 2008 (UTC)[reply]
Do you have a cite for this? There are a lot of myths about distilled water. APL (talk) 14:02, 19 December 2008 (UTC)[reply]
Pure distilled water is rather horrible-tasting but I've never seen any evidence to suggest that you could die of thirst if you drank only that. Certainly you'd miss out on some of the minerals you normally get from water, but that's a minority of your daily mineral intake anyway. Our article on Purified water contains a long and waffly section about the benefits and drawbacks of drinking it (with a neutrality tag too!). To add to the original question, I agree that it's more likely to be an ass-covering safety precaution - there's possibly an increased risk of bacteria, etc, in the water if it hasn't been prepared and stored with drinking in mind. They'd rather not get sued if someone does drink it. ~ mazca t|c 14:33, 19 December 2008 (UTC)[reply]
Isn't the issue that it would explode your cells like distilled water on an amoeba will ? That is, the water will travel through the cell walls until the level of dissolved material is equal both inside and outside the cell, at which point there will be so much water inside that the cell walls will breach. The water would, however, mix with dissolved material in your saliva and stomach, so it wouldn't be pure distilled water once it contacts the stomach walls. However, if you drank enough, perhaps it would still be close enough to distilled water after this mixing to cause a problem. StuRat (talk) 16:07, 19 December 2008 (UTC)[reply]
No, an amoeba swells or bursts in distilled water because it evolved to live in slightly salty water. The fact that it's distilled water is irrelevant: fresh water would have the same effect, as our amoeba article correctly says. It's different with humans, as we evolved to drink fresh water. We don't care about the slight difference between fresh and distilled water either, except for the taste. Distilled water won't make our cells explode any more than fresh water will. Mine's a pint. --Heron (talk) 17:51, 19 December 2008 (UTC)[reply]
Humans have more salts in our cells than there are in fresh water, although less than ocean water. One result of this is how our skin wrinkles up when we take a bath in fresh water. StuRat (talk) 18:46, 20 December 2008 (UTC)[reply]
I do not see why the salt and other minerals and ph balance would not be supplied by other food. The International Space Station has a water recovery system which distills urine, and they will in the future use that for part of their drinking water. The article says nothing about possible danger from drinking the distilled water. We do not provide medical or nutritional advice, but a Google Book search [10] will show many books, some sounding quacky, advocating the drinking of distilled water. Men on ships have been drinking distilled water for 150 years [11] , [12]. "The Doctor's Heart Cure" (Sears, 2004) oppositely cautions against drinking distilled water, saying "Long term use of distilled water can lead to mineral deficiencies that can cause heart beat irregularities and hair loss." He cites "Day, C., 'Why I say no to distilled water only,' Health and Beyond Weekly Newsletter,' reprinted at [13] , but that site says the page is no longer available. Pros and cons of drinking distilled water are discussed at "Plain talk about drinking water," page 97 (2001) by Symons, published by the American Water Works Association. Any results from medical/nutritional textbooks from respected publishers such as university presses, or recent reliable peer reviewed scientific publications? Note: the article Purified water says that "demineralized" or "deionized" water may have had the mineral salts removed by ion exchange resins, which do not remove organic molecules, viruses, or bacteria, so it could be full of things which could make you sick. It is not steam distilled. Edison (talk) 18:03, 19 December 2008 (UTC)[reply]
But even if distilled water doesn't have minerals and stuff that you need - that doesn't mean that the water will harm you - it's the lack of minerals in your diet - which could be gotten in a bazillion other ways. That's like saying eating bananas will kill you because they don't have enough vitamin C. (Actually, I have no idea whether bananas have vitamin C or not - but I'm sure you understand my point). I'm pretty sure that true, pure distilled water is very safe to drink - but in all likelyhood, what was in that bottle of stuff you got to put into your car is no longer that pure. Aside from anything else, it's sure to have picked up impurities from the bottle it's sold in - and it's very possible that they used some kind of toxic substance as the mould release agent or something. The labelling criteria for stuff that goes into cars isn't as strict as for food items - so it may not actually be pure distilled water at all! After all - "High Octane Gasoline" doesn't have any more "Octane" in it than regular stuff - it's other additives that produce the EFFECT of higher octane levels. SteveBaker (talk) 21:44, 19 December 2008 (UTC)[reply]
Partially distilled water, ie. previously boiled water, is usually safe to drink. However, if you drink too much water that does not contain any electrolytes, you can die from water poisoning. ~AH1(TCU) 21:53, 21 December 2008 (UTC)[reply]
But you can die from that with mineral water, water from your kitchen tap - or distilled water. That's not something that's specific to distilled. SteveBaker (talk) 17:15, 23 December 2008 (UTC)[reply]

beta sisteral

what is beta sisteral —Preceding unsigned comment added by 71.199.15.172 (talk) 12:35, 19 December 2008 (UTC)[reply]

I believe you are looking for beta-sitosterol. -- kainaw 13:40, 19 December 2008 (UTC)[reply]

Symbol for Litre: uppercase or lowercase?

In a recent response to Density (follow on from the above), one of the respondents used the symbol "ml" for millilitre, which I am always tempted to correct (I find "mL" to be much less ambiguous, and consider "L" to be the preferred symbol for liter). I decided to check my facts, at least superficially, and was somewhat surprised to see that the lead to our Litre article states that "l" (lowercase) is an accepted abbreviation for litre. Of course there are other sources to check, and I did find that in the U.S. the accepted abbreviation for litre is "L", at least according to NIST. To me, given the importance of legibility in many fields, using uppercase "L" is a no-brainer. I would be interested in any comments from the RD (sorry if this is a FAQ - I'm relatively new here). --Scray (talk) 12:36, 19 December 2008 (UTC)[reply]

My experience is that 'l' is the only acceptable abbreviation. Litre#Symbol indicates this is a national difference. Algebraist 12:41, 19 December 2008 (UTC)[reply]
I'm quite ashamed that I did not read down that far on the Litre page. Thank you - that settles this question for me. --Scray (talk) 12:54, 19 December 2008 (UTC)[reply]
Algebraist, I see you study at Oxford and am guessing you are English? Although l and ml are often used in shops, etc. At University we use L/mL and l/ml synonymously. --Mark PEA (talk) 13:54, 19 December 2008 (UTC)[reply]
Typically, units which aren't named after people have lowercase symbol, but for the litre an exception was made to allow L as well as l, as the latter looks like the digit 1 in some fonts. -- Army1987 – Deeds, not words. 19:48, 19 December 2008 (UTC)[reply]
According to International System of Units: "The American National Institute of Standards and Technology recommends that "L" be used instead, a usage which is common in the US, Canada, Australia (but not elsewhere). " - so everywhere else on the planet, stick with 'l'. But the odd thing is that the litre is neither an SI base unit nor (to my surprise) one of the 'approved' derived units (ie it's not in the standard list of 20 derived units in SI derived unit). So I think the "correct" thing is simply not to use it and to stick with either decimeter-cubed: dm3 or (more likely) 10-3m3. That's backed up by Litre which says pretty much the same thing. Obviously, this only matters in strictly formal settings - everyone who uses SI will be very clear on what is intended by either 'l' or 'L'. SteveBaker (talk) 21:35, 19 December 2008 (UTC)[reply]
Shouldn't be any real surprise there. The liter is not and never will be a part of the SI, because the SI is a "coherent" system of units (each unit is a unitary combination of the base units; the unit of volume is the base unit multiplied by itself three times). That unit, the cubic meter, could have a name that could become part of SI, but the name sometimes used for it has not been made a part of SI.
The liter is officially recognized among the units acceptable for use with SI. That's the most it ever will be; it won't be a part of SI. But even after the introduction of SI, the CGPM has had three different resolutions dealing with the liter, IIRC.
You are obviously too young to have gone through the punishment some of us had to deal with, in the pre-1964 days when we had to learn the difference between a liter and a cubic decimeter, or between milliliter and a cubic centimeter, which then differed by about 28 parts per million. That was the result of an ill-advised redefinition of the liter in 1901, causing a couple of generations or so to waste a lot of time learning a distinction that usually didn't matter at all in any real-world measurements. Gene Nygaard (talk) 08:42, 22 December 2008 (UTC)[reply]
And, in fact, when the International System of Units was introduced in 1960, the liter/litre wasn't even among the units acceptable for use with SI. At that time it didn't even have a powers of 10 relationship to the base units. It was only after the 1964 restoration of the definition as a cubic decimeter that the liter was declared acceptable for use with SI. Gene Nygaard (talk) 08:44, 22 December 2008 (UTC)[reply]

Symbols for the litre (CR, 101 and Metrologia, 1980, 16, 56-57)

Resolution 6

The 16th Conférence Générale des Poids et Mesures (CGPM),

recognizing the general principles adopted for writing the unit symbols in Resolution 7 of the 9th CGPM (1948),

considering that the symbol l for the unit litre was adopted by the Comité International des Poids et Mesures (CIPM) in 1879 and confirmed in the same Resolution of 1948,

considering also that, in order to avoid the risk of confusion between the letter l and the number 1, several countries have adopted the symbol L instead of l for the unit litre,

considering that the name litre, although not included in the Système International d’Unités, must be admitted for general use with the System,

decides, as an exception, to adopt the two symbols l and L as symbols to be used for the unit litre,

considering further that in the future only one of these two symbols should be retained, invites the CIPM to follow the development of the use of these two symbols and to give the 18th CGPM its opinion as to the possibility of suppressing one of them.


(And derived units are products of powers of SI base units; the category in which the litre, the minute, the hour, etc. are is "Non-SI units accepted for use with the International System of Units".) -- Army1987 – Deeds, not words. 00:15, 20 December 2008 (UTC)[reply]

Per Wikipedia:Manual of Style (dates and numbers): "For reasons of legibility, the preferred symbol for the unprefixed liter is upper-case L." --—— Gadget850 (Ed) talk - 01:03, 20 December 2008 (UTC)[reply]

Burt Rutan comments on V-22 Osprey and 1960's tiltrotor prototype

I recall reading an interview with Burt Rutan, designer of Spaceship One and other revolutionary aircraft. In this online article, he had some negative comments about the V-22 Osprey. He compared it unfavorably to a prototype tiltrotor aircraft developed in the 1960's or thereabouts. I believe it may have been developed by Bell. Anyway, Rutan argued that the old prototype was sigificantly superior to the modern Osprey in various flight characteristics, and especially in durability and ruggedness.

I have run extensive Google searches, trying to locate this article. Can anybody recall where this article might have appeared, or where I might otherwise find Rutan's comments and views on the Osprey? Thank you. —Preceding unsigned comment added by 69.255.102.84 (talk) 13:15, 19 December 2008 (UTC)[reply]

optics

newton propounded corpuscular theory of light and he is against the wave theory. but an experiment known as newtons rings is a popular one which is considered as proff of wave theory. to me this is a contradiction. newton gave corpuscular theory, but the experiment in his name is proving the wave theory. explain this paradox. —Preceding unsigned comment added by 118.95.105.71 (talk) 14:36, 19 December 2008 (UTC)[reply]

Just because it's called Newton's Rings doesn't mean Newton designed it, it was designed to test Newton's idea, or that even if it was designed to do so, that it didn't wind up proving him wrong. Names are just names:) DMacks (talk) 16:35, 19 December 2008 (UTC)[reply]
Meh. Neither the corpuscle theory or the wave theory are particularly correct; they both have some aspects that explain some behaviors of light. Light is light, however, and that it sometimes behaves like a wave and sometimes like a particle is not lights problem. Its ours... --Jayron32.talk.contribs 17:59, 19 December 2008 (UTC)[reply]
This is another good example of Stigler's law of eponymy in action. --Bowlhover (talk) 18:43, 19 December 2008 (UTC)[reply]
According to Newton's rings, Newton was the first to analyze the phenomena (but not the first to describe it). Dragons flight (talk) 18:51, 19 December 2008 (UTC)[reply]
"Light is light, however, and that it sometimes behaves like a wave and sometimes like a particle" is little misleading. To explain some of its behavior, we use wave theory and to explain some other behavior we use particle theory. It is not that when light is wave it is not particle. Light is both particle and wave, see Wave–particle duality. manya (talk) 06:41, 20 December 2008 (UTC)[reply]

How the heck does a car move?

Or to be more precise: given the power and torque curves, gear ratios, wheel and tyre sizes etc. of a standard Honda S2000, launched from a full-ball standing start and only gearing up at 9,000 rpm each time, what would the displacement-, velocity- and particularly the acceleration-time graphs look like?

Asked in a different way (what I'm REALLY asking): how does the power, torque and gearing on a car combine to translate into motive force? Say we compare three cars of equal power, a 177 kW 208 Nm high-revving Honda S2000 with its standard (short) gearing, a 177 kW 300 Nm Seat Leon Cupra with the Honda's gearing, and then the same SEAT with its standard gearing. Would be interesting to compare the 3 cars plotted against each other for displacement, velocity and acceleration.

For the purpose of the exercise ignore things like air resistance (and then add it back in once we're done with the simple case), as well as tyre slippage and drivetrain losses etc. and assume the cars are the same mass. And of course linearise the power curves piece-wise to make the calculus easier.

I got into a horrible tangle trying to determine the power as a function of time. It is a function of RPM clearly. The RPM is a function of velocity and gearing. Velocity v = ds/dt. However P = dW/dt where work W = F x s. Force F = ma = m d2s/dt2. You can see it gets horribly complicated at this point. Help! Zunaid 14:56, 19 December 2008 (UTC)[reply]

It would have thought our article vehicle dynamics would provide an overview, but it doesn't. There is some information in our Wikibooks Subject Automotive Engineering, but I don't know if that's what you need. So, short of actually spending years studying automotive engineering, I would recommend buying a good old-fashioned paper book about it. — Sebastian 20:14, 19 December 2008 (UTC)) corrected 20:20, 19 December 2008 (UTC) [reply]
Our vehicle dynamics article is just a list it seems, and the Wikibook doesn't cover these topics unfortunately. Zunaid 13:23, 21 December 2008 (UTC)[reply]
Well, the thing to remember is that the engine, the transmission and the wheels are all connected - if we assume no slippage in the clutch and no wheel spin then the RPM and the Speed are linearly related for any given gear ratio - you can look up the exact ratio of the gears in your gearbox in most decent car spec sheets (or maybe in the owner's manual) - but be careful because there is also gearing in the differential and there may be another fixed gear in the engine someplace after the tachometer...you need to multiply all of those ratios together to get the relation between the RPM of the engine and the RPM of the drive axles...and from that and the circumpherence of the tyres you can go from engine rotations per minute to wheel rotations per minute and from that to distance travelled per minute and then miles-per-hour. So you can stop thinking about RPM and speed separately (except when shifting gears).
Furthermore, torque-at-the-wheels and force are also linearly related for any given wheel diameter (again, assuming no wheel-spin). So you can take the standard torque-at-the-wheels-versus-RPM graph that you get out of a dynamometer and use that to directly calculate linear force versus speed in each gear. Dyno curves for pretty much any car can be found from an enthusiast somewhere - or you can take your car to a dyno center and for some outrageous amount of money (less than $100 - but more than $50) you can get that graph. OK! Now where getting somewhere!
Now we need to turn force into acceleration - and it's tempting to grab our old friend F=Ma and say that the accelleration depends on linearly on the force for a given mass - which is more or less constant except for plus fuel and passengers. That would be OK if there was no friction and no drag and the car was on level ground...and I know you're happy to ignore those things. But sadly, you can't...not if you want to get anywhere close to the right answers. Air resistance particularly is an enormous force. At 100mph it's by FAR the biggest force...and (critically) it's what limits the top speed of most cars. In my MINI Cooper'S - the rev limiter cuts in in 5th gear at about the top speed of the car...if it were not for air resistance, I could shift into 6th gear and go even faster...but when I do that, the car actually slows down because there isn't enough torque in that gear to overcome air resistance at the top speed of the car.
Air resistance is simply not a reasonable thing to ignore because at anything much over about 30mph it BY FAR dominates the results you're going to get.
OK so now what? Well, you need to know the drag force on the car and the amount of rolling resistance. The coefficient of drag for most cars is available in the spec sheet you get from the car dealership or the manufacturer's web site. You also need to know the viscosity of the air (Wikipedia is your friend!), the frontal cross-sectional area of the car (you can estimate that fairly well as the height times the width of the car) and a few other things. This can be plugged into the equation in Drag (physics) to get you an equation relating speed to the drag force.
So - at any given speed - the forward acceleration is:
  • The force we got from converting the torque/RPM curve into a force/speed curve
...minus...
  • The drag force comes from the equation in Drag (physics)
  • The rolling resistance (friction) in the transmission, tyres, etc. (We'll talk some more about this in a moment).
This gives us a NET force at any given speed, for a particular gear.
OK - so NOW we can grab a hold of F=Ma and figure what happens if your foot is on the floor at some given speed. It would be nice to toss some math in here and get a graph - but sadly the torque versus RPM curve isn't a nice mathematical function - it's a weird shaped mess that's different for every car on the planet. (And it's different in winter and summer - on one brand of gas versus another, etc).
So using math to plot the position or speed of the car versus time requires some ikky small-time-increment numerical integration.
For that we can use a computer. We can say what the speed is now - plug that into the math and get out the acceleration now. Then we can say that over the next (say) millisecond, how much will the car accellerate - we can use that to get the new speed one millisecond into the future - and plug THAT into the same pile of math to get the speed 2 milliseconds from now - and so forth.
This computer program is something I've written for my MINI and it works great - it fits reality very accurately. However, there are a couple of gotcha's. One is that you have to shift gears - that takes time - during which the torque from the engine drops to zero - the force from the engine goes away and drag and friction start to kill your speed until you hit the next gear. Assuming you're driving smoothely, it's reasonably save to say that you matched the RPM's correctly - so all you need to do is to switch the gear ratio in the bowels of all that math - and take away the engine torque for as long as it take you to shift.
So - you do all that and you get a 0-60 time that looks quite a bit better than the manufacturer's spec sheet (if they are being honest) - and when you try to make the car do that - it won't. That's mostly because you need practice at actually driving the car like that - but with a lot of highly disciplined effort - you can get the car to do EXACTLY what the computer says it will. Hooray! Simulation works!
There is one more 'gotcha' I missed. What is the rolling resistance and the friction in the car's drive-train? This is never written down on any car spec sheet and there isn't a reasonable way to calculate it...so now what?
Well, here is what you do. You take your car - get it up to a reasonable speed then stand on the clutch - the car will slow down...and down...and down...and eventually stop. The amount of time that takes is dependent on the drag and the friction in the drive train/tyres/etc. We can plug the numbers for the free-wheeling car into our car simulation software and get an answer for how long it would take for the car to slow down from (say) 60mph to (say) 30mph if there was ZERO rolling resistance (ie just air resistance)...and we can plug in different values for the rolling resistance until we get an answer out that matches what we see in the real car. Sadly, this number is not linear with speed (at least it isn't in my car) - so you have to measure how long it takes the car to go from (say) 100mph to 90mph, from 90mph to 80mph...and so on all the way to zero mph. This is tedious (and, on open roads, downright dangerous!) - but if you are determined - you can do it and plot a graph of friction versus speed.
But since you're doing that - you can actually just use the graph of speed versus rate of slowing down with the clutch pedal pushed down (the deceleration) to directly measure the sum of rolling resistance, drive train friction AND wind resistance at a whole range of speeds. Knowing that deceleration curve (and the mass of the car) - you can produce an empirical measurement of the net forces on the unpowered car as a function of speed - so you can forget about all of those ugly drag calculations (which is good because some of the things like the coefficient of drag and the cross-sectional area are hard to calculate directly).
Now you can factor that into your math.
Yes - this isn't easy. Sorry about that! But I did it - and it works.
SteveBaker (talk) 20:18, 19 December 2008 (UTC)[reply]
Thanks Steve (I somehow suspected you'd turn up with a detailed answer. Thanks!). The S2000's torque curve is fairly flat (i.e. constant) in two sections (before and after VTEC at 6,100 RPM) so should lend itself quite nicely to being defined piece-wise as two constant functions (of which the first half only applies in 1st gear anyway). Otherwise numerical integration would be the more accurate way to go. I was just hoping to do some juicy differential equations but alas the problem is more practical than that as you've explained. Mmmm okay so I can't leave the drag out, that's a drag ;) There are a few at-the-wheels dyno charts of an S2000 online which I shall grab. I also know the speed vs RPM for each gear which I've already determined emperically, which would take account of all the "internal" gearing between the engine and the wheels. Would you be willing to give out your code? I'd love to play around with a few numbers and see what comes out. Have you by any chance produced graphs of the displacement, velocity and acceleration vs time using your program? Would be interesting to see them. Also, I think vehicle dynamics could do with your input. At the moment it is just a list article. Zunaid 13:23, 21 December 2008 (UTC)[reply]

Mysterious Music Box Malfunction

A music box in my house, which has not been wound-up in at least a year, went off on its own a few days ago. It played one chorus and then stopped. I don't know anything about the inner mechanisms of music boxes, but I'm curious as to what can cause this sort of malfunction. Any ideas?

It was by itself in a corner of a room and no one had been near it all day, so it was undisturbed by exterior forces. So there's probably something wrong with the mechanisms inside it. It's about a decade old. "Music box" may be a misnomer since it's not shaped like a box. It's actually a large snow globe on a stand that I assume houses the musical component, and sticking out of the stand is the key-shaped metal thing that you turn to wind it up and make it play. Maybe something (liquid? confetti?) from the snow globe leaked down into the music box part and set it off?

After it went off on its own we tested it and it still plays just fine when wound up. 96.233.7.70 (talk) 20:29, 19 December 2008 (UTC)Perplexed[reply]

Most music boxes are powered by a mainspring similar to some mechanical watches and clocks. My guess is that temperature changes caused either expansion or contraction of the Mainspring, which caused the music box to fire off. No need to alert Craig T. Nelson or anything... --Jayron32.talk.contribs 21:06, 19 December 2008 (UTC)[reply]
There has to be some kind of ratchet inside that holds against the tension of the mainspring - when that fails, all the energy in the spring is released at once. If it was already close to being wound down - this event might release enough energy to play a chorus or so before it utterly wound down. Why it chose that moment to come loose is anyone's guess - temperature changes - vibration - who knows? I have a large "30 day" clock at home (you only have to wind it every 30 days) - it has two mainsprings - one for the chimes and one for the movement. A few months ago, I'd wound up one spring and was in the process of winding up the other when the first mainspring's ratchet failed in some way - the key rotated amazingly quickly - and the 'wings' of the key whacked me across the knuckles several times - it was enough to draw blood and left me with a finger that was so badly bruised that I thought it was broken...fortunately it wasn't but I had to keep it in a splint for 10 days anyway. Never underestimate the power of a clockwork motor! SteveBaker (talk) 22:55, 19 December 2008 (UTC)[reply]
Another culprit could be a drop in humidity alone, or in conjunction with a temperature change. Humidity makes things "sticky", including increased friction that might have caused the motion to stop near the end of the winding, and drying out could decrease the friction enough to allow it to play a bit more. Humidity is especially important with wood and lacquer components. StuRat (talk) 01:44, 20 December 2008 (UTC)[reply]

Thanks so much. :-) 96.233.7.70 (talk) 20:45, 21 December 2008 (UTC)NoLongerAsPerplexed[reply]

Time delayed bombs in World War 2 had clockwork mechanisms to delay the detonation for some period such as 24 hours, to increase the terroristic value of the weapon. Many of these stopped running before detonation, often when they were nearing the end of the period and the clockwork was just starting to press on the detonator trigger. Decades later, it was common for one to resume its ticking and detonate, either because someone fund it while excavating for a building, or just because a truck hit a bump on the nearby road, or because of a spring thaw. Similar minor jarring or temperature changes could cause the music box mechanism to run a bit longer, to more melodious effect. Edison (talk) 21:02, 21 December 2008 (UTC)[reply]

December 20

Eye Layers

The eye has 3 liquid to semiliquid layers over the sclera and cornea: tear, mucus, and what is the 3rd one?96.53.149.117 (talk) 01:48, 20 December 2008 (UTC)[reply]

You linked to all those articles and yet didn't bother to read them? Nil Einne (talk) 05:05, 20 December 2008 (UTC)[reply]
tldr. Why do 99% of people ask questions here?96.53.149.117 (talk) 05:15, 20 December 2008 (UTC)[reply]
Too long didn't read? I found the answer to your question within 1 minute. No one said you had to read them all from top to bottom, a bit of common sense and use of the find function that nearly any browser has goes a long way. As to what other people do, quite commonly a lot of questions are not answered so simply and clearly in one of our articles. While there's no doubt some people do ask a question without reading or searching, those questions are often ignored or given less then straight answers, as in this case. As the header says, many people can't be bothered helping those who show no sign of helping themselves and you will often get an answer faster if you make the minimum of effort to look it up. (As it stands, it's taken 3+ hours to get an answer you could have had in a minute). If you want, I could explain how I found the answer within a minute although I would recommend you seek further help since basic search skills are vital in this modern day and age Nil Einne (talk) 05:31, 20 December 2008 (UTC)[reply]
yes, that is what tldr means. What amazes me is that you even take the time to make an inflammatory comment at the start. And considering your net savy, you had to ask for clarification of the initialism?96.53.149.117 (talk) —Preceding undated comment was added at 05:58, 20 December 2008 (UTC).[reply]
This is all too OT for me. Replied to your talk page Nil Einne (talk) 08:05, 20 December 2008 (UTC)[reply]
For the benefit of future readers, the answer is in tears Nil Einne (talk) 08:38, 20 December 2008 (UTC)[reply]

Question on Perception

Are the theories stated in this video true? http://www.youtube.com/watch?v=AqnEGu8VF8Y —Preceding unsigned comment added by Ncgnells (talkcontribs) 01:51, 20 December 2008 (UTC)[reply]

I managed to watch about five minutes of it and skimmed another ten. I heard no theories, just a garbled rehash of the problem of perception vs. reality. (The motivational "music" drove me half nuts, or I would have watched the whole thing out of curiosity.) The producer of this video is relying on the same thing that storytellers rely on: if you don't mention a thing, the audience won't think of it. The problem with saying that the fact that reality is an illusion because all we can know is what we perceive with our senses, that dreams are as real as our waking world, is that the universe, reality if you will, was here before you and will exist after you. Your perception is your reality, and the illusion is that that means that reality is your perception. It's just a sort of sleight-of-mind trick on the part of a confidence trickster or astral prankster. I'm going to go do some math now to get the taste of that baloney out of my mouth. --Milkbreath (talk) 02:53, 20 December 2008 (UTC)[reply]
Yeah, I got about 5 minutes through, and then called it quits myself. For some situations, perception can affect reality (like, say, the placebo effect), or your perception of a situation can cause to to react in a certain way, which will then change the reality of your world. But the bullshit that "everything you see is just what your mind made up, and none of this is here" is a useless philosophy. Consider a sort of "athiests" version of Pascal's wager: If we take two positions: either what you see is reality, or what you see is all made up in your mind and reality does not exist.
If you bet on the former position (that reality is really real!), then you live life as though everything you see is real, and if you are right, your life goes fine. If you are wrong, then there is no harm, since your mind is just making it up anyways, so there are no consequences.
If you bet on the later position (that there is no difference between the real world and the imagined, and that its all about perception anyways), and accept it, then there are no consequences for any of your actions, so you can go through your perceived life doing whatever you feel like, and you don't have to worry about any of the destruction you cause to the real world, since who cares, cause none of it is real. If you are WRONG about this, however, you can cause lots of real damage to yourself and to others.
The moral is that you might as well live your life as if there really is an objective reality outside of your senses, because if you are wrong there is no downside. If you live life as the opposite perspective, which is to say that reality does not exist outside of your own perception, being wrong has a HUGE downside. So it only makes sense to believe in the former position, regardless of what "Truth" really is. --Jayron32.talk.contribs 04:12, 20 December 2008 (UTC)[reply]
Haven't checked the video, but from your descriptions it sounds as though someone over-simplified John Archibald Wheeler and his late theories are way out of mainstream. 76.97.245.5 (talk) 12:41, 21 December 2008 (UTC)[reply]
"Reality is that which, when you stop believing in it, doesn't go away." – Phillip K. DickThe Hand That Feeds You:Bite 01:25, 22 December 2008 (UTC)[reply]

Milk

Why drink milk after ingesting SOME (certian) toxins?96.53.149.117 (talk) 02:07, 20 December 2008 (UTC)[reply]

Such as Draino; what I mean is that on some of the safety labels, they will say, "If ingested give a glass of milk and call a physician.".96.53.149.117 (talk) 03:56, 20 December 2008 (UTC)[reply]

Milk has a mild buffer effect, meaning that it will not only dilute the acid/and/or/base but it will also tend to neutralize it. Pure water will only dilute it, and for particularly strong acids or bases, dilution is not usually enough. The same volume of milk compared to water will have a slightly greater nutralizing effect. Mind you that a) its not likely to be much of a difference and b) if you just drank drano, you just turned your insides into soap. A glass of milk is unlikely to make a huge difference at this point... --Jayron32.talk.contribs 04:03, 20 December 2008 (UTC)[reply]
Also, milk contains fat (except skim milk), so helps to dilute fat-soluble poisons, while drinks without fat will not. StuRat (talk) 04:31, 20 December 2008 (UTC)[reply]
Actually, that may be a bad idea, since the fat in milk may actually help increase absorbtion of fat-soluble compounds into the bloodstream... --Jayron32.talk.contribs 06:18, 20 December 2008 (UTC)[reply]
(wild speculation warning) That was my train of thought, too. But maybe the idea is to absorb fat-soluble compounds and hold them in the milk suspension until the stomach could be pumped. But sodium hydroxide is water soluble (although it would be soluble in milk also). Some other possibilites - Milk might coat the stomach better. Also, with Drano (assuming the amount was fairly small), maybe the milk would provide a substance other than you to undergo saponification (just don't use skim milk). But I've seen "drink milk" on other types of chemicals as well. (Side question: is water is absorbed more quickly than milk? That could be a reason) --Bennybp (talk) 07:10, 20 December 2008 (UTC)[reply]
Water must definatly be absorbed faster than milk. Milk is a suspended solution.96.53.149.117 (talk) 01:47, 21 December 2008 (UTC)[reply]
Also, that IS my other question. What about other chemicals? I really didn't mean, and didn't want, this question just for draino.96.53.149.117 (talk) 01:47, 21 December 2008 (UTC)[reply]
Water passes through the stomach about five times faster than milk. Someguy1221 (talk) 10:40, 20 December 2008 (UTC)[reply]
I think the greater thing is the same reasoning behind having kids hide under their desks during a nuclear attack. If they just printed "If you drink this, there's not much you can do for yourself. Go ahead and call an ambulance, but you're likely a goner" on the label, it wouldn't look too good. The idea is that people feel psychologically better doing something, even if that something is entirely ineffectual at solving the problem, like taking off your shoes at the airport or hiding under your desk to somehow save you from a 10 megaton blast from a nuclear bomb... --Jayron32.talk.contribs 03:04, 21 December 2008 (UTC)[reply]
Taking off shoes at an airport? What does that do? But I disagree with you about putting something on a label for the sake of reassurance. They didn't choose milk for any reason.96.53.149.117 (talk) 04:51, 21 December 2008 (UTC)[reply]
Taking off shoes at an airport? What does that do? Make Dr. Scholl happy because of many added athlete's foot cases. The label designers would also have to consider the added complication of finding something that is considered readily available, even if less effective than some not so common aid. Everyone knows pretty fast where to find their milk. Baking soda or vinegar or whatever could lead to a time consuming search. So a little now is better that a lot, too late. 76.97.245.5 (talk) 12:29, 21 December 2008 (UTC)[reply]
Drinking milk is Security theater? -- JSBillings 15:12, 21 December 2008 (UTC)[reply]
When I was younger a chlorine tank (which my neighbor who ran a pool cleaning business was storing illegally) burst in my neighborhood. I started coughing uncontrollably, the poison control center told me to drink some milk and the coughing subsided. I never really gave any thought to why that worked before, but now I'm confused. -- Mad031683 (talk) 23:37, 22 December 2008 (UTC)[reply]

Why do young women giggle all the time?

And I am not just talking about when I am around. Though they seem to do it even more when I am. —Preceding unsigned comment added by Trevor Loughlin (talkcontribs) 06:15, 20 December 2008 (UTC)[reply]

If you are talking about teenagers, I would suggest that the onset of puberty leaves them with with more highly charged emotions in general at times, and that therefore they experience joy, giddiness, whatever you want to call it, a lot more. It also *seems* that they also face more "crises" than boys, from what I've seen, so they cry more and such, too, at that age. However, it may only seem that way because females, at least in Western culture, are often more open about showing their emotions.Somebody or his brother (talk) 14:25, 20 December 2008 (UTC)[reply]
Indeed, for good or bad western society tends to demand stereotypically masculine behavior from young men. This generally does not include giggling, therefore boys tend to restrain their emotions more. Young woman may also be influenced by equally stereotypical depictions of females in the media, which often tend to fall along the lines of valley girls. --S.dedalus (talk) 04:51, 21 December 2008 (UTC)[reply]
I strongly disagree with that explanation. So far as I know, almost every culture's girls express emotions more strongly than do its boys. You can't explain that away using only stereotypes, which have no reason to remain constant across societies that developed nearly independently of each other. --99.237.96.81 (talk) 07:37, 21 December 2008 (UTC)[reply]
Perhaps it's related to the fact that the start of puberty often starts earlier in female humans than in male humans? Also, the changes in women occur much faster than in men, and the hormonal changes often lead to an imbalance of neurotransmitters (according to the puberty article) which would make one more emotional. -- JSBillings 15:10, 21 December 2008 (UTC)[reply]
While stereotypes may be the wrong word, I don't think you can really rule out cultural factors. For starters, I don't think its true that cultures developed largely independently. A lot of cultures have been quite interconnected for a while. Some like the indigenous Australians for example were isolated for a long while but I doubt we know very well whether they have traditionally shown the trends of females displaying emotions more strongly then males. They didn't have written records and I don't know whether their oral records will really tell us such minutae accurately. There may be records of how they behaved from earlier European encounters but those would have been through the eyes of thoese people and I'm doubtful comparing female and male displays of emotion would have been something they observed in depth unless for whatever reason it seemed highly unusual to them which I doubt it would have been even if their displays were similar. Written records do exist for less isolated cultures, e.g. East Asian ones but given they were less isolated it's hard to rule out cross-cultural influences. Besides has anyone ever looked into the records in detail to find out if displays of emotion by females was stronger then by males? Most significantly perhaps the cultural factors like men as hunters and providers and females as care givers developed a long time ago and are common in the vast majority of cultures. There are obvious genetic and biological reasons why these developed but the important point is that it's easy to see differences in displays of emotions arising out of this, without any real biological or genetic reason (presuming as I've stated these differences are really unanimous throughout tradiotional cultures which I'm not sure we we know). Nil Einne (talk) 11:54, 22 December 2008 (UTC)[reply]

Artificially recreating the earth's magnetic field

At some point in the distant future, the earth will lose its magnetic field because its core cools to a point that the current of molten iron will stop flowing. Without its magnetic field, the earth's atmosphere will be stripped away by solar wind. It is impossible to tell if humans will still be around at that time, and if they are, what kinds of technology will be available. However, based on what we know about physics today, are there insurmountable obstacles to replacing the earth's natural magnetic field artificially? —Preceding unsigned comment added by 98.114.98.236 (talk) 08:32, 20 December 2008 (UTC)[reply]

Atmosphere will not be stripped away. The field has reversed many times, and during each reversal there was a time when the field was zero, but nothing like that happened. There will be many more cosmic rays reaching the Earth, increasing the frequency of spontaneous mutations in DNA, but not as much as to endanger the survival of the human species. -- Army1987 – Deeds, not words. 14:29, 20 December 2008 (UTC)[reply]
I read about the atmospheric loss as a consequence of magnetic field loss in a science article. The theory sounds plausible to me, but I'm not a geophysics expert. Maybe the loss occurs slowly enough that the temporary loss of magnetic field during a geomagnetic reversal only cause a little atmospheric loss. However, if the earth's core cools down, the loss will be permanent; even slow loss will accumulate. In another article that I've read (one about an upcoming global helium shortage), it is stated that when helium floats to the outer edge of the atmosphere, it escapes into space. That tends to lend credence to what solar wind can do to the atmosphere when there's no magnetic field to mitigate the effects. --98.114.98.236 (talk) 14:58, 20 December 2008 (UTC)[reply]
What is the basis of your claim that the atmosphere will not be stripped away? The questioner is not asking about a very short period of null magnetism. This question is about a permanent state of no magnetic field - for centuries and centuries - until the Sun expands and swallows the Earth. Looking around, planets without magnetic fields have all the upper atmosphere stripped away. Are you claiming Earth is special and the Sun will be extra gentle to make sure none of the upper atmosphere is harmed? -- kainaw 15:03, 20 December 2008 (UTC)[reply]
The upper atmosphere? Maybe. But all the atmosphere sounds implausible to me. Also, the other planets you refer to have weaker gravities (the Earth has the greatest acceleration due to gravity of all terrestrial planets in the solar system). -- Army1987 – Deeds, not words. 15:17, 20 December 2008 (UTC)[reply]
According to Solar wind, it only stripped away one third of Mars's atmosphere. In four billion years. And Mars is farther from the Sun than the Earth is. And its atmosphere was much less dense than that of Earth. -- Army1987 – Deeds, not words. 15:24, 20 December 2008 (UTC)[reply]
Exactly - the upper atmosphere is easy for solar winds to strip. That is what the questioner was asking about. Refusing to answer the question because some heavy gasses might survive is being a bit of an ass. The question is: Is it possible - with any technology that could ever be developed - to create an artificial magnetic field around the Earth. I personally do not believe it is. We create fields like that with electricity. The amount of current we'd need to pump through the Earth to get that big of a field would do more harm than good. If a new way to create magnetic fields was invented, it could be possible. -- kainaw 18:40, 20 December 2008 (UTC)[reply]

← You're right that I didn't answer the OP's question (I apologize), but I was pointing out that the scenario he described is implausible. (And it is the "some heavy gasses" that we most care about, as we breathe them, not the hydrogen and helium which tend to go up.) Also, I don't think that the Earth's core is going to freeze over within a reasonable amount of time (where by "reasonable" I mean "before mankind will be very likely to be extinct for some other reason"). I suspect he's watched The Core recently and took it too seriously. -- Army1987 – Deeds, not words. 20:38, 20 December 2008 (UTC)[reply]

Indeed. When we see these events in isolation, like "Someday, the moon will escape the earth's gravity and drift off into space" or "The earth is slowing down and some day it will stop turning on its axis" or "Someday the core will cool to the point when it is solid" ignores the fact that none of these events happens in isolation. Remember the most important event, that the sun will essentially swallow the earth in 5 billion years or so (see Stellar evolution); all of these scenarios require timespans SIGNIFICANTLY longer than that. So, no, the earth's magnetic field is not going away at any time, unless being vaporized by the expanding sun counts as destroying the magnetic field... --Jayron32.talk.contribs 02:55, 21 December 2008 (UTC)[reply]
By the way, the Moon won't drift off into space, it will gain angular momentum until Earth's rotational period equals the period of the Moon's revolution, and at this point it will be still within the Hill sphere (less than 600000 km from Earth). Icek (talk) 08:28, 21 December 2008 (UTC)[reply]

What about Venus? It has no internally-generated magnetic field but a dense atmosphere and lower surface gravity than Earth. Icek (talk) 08:28, 21 December 2008 (UTC)[reply]

The atmosphere of Venus is primarily CO2. There is a touch of nitrogen and not much of anything else. On Earth, the atmosphere is a good mix of nitrogen and oxygen. They are not comparable. Gravity alone can hold heavy CO2 to the planet. The question is: Did Venus used to have lighter elements in the atmosphere? If so, where did they go? If they went into space, was it lack of gravity, solar ray stripping, or a combination of both? -- kainaw 20:54, 21 December 2008 (UTC)[reply]

Flash point of fission in atmosphere

What is the flash point of fission in the atmosphere or the hottest temperature in the atmosphere before there are adverse reactions? Tiailds (talk) 08:35, 20 December 2008 (UTC)[reply]

For the first, impossibly high (if such a concept even works). Spontaneous uranium fission almost never happens, and that only in ideal underground conditions. Sustained fissioning of atmospheric elements, which we don't produce even in labs, can be safely ignored as a threat to humanity. Certainly your latter question doesn't require such extreme conditions. Air temperatures over 100°C would be catastrophic. Temperatures just 10° greater than those at present (perhaps even less) would be sufficient to melt the bulk of the ice caps. But the whole planet turning into a giant nuclear bomb? That doesn't even meet the threshold for bad science fiction. — Lomn 14:41, 20 December 2008 (UTC)[reply]
(Spontaneous fission in general is not that uncommon. Self-sustaining chain reactions almost never happen. There's a big difference there. An atom or two fissioning—who cares. It's only a chain reaction where that becomes an issue.) --98.217.8.46 (talk) 15:56, 20 December 2008 (UTC)[reply]
(And before someone else mentions it, there were, of course, some concerns that nuclear reactions set off by the first atomic bomb would accidentally induce a fusion chain-reaction in the atmosphere, turning the planet into a giant hydrogen bomb. But the problem was studied carefully and it was determined that it was not at all likely to occur. It turns out self-propagating fusion reactions are pretty hard to induce even in ideal situations.) --98.217.8.46 (talk) 16:01, 20 December 2008 (UTC)[reply]
I think the last thing is what the original poster was asking about, rather than a "before someone else mentions it". Anyway, you've provided a link. --Anonymous, 18:50 UTC, December 20/08.

Why do some children have the concept of growing down after one grows up?

I have heard a number of times that little children have the idea that one grows down after one grows up. Why do some get that idea? I don't think all do, my younger cousins never did, from what I noticed. It usually is with children around 2-3, I think; maybe 4, too.

I'm thinking it has to do with the idea that their minds pick up on patterns like the bedtime routine, days of the week, etc., but it could just be the attempt to make sense of the world. (i.e.: "Mommy has to be a baby sometime, I never saw her as a baby, therefore she must be a baby sometime in the future.") It is definitely from the preoperational stage, as I' think 4 is the oldest I ever heard of tha child thinking this.Somebody or his brother (talk) 14:06, 20 December 2008 (UTC)[reply]

"What grows up must grow down" ? StuRat (talk) 17:23, 20 December 2008 (UTC)[reply]
"... Last scene of all,
That ends this strange eventful history,
Is second childishness and mere oblivion;
Sans teeth, sans eyes, sans taste, sans everything."
Well, humans do shrink slightly if they make it beyond middle age - the skeleton starts to degenerate, the posture becomes more stooped etc. Exxolon (talk) 02:29, 21 December 2008 (UTC)[reply]
Additionally, due mostly to better nutrition, each of the past few generations has been slightly taller, on average, than its predecessor. Your parents are probably taller than your grandparents. A young child may assume that all adults top out at the same height, so the older generation must be now be shrinking. B00P (talk) 08:10, 21 December 2008 (UTC)[reply]

Why does chronic stress sometimes lead to stroke

I can understand why stress can lead to a heart attack - that's pretty logical, with the strain on the heart. I'm a little confused, though, on why stroke can be a result. Is stroke only a result because of the impact on the heart itself, which thus leds to blockages? Or, does persistent, extreme stress actually do something to the brain itself, which causes clots to form there, just as it can lead to a headache, etc.?

Or, perhaps it can be either, depending ont he type of stroke, which is quite plausible.209.244.30.221 (talk) 16:04, 20 December 2008 (UTC)[reply]

Strokes that are caused by chronic stress are due to a disturbance of a blood vessel in the brain. Chronic stress increases your blood pressure, which increases the risk of rupture of a blood vessel. If this blood vessel is in your brain, and it ruptures, it can cause a stroke. —Cyclonenim (talk · contribs · email) 16:21, 20 December 2008 (UTC)[reply]
Stress may also drive a general inflammatory state, leading to hypercoagulability (this study is an example: [14]), endothelial dysfunction (e.g. in the carotid artery or other location), and then embolization resulting in ischemic stroke that can sometimes become a hemorrhagic stroke. Stress has many negative effects, and exercise is a positive balancer for almost all of them (whereas watching TV is not). --Scray (talk) 19:12, 20 December 2008 (UTC)[reply]
Rare, but in people with Sticky platelet syndrome [15] stress can also set off a stroke. http://www.cli-online.com/products/haemostasis-coagulation/haemostasiscoagulation/clinical-laboratory/testing-the-sticky-platelet-syndrome/index.html?tx_ttproducts_pi1[backPID]=1031&cHash=24a96e1d04 (Don't know the latin name, so we may have a page)76.97.245.5 (talk) 12:07, 21 December 2008 (UTC) Sorry the long link won't work otherwise because it contains angled brackets. If s.o. knows how to fix it, plse. do.[reply]

is it possible to produce a novel algorithm USING THE POWER OF THE MIND ALONE? (no other tools of any kind)

Is it possible to produce a novel algorithm USING THE POWER OF THE MIND ALONE? (no other tools of any kind) —Preceding unsigned comment added by 79.122.90.7 (talk) 18:31, 20 December 2008 (UTC)[reply]

Since I know of no physical effects associated with a mind as opposed to workings of the brain I would say no. If by mind you mean just thinking about things, power of the mind as th ability to think and find patterns, and algorithm as a way of doing something then I would say yes, where else would the novel bit come from, even noticing something novel is something that has to be thought. Dmcq (talk) 18:52, 20 December 2008 (UTC)[reply]
It's hard to imagine any algorithms that were not produced using the power of the mind alone! So, for example, Quicksort is a well-known sorting algorithm developed by C. A. R. Hoare - I imagine he used his mind alone to develop it. My son and I are writing a computer game (to give to my wife for Xmas!) - it has a couple of new algorithms in it (well, they are new to us) - and I just sat down and thought them up. I didn't need a pencil and paper or a computer to do that.
This seems (to me at least) like such a ridiculously simple answer - that I have to imagine that the OP is actually asking something more complicated. Perhaps a clarification is needed? SteveBaker (talk) 20:27, 20 December 2008 (UTC)[reply]
One way to look at it... You can imagine an algorithm and suspect that it will work as well as you think it will. Until you physically test the algorithm, you will not know if you made a mistake in your thinking. So, the mind will create the algorithm, but you need to test it afterwards. I know, there are thousands of mathematics students who will swear that you can prove an algorithm without testing - but that assumes you didn't make a mistake in your proof. You still need to test it to make sure there were no mistakes in the creation of the algorithm. Want a silly little anecdote? Many years ago (when I got my brand new TI-99 4/A), I thought of a cool algorithm for using an evolutionary approach to solving simple problems. I thought it through, then wrote it down, and finally programmed it in. My test problem was very simple. The bot (I called it) got two single digit numbers, 1 to 9. It was supposed to produce the sum of the numbers. The further away it was from the sum, the worse it was punished. After a few hundred generations, the bot kept answering 10. Why 10? My algorithm was bad. The bot didn't learn to add two numbers. It learned that if it answers 10, it will limit the punishment it receives over the long term. -- kainaw 00:58, 21 December 2008 (UTC)[reply]
Well - that's not a really fair example - you didn't come up with an algorithm to solve your problem - you essentially said - I don't want to be bothered with figuring out the algorithm, so I'll come up with an algorithm that'll allow the computer to learn - and I'll hope that IT can figure it out. Your algorithm (the learning thing) worked just fine - it learned...but because it's fairly stupid, it learned the wrong thing.
Obviously when we create algorithms using our minds alone, we sometimes make mistakes. Testing to see if a mistake has been made can certainly be done by implementing the algorithm in a computer and testing it - but our OP asks whether we can create algorithms without a computer - and we CERTAINLY can. There are plenty of simple algorithms we can test in our heads. It's the essence of a good computer programmer to be pretty certain that somethings' going to work BEFORE sitting down to write the code. You can't always be 100% certain - but sometimes you are - and the skill of computer programming is in spotting problems as early as possible. At any rate - we CAN and DO produce algorithms 100% in our heads. SteveBaker (talk) 01:30, 21 December 2008 (UTC)[reply]
I'm sorry, but you appear to have misread my comment. I came up with an algorithm to have a computer figure out an algorithm. My algorithm was how it would learn. I didn't have the computer figure that out. However, I explained that my algorithm (which seemed fine to me when I thought it through) didn't work. -- kainaw 04:16, 21 December 2008 (UTC)[reply]

I think the following question is more interesting than the original, "Is there any algorithm that has not originated in a human mind ?"
I realize that the phrasing is fuzzy and the answer will depend upon how we define the terms "algorithm", "originated", "mind" etc - so in order to avoid a semantic or philosophical discussion, let me specify that I am interested in a computer generated algorithm that is analogous to (say) the computer generated proof of the four color theorem. Any examples ? Abecedare (talk) 03:17, 21 December 2008 (UTC)[reply]

You have to define "algorithm". It is normally considered something like "a sequence of steps to solve a problem." There are many algorithms that were not created in the human mind. For example, a hedgehog knows that if it feels threatened, the sequence of steps is: 1) Try to run away. 2) If that fails, roll into a ball. It isn't a great algorithm, but it is a series of steps to solve the problem. DNA replication is another (more complicated) algorithm that was not created in the human mind. But, if you want to claim that all biological, social, and natural processes are omitted from the things you want to call "algorithms", then you have decided to limit algorithms to humans and humans alone (assuming humans are not biological, social, or natural). -- kainaw 04:21, 21 December 2008 (UTC)[reply]
I think the example Kainaw gave of a learning machine is the kind of thing where computers do (in a sense) come up with an algorithm. There is an old story (and it may be apocryphal) of a Japanese subway operator who had problems with the platforms of his stations filling up with so many people that it became dangerous. Doors were added at the entrance to the platform that could be closed when the platform became too crowded. They decided to use a camera and a neural network software package to look at the images of the platform and close the doors when it started to become too crowded. They showed the software lots of pictures of crowded platforms and uncrowded ones - and trained it (as one does with neural nets) to recognise one set of images from the other. When they put the resulting (trained) network into service, it worked just great - reliably shutting the gates when the platform started to fill up - and opening them again as it emptied. The system was in service for almost a year. However, on Xmas day, it failed. Shutting the gates when the platform was completely empty. The story goes that the engineers work hard to find out why the neural network failed and they discover that only the neurons that are looking at the very top of the image from the camera are actually doing anything. The others have shut themselves off. It turns out that the camera was looking at the clock on the wall of the station and shutting the doors at the exact times of day that the station tended to get overly full.
I don't know whether that story is true or not - but (as Kainaw and numerous others have discovered) computer learning software doesn't always learn the things you want or expect it to. The story persists precisely because it is an excellent description of exactly the kinds of things that happen with learning software.
If the Japanese subway story is true - then the neural network "discovered" an algorithm for shutting the gates that worked pretty well. It had not been taught about clocks or anything like that - it had (in essence) 'deduced' an entirely different way of achieving the stated goals. Just as C. A. R. Hoare discovered a better way to sort things when he figured out 'QuickSort'.
SteveBaker (talk) 06:52, 21 December 2008 (UTC)[reply]
Are Japanese subways busy on Christmas day? There was a similar story about neural networks being trained to identify tanks in recon photos. Long story short the neural network was just checking for cloudy days. Again, I have no idea of the origins of this story, but I have a nagging suspicion I heard it on "The Machine That Changed the World" or a similar documentary. APL (talk) 17:26, 21 December 2008 (UTC)[reply]
I don't know about how busy Japanese subways ACTUALLY are on Xmas day - but the implication from the story is that the station was deserted because of the holiday - and the computer went ahead and shut the gates, not because the platforms were full but because the clock was showing the time of day at which the platform is USUALLY full. I'd heard the ones about tank recognition too. Somewhere in the boxes of books in my garage I have a book on Neural Networks that tells both stories (and several others also). The field of computer learning (be it neural networks or evolutionary algorithms or whatever) is simply littered with these kinds of stories. Some are certainly true.
One that I like (and I know is true) is the guy who was trying to evolve 'virtual creatures' using computer graphics and such. He was interested in having them evolve to be able to move around in their virtual world. He had simulated muscles and bones and such - with neural network intelligence to drive them. He set the simulation up and ran races between creatures with different 'genes' to see which would be the first to cross a line some distance away. The evolutionary part favored the genes that generated creatures who crossed the line the fastest - or in the even that none of them got there within a reasonable amount of time - rewarded the creature that got closest. The simulation ran overnight and the researcher was bitterly disappointed to discover that all of the creatures that survived the process were simply tall vertical columns - no intelligence, no muscles, nothing. The trick was that the creatures simply fell over. They'd grown taller so that when they landed, they'd be closer to the finish line - and the tallest creature 'won' the race every time. In essence, the creatures had found an 'algorithm' (falling over) that solved the given problem (crossing the finish line as quickly as possible) in a manner that was vastly simpler and more efficient than the human designer imagined. SteveBaker (talk) 21:29, 21 December 2008 (UTC)[reply]

What does the "LIO" prefix mean in the scientific name of "Pocket Mice"?

Many scientific names of species have prefixes that define a group of animals within a larger group. Thus, among the various mice ("MYS" in classical Greek) the earth-based mice called in English "pocket gophers" are named in the scientific taxonomy Geomys (literally "earth-mice"). In Greek the prefix "Lio" denotes smooth, but in Latin the same prefix means lion-like or leontine. Since Greek and Latin are used indiscriminately in nomenclature, it is impossible to guess what an animal's name with that prefix means. My question is therefore twofold: 1) In the particular case of "LIOMYS", the Mexican Pocket Mouse, does the "Lio" prefix mean smooth or leontine? 2) In general terms, is there a way to tell which is correct, or does one need specific knoledge for each and every case this prefix is used? Thank you! --Bergeronz (talk) 20:11, 20 December 2008 (UTC)[reply]

You also asked this on the Language reference desk and it has been answered there. Please do not multi-post. If you really aren't sure which is more appropriate, you could post a query on one desk and a link to it on another. --Anonymous, 03:20 UTC, December 21, 2008.

December 21

Hey, have they ever expanded on that life creating experiment thing?

I mean well beyond the Miller Urey experiment. —Preceding unsigned comment added by THE WORLD'S MOST CURIOUS MAN (talkcontribs) 00:40, 21 December 2008 (UTC)[reply]

There's some additional studies mentioned at the end of the Miller-Urey article. Also, check out the references in Abiogenesis article. -- JSBillings 01:03, 21 December 2008 (UTC)[reply]
Yes - there have been many recreations of the original experiment - and others with conditions closer to those that we now believe to have been present in the early earth. However, these aren't really serious attempts to create life - only to show that the basic building blocks (Amino acids, carbohydrates, etc) could have formed spontaneously in the early planet. Life itself is likely to have been an extreme statistical flook. When the oceans were full of amino acids and other nutrients, it would only take ONE molecule of a self-replicating DNA/RNA type of substance to appear in order for life to get started and for evolution to rapidly do it's thing. Since the process of creating that one molecule could easily have happened just once - in a half billion years and in trillions of cubic meters of water - the probability of being able to reproduce that in a laboratory using the original conditions during the lifetime of the researcher is essentially zero. So the experiment would certainly appear to fail - in that it would not produce life - but that would only prove how statistically infrequent such events must be. SteveBaker (talk) 01:19, 21 December 2008 (UTC)[reply]
For quite different kinds of life creating experiments, see Mycoplasma laboratorium. Icek (talk) 08:17, 21 December 2008 (UTC)[reply]
Yes - indeed. It seems very likely that we'll be able to produce a completely synthetic life form in perhaps the next decade - but that's a very different thing from reproducing the means by which the first living thing came about on Earth. SteveBaker (talk) 21:03, 21 December 2008 (UTC)[reply]
I thought they had given up on the "live from the big ocean" concept and gone to hydrothermal vents and hot springs because the "chemical soup" there would be more reactive? 76.97.245.5 (talk) 12:51, 21 December 2008 (UTC)[reply]
Probably - but in terms of answering this question it's still the same deal. We look at ALL of life on earth and every single living thing is clearly descended from a common ancestor because so much of the DNA is shared. This suggests that the 'abiogenesis' event (the first self-replicating molecule coming out of non-living material) may well have been a one-off event - something so amazingly statistically unlikely that it took billions of years and vast number of "attempts" for it to happen. Even if it had to happen at a hydrothermal vent - there are huge numbers of those - and we still have a billion years or so for just one such event to happen....so it's still spectacularly unlikely that we'd be able to reproduce it experimentally. SteveBaker (talk) 21:03, 21 December 2008 (UTC)[reply]

Lipid Layer

Which part of the eye produces the lipid layer?96.53.149.117 (talk) 01:51, 21 December 2008 (UTC)[reply]

Aren't you the expert on this by now? [16] [17] ? Perhaps you should consult a physiology textbook if you're having a hard time finding the information you need for these articles on the internet at large - I recommend Tortora and Grabowski as an introductory anatomy and physiology textbook. Nimur (talk) 02:04, 21 December 2008 (UTC)[reply]
For readers not aware of prior discussion, the answer is quite prominently displayed in a table near the beginning of our Tears article. If the OP is asking something less blindingly obvious and repetitive of his/her recent questions, please clarify. --Scray (talk) 03:09, 21 December 2008 (UTC)[reply]
Oh fuck, you are right! I knew I shouldn't have asked this question. Lol, I knew I'd find it there, my bad!96.53.149.117 (talk) 04:46, 21 December 2008 (UTC)[reply]
You did know that you would find this fact in that article, because you put it there, as I pointed out earlier. Nimur (talk) 23:14, 21 December 2008 (UTC)[reply]
If you had entered your exact question "Which part of the eye produces the lipid layer?" google would have answered your question fully with its first reply. Not a wiki answer but if you put in this query instead "Which part of the eye produces the lipid layer wiki" the first reply is to the wikipedia entry on tears. Please learn to do these very basic types of search first before wasting peoples time, see the bit at the top of this page about trying to answer your question yourself first. Dmcq (talk) 11:35, 21 December 2008 (UTC)[reply]

Identifying an unknown mammal

Can someone please help identify this mammal, found in the forests of Peru?

[18], [19] and [20].

I think it is a shrew opossum (Caenolestes caniventer or Lestoros inca), but then I might not be right. Let's see if we can find out! --Leptictidium (mt) 10:08, 21 December 2008 (UTC)[reply]

Guessing more like a Thylamys "mouse opossum"? (shorter snout, bigger eyes, monochrome tail) Julia Rossi (talk) 10:54, 21 December 2008 (UTC)[reply]
This [21] site has pictures plus a map of S-American mouse opossumice. --Cookatoo.ergo.ZooM (talk) 11:37, 21 December 2008 (UTC)[reply]

Skua vs. Great Black-backed Gull - who would win?

No, this isn't going to be another one of *those* threads - rather it's a question based on something I just read in WPs Skua article, namely this:


This is absolute bollocks, right? I could envision a Skua taking GBB chicks or eggs, or battling with an adult over food - but really, a fully-grown GBB is *bigger* than the largest Skua and every bit as mean-tempered and predatory. As far as I am aware, nothing short of one of the great eagles would even attempt to make a meal out of a healthy adult bird. --Kurt Shaped Box (talk) 10:58, 21 December 2008 (UTC)[reply]

Do you know whether either or both of those species might hunt in groups? OR We have a flock (?) of about 6 crows flying around that harass everything in the neighborhood, including a bird of prey. (Sorry haven't had time yet to look him up.) I've also seen a crowd of swallows take on a magpie. Size isn't everything. 76.97.245.5 (talk) 11:48, 21 December 2008 (UTC)[reply]
Not as far as I know. Gulls and Skuas will defend the nesting areas from potential threats en masse but when it comes to hunting and scavenging, it's strictly a competitive activity, with much (often counter-productive) squabbling and bickering. Sometimes I get the impression that preventing a rival from feeding is almost as important to them as getting a meal for themselves (two birds fight, third bird comes in and steals food). --Kurt Shaped Box (talk) 13:11, 21 December 2008 (UTC)[reply]

Whats so special about Half-Integer spin? 59.93.92.192 (talk) 13:23, 21 December 2008 (UTC)

Why is it that only particles with half-integer spin, ie fermions, obey the pauli exclusion principle which disallows more than one of the same particle to exist in the same quantum state? —Preceding unsigned comment added by 59.93.92.192 (talk) 13:21, 21 December 2008 (UTC)[reply]

The Pauli exclusion principle applies to states with anti-symmetric wavefunctions (Fermions). The Spin-statistics theorem shows that Fermions have half-integer spins. 86.134.187.65 (talk) 16:06, 21 December 2008 (UTC)[reply]

Mystery flower

If there's any botanists here, can anybody identify this flower? Unfortunately, I neglected to look at the stem or take a photograph further out, so this is about all I have to identify it with (other than it's in North Carolina, USA). Tastyduck (talk) 15:18, 21 December 2008 (UTC)[reply]

Did you find it in a garden or out in the wild somewhere? Since it's dried up for the winter things like color of the flowers etc. can no longer be used to identify it. That makes it a bit tricky. 76.97.245.5 (talk) 23:59, 21 December 2008 (UTC)[reply]
This site [22] might help. 76.97.245.5 (talk) 03:06, 22 December 2008 (UTC)[reply]

Overcooked vs. burnt food

Chemically speaking, what's the difference between food that is overcooked and food that is burnt? Can food be burnt and *not* be overcooked? --70.167.58.6 (talk) 16:40, 21 December 2008 (UTC)[reply]

Well, sort of. You could char a steak on the outside, yet the steak inside is perfectly done. I'd say that "overdone" typically means that something has been cooked so that the chemical properties break down. Cakes become dry, eggs become rubbery, steak becomes a brick. On the other hand, 'burnt' means that you have actually started combusting thing. --Mdwyer (talk) 17:21, 21 December 2008 (UTC)[reply]
(ec) 'Burnt' usually implies that there are products of combustion (or pyrolysis, if in a low-oxygen environment) present. The food has been heated past the point of dehydration, allowing elevated temperatures and charring. Note that 'burnt' can be somewhat subjective. The process of caramelization tends to happen just before you get to burning, lending additional flavours. The savoury 'crunchy bits' of grilled and fried foods may be technically 'burnt', but still important to the overall taste of the dish. (Think grilled chicken, calamari, or steak — they wouldn't taste the same without streaks seared into them from the grill surface.)
Overcooked food, meanwhile, need not be burnt. Think of soggy vegetables that have been steamed or simmered into mush, losing all flavour, texture, and nutrients.
'Burnt but not overcooked' is a bit of a semantic issue. While a particular part of a dish might have trouble meeting both criteria, the entire dish overall can have no difficultly being both 'burnt' and 'undercooked' simultaneously. Think of a cake baked at too high a temperature for too short a time — the outside will be blackened, but the inside can still be raw. (Of course, you can do this deliberately; see Baked Alaska.) TenOfAllTrades(talk) 17:24, 21 December 2008 (UTC)[reply]
Just for the record, the outside of a Baked Alaska is supposed to be a "delicate brown", not blackened. At least in this neck of the woods. CBHA (talk) 17:39, 21 December 2008 (UTC)[reply]
Yeah, I should have been a bit more specific. I was referring to the general principle of the insulating properties of cake, rather than the specific details of the recipe. :D TenOfAllTrades(talk) 17:52, 21 December 2008 (UTC)[reply]
I figured that was probably the case. I just did not want anyone burning their Alaska based on what they read in Wikipedia. After all, one can see Russia from there. Or so I hear. Wanderer57 (talk) 18:42, 21 December 2008 (UTC)[reply]
One might want to read about the Maillard reactions, a class of chemical reactions which include both "browning" and "carmelization" and a number of other chemical reactions that make cooked food "tasty". --Jayron32.talk.contribs 02:03, 22 December 2008 (UTC)[reply]
Crème brûlée? --Maltelauridsbrigge (talk) 11:02, 22 December 2008 (UTC)[reply]

Wood-burning stoves, creosote and soup cans

The local farmer who blows out our driveway and shovels the deck around the house likes to give advice to us former "city slickers". Sometimes, I think he is pulling our collective legs; however, as today's tip was about fire safety, I thougt I'd check it out here. (I have googled various combinations of the title of the section to no avail.) Creosote build-up in a chimney is the biggest danger for house fires. We do all the sensible things: burn only well-seasoned hardwood, clean the chimney at least twice a season, and burn the wood at a sufficiently high temperature to discourage the creosote from sticking to the chimney liner. The farmer says that another trick is to throw in a couple of empty soup cans and burn them daily. He claims there is a chemical given off in the burning that also inhibits the formation of creosote on the liner. He didn't know what chemical it was, or if it was in canned goods other than soup, or if it could be purchased in some other form. I doubt we open two tins in a month, but are willing to make some sacrifices (or borrow tins from the neighbours' recycling bins) if it will help prevent a house fire. Any thoughts, pro or con? Thanks for any help you can provide. ៛ Bielle (talk) 18:03, 21 December 2008 (UTC)[reply]

I doubt he's pulling your leg intentionally, but he might be a victim of some old wives' tale. There do exist chemicals that can be burned to clean out your chimney. In theory, they catalyze the creasote into a form that falls down the chimney instead of sticking. However, there seems to be quite a lot of doubt as to if they actually work or not. [23] In any case, a chimney fire can be devastating, so have your chimney inspected at least every couple of years. --Mdwyer (talk) 18:16, 21 December 2008 (UTC)[reply]
If you only look down your chimney every couple of years, on the off years you won't be able to enjoy your fire because you'll be sitting there wondering how much creosote is in there. Make a chimney inspection part of your winterization routine, and run a brush down it anyway. There's another old husband's tale about flashlight batteries, but I never tried it, for two reasons: I didn't want whatever was released by burning a battery to be near me (zinc, at least, I guess), and I didn't want stuff in my ashes because I used them for traction on ice in my truck. --Milkbreath (talk) 12:47, 22 December 2008 (UTC)[reply]
Have a look at our Bisphenol A page. If your cans are coated inside with synthetic resin, which I assume is precisely what is supposed to create the presumed effect, you would end up with this and similar/worse chemicals in the air around your fire. Ideally it would all go up the chimney and "harmlessly" pollute the air outside, but do you really wish to bet your health on that. If the stuff is caustic enough to scrub creosote off the large surface area of you chimney, I don't want to know what it will do to mucous membranes and lung tissue of your respiratory tract.76.97.245.5 (talk) 07:37, 22 December 2008 (UTC)[reply]
Yeah, at best it sounds like an old-wives-tale, at worst actively harmful. I'd say what you're doing now (regular inspection/cleaning of the chimney, not burning dodgy wood) is all you need to be doing. ~ mazca t|c 09:22, 22 December 2008 (UTC)[reply]

Esclorophile Bush?

The "word" esclorophile appears in the article Tlalpan. Not knowing what it is, I searched for it using Google and found it in only three places on the internet. I'm wondering based on this if it is a real word or an error or vandalism.

Can anyone help with this? Thanks. Wanderer57 (talk) 18:28, 21 December 2008 (UTC)[reply]

The three Google hits were all from the Wikipedia page. If you look at the next paragraph in the Wikipedia article, the word appears spelled "esclerophile". This word is not in any online dictionary I could find, including the Oxford English Dictionary, but the "-scler-" part gives us a toehold. In words like "sclera" and "sclerosis" it means "hard" after the Greek. It's my guess that it's really "esclerophile" and means "hard-loving" in reference to the rocky terrain. "Esclerophile" yields a few non-Wikipedia hits on Google. --Milkbreath (talk) 18:54, 21 December 2008 (UTC)[reply]
I've just cleaned up that article, and I chose to retranslate that word as "sclerophyllous". How did I know? Because the Spanish WP article es:Esclerófilo is cross-linked to the English WP's Sclerophyll. --Heron (talk) 19:54, 21 December 2008 (UTC)[reply]
I have redirected Esclerophile to Sclerophyll in case it pops up in the future. Nimur (talk) 23:34, 21 December 2008 (UTC)[reply]

Paranoid park (spoiler warning)

In Paranoid park a guard gets cut into two, if I remember correctly somewhere below the waist, and after this for some seconds his upper part crawls away (supported by the arms) from the lower part. Is this really possible??? Lova Falk (talk) 19:58, 21 December 2008 (UTC)[reply]

I would expect the blood loss, organ damage, and likely spinal damage to make such a thing very unlikely... --98.217.8.46 (talk) 20:48, 21 December 2008 (UTC)[reply]
It would depend upon where the guard got cut into two pieces. If it was the lower spinal region, it would avoid any paralysis in the upper body so the arms could still be used. This would also avoid damage to organs except, perhaps, the intestines. The blood loss is an issue but if the person was particularly strong willed to pain it might be possible for a few seconds. —Cyclonenim (talk · contribs · email) 21:20, 21 December 2008 (UTC)[reply]
Isn’t the guy cut in half by a freight train? It’s not a super clean cut like from a nihontō or something. A train is likely to cause much more physical trauma. I think that degree of movement would be unlikely. --S.dedalus (talk) 21:59, 21 December 2008 (UTC)[reply]
I wouldn't know, I've never seen the film. You're right, if it's a train then I doubt very highly it could occur. But if the person were cut in half in a clean cut, it might. —Cyclonenim (talk · contribs · email) 23:21, 21 December 2008 (UTC)[reply]
This seems to me just an exaggeration of Mike the Headless Chicken. — Sebastian 05:45, 22 December 2008 (UTC)[reply]
Ah, what a sissy! Klaus Störtebeker walked along twelve of his companions after he was beheaded! 95.112.137.182 (talk) 18:14, 22 December 2008 (UTC)[reply]
As also St. Denis, who walked about two miles after being beheaded, carrying his head and delivering a sermon. - Nunh-huh 20:52, 22 December 2008 (UTC)[reply]
Though some say St.Denis was a ventriloquist [24], and only moved his lips pretending he was speaking --PMajer (talk) 10:24, 23 December 2008 (UTC) 21:08, 22 December 2008 (UTC)[reply]
If the injury is sharp (a cut rather than a crush) then what you describe wouldn't happen. Such an injury would sever the inferior vena cava and the descending aorta. Blood would squirt from the openings of both vessels, the arterial blood with some force. This would leave the blood pressure in both vessels very low, and in turn the blood from the superior vena cava and the carotid arteries would drain under pressure into their lower counterparts. The cerebral blood pressure would immediately collapse, causing the prompt cessation of cognitive and motor functions. The uncinematic reality of most mortal injuries is that people just crumple up like a puppet with its strings cut, rather than staggering around delivering lapidary epitaphs before lying down. 87.114.130.249 (talk) 21:36, 22 December 2008 (UTC)[reply]
Going back to the OP we should recall that the scene of the guard's death was an oneiric sequence or in any case, a scene in the imagination of the boy, who got shocked hearing about an accident in the news; so the episode is expressly unreal (is it so, or is it mine a dream of a dream?) --PMajer (talk) 10:47, 23 December 2008 (UTC)[reply]
Possibly - but it's irrelevent - we know that the scene is in a work of fiction, that it's a fiction inside a fiction doesn't help! The question is about reality...and it's pretty clear that the fiction (or meta-fiction...or fiction2) is incorrect. Certainly, blood pressure would drop to zero pretty much instantly (the veins and arteries are not going to pose much of a resistance to the blood flowing out) - which means that the motive power to the brain gets switched off like a light switch and loss of consciousness is almost instantaneous. Cases of bodies twitching and eyes rolling and other 'spooky' things happening shortly after death are due to now uncontrolled muscles using up the last of their energy...but the idea that there is still intent and consciousness driving them is not reasonable. Mike the chicken is weird...but he wasn't COMPLETELY headless - he still had one ear and enough brain-stem for the kinds of things chickens need to do to behave. That he survived the trauma and blood loss is the amazing thing. SteveBaker (talk) 14:36, 23 December 2008 (UTC)[reply]
Well, I assumed the question was mainly about the movie rather than about the human anatomy. The scene appears indeed unreal even without a PhD in medicine, so one can ask, what is the meaning of it. Anyway, I feel better since you confirm that it is scientifically impossible :) --PMajer (talk) 20:16, 23 December 2008 (UTC)[reply]
No, the question was about human anatomy. I did not interpret the scene as something happening in the imagination of the boy. Lova Falk (talk) 20:30, 23 December 2008 (UTC)[reply]
It is true that blood pressure would collapse immediately, but that is not to say that someone cannot remain conscious even momentarily for a few seconds! We know that cells can survive by anaerobic respiration, which is why the brain can survive for a few minutes without O2 so with significant pain tollerance it could be possible to remain conscious for a few more seconds. This is why you can survive a heart attack temporarily before CPR is initiated, since cells can survive adequetely for a while on no oxygen. —Cyclonenim (talk · contribs · email) 19:41, 23 December 2008 (UTC)[reply]
No blood FLOW is not the same thing as no blood PRESSURE. When your heart stops, the blood still in the vessels inside your brain still contains oxygen that can presumably be extracted to keep you alive for quite a while...after all, blood flows around in a big loop giving up a little oxygen to each part of the body as it goes around. So if it merely stops flowing for a while, you'd presumably still be extracting some oxygen from it for quite a while. But when the pressure drops to zero and it flows out - that's simply not possible - so I'd expect you to black out so much sooner. SteveBaker (talk) 21:23, 23 December 2008 (UTC)[reply]

Metal detectors for non-magnetic metals

If I remember right, metal detectors (I'm talking about the security ones, not the ones people use to go and find things underground) can detect non-magnetic metals. For example, if you've had a titanium rod or plate put into your body in an operation, you have to tell the metal detector operator because you'll set it off simply with the metal in your body. The metal detector article seems to say that metal detectors depend on electromagnetic processes (forgive my lack of electromagnetic knowledge) to find metal, but I couldn't find anything in the article regarding non-magnetic metals. How is it that these detectors pick out titanium, aluminium, etc.? Nyttend (talk) 22:15, 21 December 2008 (UTC)[reply]

Titanium and aluminium / aluminum are paramagnetic. An external magnetic field will, therefore, still detect the metal as it becomes weakly magnetic, albeit temporarily. --Cookatoo.ergo.ZooM (talk) 22:49, 21 December 2008 (UTC)[reply]
How is it, then, that people with titanium in their bodies can, in certain circumstances, undergo MRIs safely? Is it just really weakly magnetic, too little to be pulled out of place by the MRI magnets? I'm not challenging you, just confused. Nyttend (talk) 22:58, 21 December 2008 (UTC)[reply]
Titanium is not magnetic enough to be pulled around in an MRI. Also, it produces very little "artifact" (a term used by MRI operators for the weird glows caused by metals in general). Therefore, titanium tools for use around MRI machines is becoming popular, as well as titanium screws, staples, and such. Now, you should note that there is not claim that titanium is not pulled around at all or that it produces no artifact at all. It just isn't enough to be of any concern. -- kainaw 00:03, 22 December 2008 (UTC)[reply]
Thanks for the helpful explanations — I think I understand this a lot better than when I posted the question in the first place. Nyttend (talk) 00:37, 22 December 2008 (UTC)[reply]
As the article says, many (most) metal detectors create an oscillating magnetic field. Therefore the magnetic field strength is changing all the time and a changing magnetic field causes induction of electric currents in a conductor - in this case eddy currents, which in turn create their own magnetic field which is opposed to the original magnetic field (see Lenz's law). Thus, the total magnetic field strength (which is measured by the device) will be smaller in the presence of a conductor. While human tissue is a conductor too, most metals are better conductors, and cell membranes surpress eddy currents (as a side note, when constructing a transformer, you want to minimize eddy currents, and this is usually done by using thin plates electrically insulated from each other as magnetic conductors; vibration of these plates causes the humming sound of transformers). Icek (talk) 09:13, 22 December 2008 (UTC)[reply]

December 22

Generator

Why is copper used in generators instead of other metals? —Preceding unsigned comment added by 65.5.174.233 (talk) 01:29, 22 December 2008 (UTC)[reply]

Because copper has a high electrical conductivity. Among (relatviely) common metals, copper is one of the most conductive around. Silver is somewhat more conductive, but also prohibitively more expensive. Copper is also one of the most ductile metals, making it well suited for making wire. --Jayron32.talk.contribs 01:58, 22 December 2008 (UTC)[reply]
During the second world war the Manhatten Project needed to make large electrical machines for extracting uranium isotopes to make the atom bomb. Copper was in extremely short supply because the sources were cut off and the demand for military applications was high. So the US Mint had to turn over all of its silver bullion stocks to make the cabling the project needed. So I guess the second most suitable "commonly found" metal is indeed silver. SteveBaker (talk) 14:09, 22 December 2008 (UTC)[reply]
Alumin(i)um is a third choice, and it probably has a larger installed base than pure silver wiring. While not used extensively in generator windings (as far as I know) aluminum wire is widely employed in utility transmission lines; it was also popular in residential construction during a spike in copper prices in the 1960s and 70s. TenOfAllTrades(talk) 14:41, 22 December 2008 (UTC)[reply]
I expect that a generator could be made with the stator and rotor conductors made of various metals other than copper and it could work ok. Probably even mercury could be used, if confined in strong enough tubing. Cost/weight/size/strength are the design factors which suggest copper. Consider superconductive windings, which would not be copper. Edison (talk) 20:18, 22 December 2008 (UTC)[reply]

Planet destroying energy

How big would a nuclear blast need to be to destroy Earth? To actually obliterate it into chunks? I stumbled upon this page [25] which shows the size of the Death Star in relation to the moon. I can't imagine something so small blowing up an Earth-like planet that's 60x bigger than the moon. Granted, it's science fiction. But what quantity of energy would the Death Star need to generate to blow up a planet? Is that even in the realm of fusion or matter/anti-matter reactions? --70.167.58.6 (talk) 05:37, 22 December 2008 (UTC)[reply]

You can't quite compare an explosion with an Impact force. The two behave differently as you can see from those pages. 76.97.245.5 (talk) 07:10, 22 December 2008 (UTC)[reply]
You need to overcome gravitational binding energy. The value for Earth is a bit larger than the number given in the article because Earth is not of uniform density; it's about 2.4*1032 J. That's the energy equivalent of 2.7*1015 kg. If the chunks are as small as they appear in Star Wars, the gravitational binding energy of these chunks is negligible compared to the 2.4*1032 J, so you'll need approximately that amount of energy. Icek (talk) 08:50, 22 December 2008 (UTC)[reply]
For comparison, the theoretical maximum yield of the Tsar Bomba, the largest fusion weapon ever tested, was 100 Mt or 4.2x1017 J (the actual test bomb was limited to a yield of 50 Mt). So, even if all of this energy could be utilised, you would need 5x1014 (i.e. 500 trillion) such bombs to reduce the Earth to small chunks. Gandalf61 (talk) 09:17, 22 December 2008 (UTC)[reply]
You'd also have to give those chunks enough speed to exceed their mutual escape velocity - or else they'd all come back together in a gigantic collision and form a new planet ("NEW!! Earth 2.0TM - now with fewer annoyingly destructive lifeforms!"). SteveBaker (talk) 14:04, 22 December 2008 (UTC)[reply]
Hmmm.... what do you mean? You can do this if you have 2.4*1032 J at your disposal - at least in principle. Icek (talk) 14:19, 22 December 2008 (UTC)[reply]
A pretty good website about it can be found at [26], and some technical data here: [27] --86.125.162.148 (talk) 18:47, 22 December 2008 (UTC)[reply]

Gravity hole

Could there theoretically be an equivalent for black holes when we are talking about gravity and gravitons ? 69.157.229.14 (talk) 05:40, 22 December 2008 (UTC)[reply]

What type of "equivalent" for a black hole are you trying to define? AFA our article states a black hole is a phenomenon related to gravity. Are you trying to have a location not influenced by gravity? (I think that would be impossible, the best you can hope for is offsetting forces e.g. with a magnetic field.[28]) Gravitons are (hypothetical) particles that "carry" gravitational forces, sort of like electrons carry electrical charge. or Are you thinking of supersymmetry, creating an "anti-black hole" with Gravitinos?? P.S. I put hypothetical in brackets so that we don't end up with another one of those "it's only a theory discussions". No one's found one yet, but there are a lot of clues saying there should be one, unless s.o. is going to come up with a better explanation. 76.97.245.5 (talk) 06:56, 22 December 2008 (UTC)[reply]
I guess you're asking about a region of space from which gravitons can't escape. That would be a black hole. Nothing can escape across the event horizon of a black hole, including gravity. The field of a black hole is a so-called "fossil field" left over from the collapse. It doesn't come from inside the hole. -- BenRG (talk) 08:08, 22 December 2008 (UTC)[reply]
I don't think that's right. A black hole's gravitational attraction to other objects does increase with every object it swallows, doesn't it ? Therefore, those theoretical gravitons must escape. Perhaps this is a flaw in graviton theory itself, and visualizing gravity as a deformation in space-time is more accurate. StuRat (talk) 18:43, 22 December 2008 (UTC)[reply]
Yes - it's much easier to think of gravity as space-time deformation. But (to pre-empt Jayron32 who is just ITCHING to say this) "Gravity is gravity, however, and that it sometimes behaves like a space-time deformation and sometimes like a particle is not gravities problem. Its ours."
But it seems entirely unreasonable to me that a graviton is affected by gravity. That doesn't sound right. The (hypothetical) graviton is the force-carrier particle for a gravitational field - just like the photon is for an electromagnetic field. It's what communicates the 'message': "There is a damned great black hole over there - and therefore you should be accelerating towards it!".
Photons don't get bent around by electric fields - so why would we expect gravitons to be affected by a gravitational field? If something in a large chunk of matter is emitting gravitons - and if those gravitons are affected by that gravity - then you'd need a second graviton to carry the message to the first one...then what communicates the message to the second graviton?!? That's just got to be wrong.
So I think BenRG is incorrect - if gravitons exist - they can escape a black hole because they aren't affected by gravity.
The business of the 'fossil field' is to do with the relativistic issues of matter entering the black hole being 'frozen' at it's surface by time dilation.
SteveBaker (talk) 21:19, 22 December 2008 (UTC)[reply]
I have lots of other problems with gravitons, or any gauge bosons, really. They all have in common that a seemingly infinite number of undetectable particles be constantly bouncing between every pair of elemental particles in the universe. StuRat (talk) 21:51, 22 December 2008 (UTC)[reply]
StuRat: Massive particles that fall into a black hole have gravitational fields of their own. When they get close to the event horizon, you feel the combination of the two fields, which is similar to the field of a uniform (no hair) black hole of the combined mass. Over time the inhomogeneities in the field radiate away and you end up with a larger uniform field.
SteveBaker: Everything gravitates, including the gravitational field. That's one way of understanding why GR is nonlinear: the field interacts with itself. It's classical electromagnetism that's the oddball here—all of the forces in the Standard Model are nonlinear, including electromagnetism, though the nonlinearity there is small. The strong force is extremely nonlinear: the gluons have color charge and the coupling constant is very large.
StuRat, and everyone really: Feynman diagrams are ridiculously oversold in popular books, to the point that a lot of people seem to think that quantum field theory is about virtual particle exchange and you can't have a quantum field theory without virtual particles. That's not how it works at all. Quantum field theory is about fields. Virtual particles and Feynman diagrams are a kind of series expansion for the underlying field theory, similar to a Taylor series. Taylor series are great when they converge quickly, not so great when they converge slowly, and useless outside the radius of convergence where they don't converge at all. The perturbation theory in Feynman diagrams is the same way. You can't "see" the whole field theory through Feynman diagrams, and what you do see is arguably not conducive to a good understanding of the full theory. If you do gravity with Feynman diagrams you have to choose a fixed background and perturb around that, which means you lose the background-free character of GR, which is really the most interesting thing about it. The Standard Model forces also have some of that background-free character—that's what the "gauge" in "gauge boson" means—and that's lost when you think of them in terms of Feynman diagrams. So forget about virtual bosons. "Virtual graviton exchange" is at best just another term for "gravitational attraction", and at worst it's a strictly less general notion. When I see questions about virtual gravitons I usually try to answer as though they were about fields, as I did above. -- BenRG (talk) 08:18, 23 December 2008 (UTC)[reply]

Thats an interesting point. So is there still a black hole inside?Trevor Loughlin (talk) 14:33, 22 December 2008 (UTC)[reply]

Speed of light in another universe

Could the laws of physics be different in another universe, and if so, could the speed of light be different as well? I heard somewhere that the laws of physics are mostly dictated by the nature of the particle that make up our universe. While this sound plausible, I'm sure it's far from fact. If it were true, could an alternate universe composed by alternative particles making what we would call exotic matter (but to them would be ordinary) then have different rules on how that matter acts? Tiailds (talk) 07:44, 22 December 2008 (UTC)[reply]

Can't quite see why you shouldn't be able to have a different universe with a different speed of light (in a vacuum!). The crux of the matter is that if you change one thing you have to see what else is affected. You can't just say "in this universe apples fall up". You would then also have to describe all the related issues e.g. how apple trees grow, what makes apple, falling up different from pears, falling down. Since it has taken many generations of physicists to sort out how things are connected in this universe, and we still haven't figured it all out to the last detail, I'd assume you'd have to live with some approximation of how a universe functions where light traverses a vacuum at a different speed, because it would take too long to define everything. 76.97.245.5 (talk) 08:28, 22 December 2008 (UTC)[reply]
By definition, we cannot know the laws of physics in another universe. Icek (talk) 08:38, 22 December 2008 (UTC)[reply]
It's certainly true that we can't know. Indeed we can't even known that there ARE other universes (although it would certainly explain a whole lot if there were). There are some things like the speed of light or the charge on the electron that I think most physicists would be happy to accept could be different because we have no reason to know why they have the exact values they have in our universe (although the strong anthropic principle is some sort of a reason). But there are other things - more in the realms of mathematics - that I think we'd be unwilling to agree could possibly be different - the value of PI for example. It's fairly easy to imagine a universe where light travelled 0.000001% faster than it does in ours - but it's impossible to imagine a universe in which PI differs AT ALL from the value in ours!
Somewhere between those limits we have physics/math things like dimensional analysis that say things like that 'velocity = frequency * wavelength' has to be true because velocity is measured in meters-per-second, frequency in 1/seconds and wavelength in meters - so for the dimensions of the equation to balance, it pretty much has to be that. A universe in which (say) velocity equals frequency SQUARED times wavelength would be seriously screwy! So I think we could be reasonably comfortable saying that some of the equations we're happy with in examining our universe would be fundamental enough to hold in other universes...but others, less so.
If we're happy to entertain things like different values for the speed of light - then we would CERTAINLY expect matter to behave radically differently - it doesn't take much of a change in the charge on the electron to make all atomic matter impossible and reduce the universe into a bunch of black holes with nothing else present but vacuum. Matter with bizarre behavior (and therefore 'exotic' by our standards) would certainly be expected if any of the fundamental constants were off by a relatively small amount. So unless there is some deep underlying reason why the speed of light, the charge on the electron, the universal gravitational constant and the plank length are what they are - then they could be different - and all sorts of wierdness would result.
But other universes might be different in other ways too - more or fewer spatial dimensions certainly seems like a reasonable possibility - and that would result in things being so seriously out-of-whack that it would be almost impossible to imagine the consequences. If string theory is right and our universe has 26 dimensions (or however many it is that string theorists are predicting this week!) - of which 23 are 'tightly coiled up' or 'small' and therefore impossible for us to detect - then who is to say that with different fundamental constants (like the speed of light) that they wouldn't have 17 'small' dimensions and 9 normal dimensions. Right there - we'd have a universe that would be hard to imagine and impossible to visualise.
SteveBaker (talk) 13:57, 22 December 2008 (UTC)[reply]
Indeed, if a universe would have 2 or 4 (ordinary) spatial dimensions and by analogy with the universe would have an attractive gravitational force proportional to 1/r or 1/r3, respectively, then planetary orbits wouldn't be stable - the only closed orbits would be circular (and I think this is true for any dimension other than 3; but if we look closely orbits aren't closed in the real universe either, even in the 2-body case - in General Relativity there is perihelion precession, and, even worse, gravitational radiation). Icek (talk) 14:29, 22 December 2008 (UTC)[reply]
This is kind of off-topic, but in 2+1 dimensional general relativity there are no closed orbits, even unstable, even approximately. In fact, test particles do not accelerate toward massive objects in 2+1 dimensional general relativity. It's very different from 2-dimensional Newtonian gravity. -- BenRG (talk) 22:17, 22 December 2008 (UTC)[reply]
It's hard to say what it means to "change the speed of light". Units like meters and seconds are based on human beings—a meter is about the size of a person and a second is about how long it takes to have a thought. If the speed of light were twice as fast, then we'd think twice as fast, so our second would be half as long, so the speed of light in human-based units would be the same. In some sense 299,792,458 m/s is not a physical constant at all, it's a biological constant, a measure of how much low-level signaling in the brain is needed for conscious thought. What about the speed of light in "natural units"? Well, in the usual "natural units", Planck units, the speed of light is 1 and can't be changed because it's how those units are defined in the first place. So our attempt to change the speed of light failed to change its value in natural units or in human units—in what sense did it change at all? People sometimes say that you can avoid those problems by only talking about unitless constants, but that's not really true either. For example, the proton/electron mass ratio (about 1836.15267) is a unitless constant, but it's not clear what it means to change it in isolation. The problem is the arbitrariness of coordinates. The Standard Model of particle physics has about 26 continuous parameters, but not any particular 26, in the same way that space has three dimensions, but not any particular three. You can't change your x coordinate unless you know how the x coordinate is defined, and you can't change your x coordinate "without changing anything else" unless you know what the other coordinates are too. Nonetheless, you can choose a different point in the 26-dimensional parameter space and get a description of a different universe. It's just hard to say exactly what you've changed.
Most if not all of the continuous parameters in the Standard Model can be interpreted as properties of particles, so in that sense "the laws of physics are mostly dictated by the nature of the particles". In fact (as John Baez mentions in the above-linked web page) most of the constants can be interpreted as properties of the Higgs boson, which is what makes it so interesting. -- BenRG (talk) 22:17, 22 December 2008 (UTC)[reply]
I disagree. I understand what you're saying: I suppose you can say that if all of the fundamental constants that relate to time and distance were scaled by the exact same amount - and all of the ones that relate to mass by another...and so on...then the 'absolute' values for these constants could change and you wouldn't notice a difference...and unless you could somehow transport a stopwatch, a tape measure and some kilogram test masses from our universe to the other - there would be no way to know that there even was a difference. So in that sense, two universes could have different speeds of light - and so long as all of the other time and distance values were scaled proportionately - you'd never know.
But if the speed of light was different - yet none of the other constants containing time and distance were ALSO different - then things would behave differently in fundamental and noticable ways. Restating things in Plank units doesn't change that fact. In the article you linked, the masses of the quarks (in Plank units) are now fundamental constants. In that scheme, if the RATIO of the masses of the charm and strange quarks were different than in our universe but the ratio of the up and strange quarks were exactly the same - then we'd have a different universe no matter what units or scaling you apply to it. Unless we can find a fundamental reason WHY the quarks have those particular masses - so we can eliminate all of the quark-masses from the list of constants and relate them all to something more fundamental - then they can (in principle) be different in other universes. But worse still - many of the constants are dimensionless. Change one of those and the universe is different no matter what because you can't compensate for that change.
But in the end we don't (and can't) know...it's perfectly possible for another universe to have no analogs of matter or energy or time or space or anything else - and for our universe to lack their ubiquitous snark, blarth, pling (and anti-pling) upon which all of everything is based on their side of reality. SteveBaker (talk) 14:21, 23 December 2008 (UTC)[reply]

Particle-wave separation/fusion

In light, can the particle ever disappear and leave only the wave, only to return a few instants later? 69.157.229.14 (talk) 08:16, 22 December 2008 (UTC)[reply]

No, because that's complete nonsense. Light doesn't have separate wave and particle parts. Algebraist 08:35, 22 December 2008 (UTC)[reply]
Have a look at Wave–particle duality.76.97.245.5 (talk) 08:39, 22 December 2008 (UTC)[reply]
Light is neither a wave NOR a particle - but it behaves a bit like both of them - sometimes (eg in diffraction) it seems more like a wave and in other times (eg in the photoelectric effect) it seems more like a particle. But these are just ways to describe it's behavior in more or less familiar terms. Light (or more properly 'electromagnetic radiation') is quite different from other kinds of 'stuff' in the universe. So you can't separate out the wave and the particle behaviors because they aren't separate - they are just ways of thinking about something that lies beyond our normal day-to-day human experience. As Jayron32 is fond of telling us "Light is light, however, and that it sometimes behaves like a wave and sometimes like a particle is not lights problem. Its ours." - which sums it up rather eloquently. SteveBaker (talk) 13:09, 22 December 2008 (UTC)[reply]
Another way to say it is that people have made up the concepts of waves and particles. They both happen to have some properties of light. — DanielLC 17:59, 22 December 2008 (UTC)[reply]
Did I just get quoted there (ego swells)... Thanks Steve! I have nothing further to add, since SteveBaker already said what I was going to say!. --Jayron32.talk.contribs 20:19, 22 December 2008 (UTC)[reply]
Don't get too excited - we're all bored with that one. Now you've got to find something cogent and witty to say about the lipid layer in the eye.  :-) SteveBaker (talk) 20:45, 22 December 2008 (UTC)[reply]
See the lipid layer is the lipid layer, however, and that it sometimes behaves... oh fuggit. It doesn't work as well there... --Jayron32.talk.contribs 14:20, 23 December 2008 (UTC)[reply]
I like to use the term "wavicle" to describe light: poke it one way and it looks like a wave, poke it another way and it looks like a particle. --Carnildo (talk) 02:23, 24 December 2008 (UTC)[reply]

Storing solar lights

I got some solar x-mas lights this year. What is the best way to store them, so that they will work again next x-mas? Should I take out the rechargeable batteries? If yes, what's the best way to store them? Should I recharge them at regular intervals during the year? Should I let them drain completely before I put them away? Would it be better to run the lights in the back yard all year round than storing them till next x-mas? Other ideas?76.97.245.5 (talk) 08:48, 22 December 2008 (UTC)[reply]

It depends on the kind of battery, but it's usually better to drain them "completely" (see memory effect, but also see "deep discharge" there) and then recharge them before you store them in order to prevent deep discharge. As for the lamps, if they are incandescent lamps they wear out quickly if you turn them on and off often. Just store them. Icek (talk) 12:17, 22 December 2008 (UTC)[reply]
Thanks a lot for the info and the link. The lights are led, but I guess their lifespan doesn't improve with use either. I'll just run the batteries though my charger (that takes care if draining them, too) and then put everything away. 76.97.245.5 (talk) 12:50, 22 December 2008 (UTC)[reply]
LED's last an awful long time and can take a lot of abuse - there probably isn't much you can do to hurt them. I would keep them someplace dark and dry (more for the sake of the wiring and connections than for the LED's themselves) - but otherwise no special care is needed. The batteries (as Icek said) should probably be allowed to completely discharge - ie turn the lights on - put the solar panel someplace dark until the lights go completely off then store them in the dark so it doesn't recharge! The only kinds of rechargeable battery that don't like to be stored in a discharged state are lead/acid batteries (like car batteries) that should be kept fully charged at all times - but they don't use lead/acid for things like solar-powered lights so that's not a problem here. When you take them out next year - wipe the cover that's over the solar panel to make sure it's clean and you're good to go. SteveBaker (talk) 12:59, 22 December 2008 (UTC)[reply]
Well, Li-ion batteries don't like to be stored discharged, either. I think 40% is the optimal charge for long-term storage, at least for Li-ion polymer batteries. -- Aeluwas (talk) 13:55, 22 December 2008 (UTC)[reply]
More like 75% for all lithium batteries131.172.99.15 (talk) 14:01, 23 December 2008 (UTC)snaxalotl[reply]

Earth's rotation

Looking at the image, Earth apparently spins up every year approximately at the time of Northern Summer. Is this because of snowfall in Antarctic winter which lowers the sea level a bit? (Earth's moment of inertia is about 8*1037 kg m2) it would mean that sea levels change seasonally by maybe 0.2 m (I haven't done a proper calculation) which sounds like too much. Is there another important cause of the seasonal variation? And why was Earth rotating so slow in recent years (no leap second in the years 2000 - 2005)? Icek (talk) 10:30, 22 December 2008 (UTC)[reply]

Someone on sci.physics said that the leaves on trees contribute to the effect (most trees are in the Northern Hemisphere), but the effect of that would be the opposite of what you noticed... -- Army1987 – Deeds, not words. 10:52, 22 December 2008 (UTC)[reply]
Indeed, the faster the Earth spins the steepest the descent of the curve is, so, to me, it seems that it's faster in winter, as the above would predict. -- Army1987 – Deeds, not words. 10:58, 22 December 2008 (UTC)[reply]
No, the slower the Earth spins the steeper the descent of the curve is: Then leap seconds have to be more frequently, as can be easily seen from the diagram. A leap second means that the Earth hasn't yet spun as far as the 86400 second day would predict and we have to wait an extra second for Earth.
As for the leaves on trees: If the average height of the leaf above ground is h = 20 m, and there is A = 40 million km2 more deciduous forest on the Northern Hemisphere than on the Southern Hemisphere (an gross overestimation I think), and the effective (not quite the average) lattitude is θ = 35°, and there is one tree per At = 10 m2, and there is ml = 200 kg leaves per tree, then the change in moment of inertia is:
ΔI = 2 * r(θ) * h*cos(θ) * A/At * ml = 1.37*1023 kg m2
where r(θ) = REarth*cos(θ) is the distance from Earth's axis at the effective lattitude. This is a relative change of moment of inertia - and thus of angular velocity - of 1.7*10-15. I think I certainly didn't underestimate with my choice of the parameters. That relative change would mean a change in slope of the curve equivalent to an extra leap second (or a missing leap second) once every 18.5 million years - and that is clearly way less than what we see in the diagram! Icek (talk) 11:53, 22 December 2008 (UTC)[reply]
By the way, the relative change is also at the current limits of accuracy in measuring time. Icek (talk) 11:55, 22 December 2008 (UTC)[reply]
No expertise in this but I would expect the earth to be slower in recent years with global warming because the water would be spread out rather than at the poles. The spin up during summer may be because the water goes into the ocean instead of staying on the land. Dmcq (talk) 13:12, 22 December 2008 (UTC)[reply]
That's what puzzles me - it's that Earth rotated faster in recent years. How do you think water going into the ocean instead of staying on the land causes Earth to spin up? If some amount of mass is located at a certain lattitude θ and then gets spread out over the whole surface of Earth (yes, I'm ignoring the distribution of water and land by lattitude, I could be substantially more accurate by a detailed calculation), it will decrease Earth's moment of inertia only if it was at a lattitude smaller than arccos(sqrt(2/3)) = 35.26°, and else it will increase Earth's moment of inertia (and thus slow down Earth's rotation). Icek (talk) 14:39, 22 December 2008 (UTC)[reply]
The answer (for the seasonal variation) seems to be the exchange of angular momentum between the atmosphere and the the lower part of Earth [29]. Icek (talk) 15:16, 22 December 2008 (UTC)[reply]
The atmosphere's mass is about 5*1018 kg, so its angular momentum is about M*R2*2/3 = 1.35*1032 kg m2. The effect is a relative change of maybe 10-8; that means an average angular velocity of the atmosphere of 0.006 times Earth's angular velocity - which is just 2.7 m/s at the equator. Of course not the whole atmosphere is moving, but the speed will not be that much lower closer to the poles as my calculation assumes (i. e. not like a rigid body), and it seems realistic. Icek (talk) 15:28, 22 December 2008 (UTC)[reply]
I think we are grasping at straws here. During northern hemisphere summer, the earth is at its aphelion, its farthest point from the sun. Being farther from the sun, the earth will experience less tidal drag from the sun, thus speeding up its rotation slightly. All the bullshit about this being due to changes in weight distribution due to increased snowfall at the south pole, or leaves falling off of trees, or anything like that seems like some pretty wild stuff. None of those is going to make an effect like this, I am pretty sure its just that, since we are farther away from the sun, we spin a little faster... --Jayron32.talk.contribs 20:16, 22 December 2008 (UTC)[reply]
Yep - I agree. These other things seem vastly too small to have a significant effect - tidal forces from the sun account for annual variations - the moon's gradual slowing down in orbit and the drag due to turbulence in the core can account for longer term effects. SteveBaker (talk) 20:41, 22 December 2008 (UTC)[reply]
Here's an article that doesn't need subscribing A New Spin on Earth's Rotation. I must admit to being surprised the atmosphere has such a large effect. Dmcq (talk) 22:24, 22 December 2008 (UTC)[reply]

I have to disagree with Jayron32 - tidal forces are proportional to m/r3 (with m being the mass of the body causing the tidal forces and r being the distance from this body), so the relative strength of Moon's and Sun's tidal forces can be easily calculated. The Sun's tidal forces on Earth are a bit less than 1/2 that of the Moon, and the difference between the Sun's tidal forces at aphelion (152.1 Gm) and perihelion (147.1 Gm) is a bit less than 1/20 that of the Moon's total tidal forces. That means that the seasonal change due to this effect should be less than the long term slowing due to tidal forces. But from the diagram alone one can see that the total seasonal variation is far larger. Icek (talk) 05:13, 23 December 2008 (UTC)[reply]

I disagree. You're comparing long term slowing due to the gradual slowing down of the moon in it's orbit to seasonal slowing - you should be comparing monthly effects of the moon to the annual effect of the sun. The moon raises tides comprising around 100,000 cubic kilometers of water - that dwarfs all of the other effects described (leaves, etc) - and even 1/20th of that (seasonally) is an immense variation. We'd probably see even greater lunar-monthly day-duration changes were it not for the dampening effect of the liquid core and the large inertia of the earth...but an effect that happens on an annual basis is long-lived enough to produce a measurable effect. SteveBaker (talk) 13:54, 23 December 2008 (UTC)[reply]
I don't think you are correct here: The tides do not directly change the effective moment of inertia by creating bulges of water, because these bulges don't rotate with Earth. So the change in rotation rate is by friction only, and there shouldn't be an increase in rotation rate when the tidal forces become weaker, but the diagram clearly shows an increase.
Regarding the atmospheric effect, note that this is not a redistribution of mass as in the cases of falling leaves and melting polar glaciers; it is that 2 parts (there are actually more than 2, but you get the idea) of Earth have different rotation rates, and we're measuring only one part. The atmosphere spins up in Northern winter and takes angular momentum from the rest of Earth beneath it. Icek (talk) 14:59, 23 December 2008 (UTC)[reply]
But there's just not enough atmosphere to have that effect! The mass of the atmosphere relative to the solid part of the earth is just not enough to cause such rather large changes in momentum. One second per year seems rather high for such effects, even if the entire atmosphere shifted to between the poles over the course of a year, we probably wouldn't see such effects. Yes, there are effects slowing the earth down and speeding it up due to redistribution of mass due to "tidal bulges" and the like, but even if the earth was a perfectly rigid ball, there would be a drag effect caused by the force of gravity, which is pulling against the spin of the earth, and I would think THAT effect is greater than any caused by such esoteric and tiny factors as Northern Hemisphere weather... --Jayron32.talk.contribs 18:21, 23 December 2008 (UTC)[reply]
Atmosphere moment of inertia (~2/3 M R^2): ~1.3 ×1032 m2 kg. Moment of inertia of solid Earth (~2/5 M R^2): 8 ×1037 m2 kg. So the ratio of the atmosphere to the solid earth moment of inertia is 1.6 parts per million. A variation of one second per year is 1 part in 32 million. In other words, you only need a 5% change in the angular momentum of the atmosphere to add 1 second to the angular momentum of the solid Earth (or vice versa). Dragons flight (talk) 18:52, 23 December 2008 (UTC)[reply]
Jayron32, how do you think that "a drag effect caused by the force of gravity..." will change the angular momentum of a rigid ball? How do you get a torque? Icek (talk) 21:08, 23 December 2008 (UTC)[reply]

New image and new headline

Because the annual variations are hard to see in the original image, I plotted the same data over the course of a year. The thin green lines connect consecutive days; the almost horizontal lines are distracting, they are the connections between Dec 31 and Jan 1 of the next year. The big spike occurs on 5/19/1976, it is obviously an error because the next day ∆t is given as 0. — Sebastian 22:02, 23 December 2008 (UTC)[reply]

The next highest two peaks occur on Jan 5 1833 and at the beginning of Feb 1983. — Sebastian 22:15, 23 December 2008 (UTC)[reply]

the nearest galaxy

hello, i was wondering, what is the nearest galaxy to ours? —Preceding unsigned comment added by 92.13.130.144 (talk) 18:35, 22 December 2008 (UTC)[reply]

The Milky Way (the galaxy which we are in) is part of the Local Group, a cluster of about 35 galaxies (that article, and ones it links, has a lot of good information about nearby galaxies). The nearest spiral galaxy is the Andromeda galaxy (also in the local group). The nearest galaxy of any shape is a little harder to describe. The nearest one of any significant size is the Large Magellanic Cloud. There are dwarf galaxies which are closer, such as the Canis Major Dwarf Galaxy, which is only 42,000 light year from the galactic center, or 25,000 light years from earth. According to the article it "is now thought to be the closest neighbouring galaxy to our location in the Milky Way". However, it was only discovered in 2003 (the Sagittarius Dwarf Elliptical Galaxy held the previous record), so there may be a possibility that an as-yet-undiscovered galaxy is closer still. -- 128.104.112.113 (talk) 18:48, 22 December 2008 (UTC)[reply]

Acceleration of particles in a secluded system

I read that space-time travel could be reached by means of acceleration of particles in a secluded system. What would this mean concretely ? 69.157.229.14 (talk) 20:54, 22 December 2008 (UTC)[reply]

Er..."space-time travel"? I think we first need to ask what you mean by that. Travelling in space over the course of time is what we normal people call "moving". And it's impossible to achieve movement without some acceleration of particles...but if particles are accelerating then you have movement in space (and time is doing it's usual thing). It doesn't matter whether they are 'secluded' or not.
If you actually mean just "time travel" - then no.
I think you need to clarify what exactly it is you are talking about here. SteveBaker (talk) 21:28, 22 December 2008 (UTC)[reply]

I'm not sure, but it has something to do with this. [30][31] 69.157.229.14 (talk) 21:51, 22 December 2008 (UTC)[reply]

The first link is a joke, the second is a video of a sculpture of a machine that doesn't work.... —Preceding unsigned comment added by Philc 0780 (talkcontribs) 22:20, 22 December 2008
The second video claims to be of a time machine that travels into the future at a rate of 1 SPS (which one assumes means 'second per second') - which is an old and rather lame joke. However, taken at face value, it DOES work - it just doesn't do anything useful. SteveBaker (talk) 13:41, 23 December 2008 (UTC)[reply]

Tech Bump

Look at the palm of your right hand (if you are right handed). As you face it, I believe you may find a noticable bump in the bottom left corner. I'm convinced it's a souveneir from all the time that palm lies on a table as the hand grasps a mouse. For the palm of graphics developers, this bump is absolutely inflamed and even painful. Is there a name for this scar? Sappysap (talk) 23:36, 22 December 2008 (UTC)[reply]

Please excuse my "original research" but, no bump here! -hydnjo talk 23:47, 22 December 2008 (UTC)[reply]
The hamate bone is often palpable near the hypothenar eminence. This response is simply descriptive regarding the location you describe - the RefDesk is NOT a place to ask for medical advice. --Scray (talk) 00:10, 23 December 2008 (UTC)[reply]
Sorry, where was the request for medical advice? WP:OR none found here. Edison (talk) 00:25, 23 December 2008 (UTC)[reply]
Thank you, Edison. Sappysap (talk) 00:27, 23 December 2008 (UTC)[reply]
I just felt the need to be explicit about what I was saying - that I was describing a structure at that location, as requested, but not explaining the "inflamed and even painful" bump described by the OP. I guess this came off wrong, since Sappysap thanked you. Sorry about any inferred negativism. --Scray (talk) 00:39, 23 December 2008 (UTC)[reply]
Thank you, too, Scray. Sappysap (talk) 00:47, 23 December 2008 (UTC)[reply]
Have you looked at Callus? Irritation and chronic Inflammation might also hold interesting info for you. 76.97.245.5 (talk) 00:15, 23 December 2008 (UTC)[reply]
I am right handed but I hold the mouse with my left, so the mouse bump should be left. Thinking about how the hand would be resting on the table it would be sustained by the flesh at the lower, inner side of the outer edge of the hand. This flesh would be slightly shifted upwards and there would be a bump. Now I'm anxious not to really look at my hands to avoid original research. 95.112.186.8 (talk) 07:53, 23 December 2008 (UTC)[reply]
First I went to Repetitive strain injury which seemed to fulfil the condition in concept only, the best is Splints even though it's for a horse, it seems closer to what you mean. Take from it what you can, Julia Rossi (talk) 10:00, 23 December 2008 (UTC)[reply]

Secondary minerals?

I was trying to improve a "random" page (on Wulfenite) and came across the term "secondary mineral". Searching WP, I found many pages that use the term, but no wiki links for it. Even though I am no geologist, I can guess what this term means, but it seems as though there should be a WP page to cite, or maybe there is a more precise term. Note that I am NOT asking for a definition - I'm looking for an appropriate wiki-link. Thanks. --Scray (talk) 23:58, 22 December 2008 (UTC)[reply]

Minerals mentions secondary oxidation under "Sulfate class". Maybe you, or s.o. else could add a paragraph, then everyone could link there and we wouldn't need a new page. 76.97.245.5 (talk) 00:25, 23 December 2008 (UTC)[reply]
Thanks. That content (in Mineral) is fairly narrowly-focused, and I don't have the knowledge to make a broader statement defining secondary minerals. Anyone else? --Scray (talk) 00:41, 23 December 2008 (UTC)[reply]
I referred this question to the Talk:Mineral page. --Scray (talk) 01:00, 23 December 2008 (UTC)[reply]
Secondary minerals are those formed under a weathering or oxidizing environment at or near Earth's surface, most often used in discussing surface expressions of ore deposits (least that's where I've seen the term most). That said, perhaps the best place for clarification would be the weathering article ... still thinking. Vsmith (talk) 14:45, 23 December 2008 (UTC)[reply]
For those interested, this was resolved very nicely by Vsmith! --Scray (talk) 03:38, 24 December 2008 (UTC)[reply]

December 23

Material with highest melting point?

Hello -- what material has the highest melting point? I've found listings for the melting points of various pure elements by melting points, but nothing which describes what happens to the melting point when you create alloys, etc. Is there anything that has a higher melting point than carbon in amorphous or graphite forms? --98.217.8.46 (talk) 00:51, 23 December 2008 (UTC)[reply]

The melting point article you cited lists Tantalum hafnium carbide as having an extremely high melting point. The article for Tantalum hafnium carbide says that it is the compound with the highest known melting point. --Scray (talk) 00:54, 23 December 2008 (UTC)[reply]
Wow! That's impressive - it's about 80% of the temperature of the surface of the sun! SteveBaker (talk) 21:17, 23 December 2008 (UTC)[reply]
So, what's a refractory compound? — Sebastian 22:33, 23 December 2008 (UTC)[reply]
Never mind, found it: Refraction (metallurgy). — Sebastian 22:35, 23 December 2008 (UTC)[reply]

TASTE BUDS

WHY WE DON'T GET SWEET TASTE WHEN WE HAD ALREADY A SWEET DISH BEFORE IT?AS IT IS NOT IN THE CASE SOUR TASTEM.meghajain (talk) 02:14, 23 December 2008 (UTC)[reply]

See "Neural adaptation" and "Habituation", and don't type in all caps. Our senses grow dull to a constant input. We stop feeling our clothes soon after we put them on, and we don't smell the pig farm next door after a week or two. --Milkbreath (talk) 02:25, 23 December 2008 (UTC)[reply]
I'm not an expert, but if I remember correctly sour taste is caused by a low (=acidic) pH inside certain receptor cells, I guess that internal pH is regulated by the cell with only short delays, while sweet substances stick to receptors on the outside for a longer time. Regarding sweetness, an interesting substance to look up is miraculin. Icek (talk) 06:21, 23 December 2008 (UTC)[reply]

Science/Humanities question on Egypt's buried tombs

"70 percent of Egypt's ancient monuments remain buried under sand," a news article on two 4300-year-old tombs quotes an official as saying. Was it desertification? Thanks in advance. Imagine Reason (talk) 03:13, 23 December 2008 (UTC)[reply]

In any case it is a policy of modern archaeology not to exploit completely an archaeological site. The idea is that in the future more advanced tools and techniques will be available, and doing all the job today would destroy important elements that tomorrow we could be able to detect. This is indeed what has happened in the past: many Egyptian lefts passed 4000 years in perfect state to their end by the enthusiastic hands of archaeologists.--PMajer (talk) 12:34, 23 December 2008 (UTC)[reply]
The ancient Egyptians placed tombs and burial pyramids in areas that were desert at the time, since that allows both buildings and bodies to last longer. They lived in moister areas, however. StuRat (talk) 14:44, 23 December 2008 (UTC)[reply]

Why is the static between channels loud?

Tuning an analog radio with the knob, one hears static between stations. The static typically has a loudness about equal to the stations themselves. But how is this possible, when the real stations have the same noise layered onto the transmission, and yet manage to produce silences in the programming that are much quieter than the sound heard when there's no transmission at all?--84.70.209.18 (talk) 03:49, 23 December 2008 (UTC)[reply]

See Automatic gain control for a start--GreenSpigot (talk) 04:41, 23 December 2008 (UTC)[reply]
In short, when you hear static the radio is trying to pick up a weak signal and not really finding one. When you hear silence in a program, it has found the carrier signal and "knows" how loud a broadcast it could carry and therefore that the silence is desired. --Anonymous, 06:09 UTC, December 23, 2008.
AGC reduces the sensitivity of an AM radio receiver when a broadcast signal is being received. The AM RF signal is constantly varying in strength (amplitude) due to the sound modulation imposed on the RF. The AGC circuit smooths the amplitude variations to provide a DC voltage whose amplitude is determined by the average strength of the received signal.(The smoothing capacitors in a simple power supply similarly smooth the AC ripples superimposed on the rectified DC.) The smoothed AGC voltage is used to decrease receiver sensitivity. The stronger the received signal, the higher the AGC voltage and the more the sensitivity is reduced. Silent moments in a signal stay silent because receiver sensitivity remains reduced; that is because the carrier wave (unmodulated signal) is still being received and the carrier wave produces AGC voltage. AGC not only prevents atmospheric noise (white-noise static) from marring a received signal, it also prevents overloading of audio stages when a strong signal is received. Overloading would cause audio distortion. — GlowWorm
Yes - exactly. The radio needs 'AGC' (Automatic Gain Control) because the amount of radio energy received by the radio depends dramatically on how far you are from the transmitter (amongst a bunch of other factors). If you are 1 mile from the transmitter, you get about four times as much energy than if you are 2 miles away and a hundred times more than if you are 10 miles away. This HUGE variation in the amount of signal the radio receives would result in the volume produced by the radio varying by perhaps a factor of ten thousand depending on how close to the transmitter you are, etc. So the radio has to automatically alter the 'gain' (volume) of the signal to try to keep the same audio levels no matter where you are. As GlowWorm says - that circuit has to monitor the level of the 'carrier' wave and use that to adjust the sound volume. When there is no carrier (ie, between stations), the AGC circuit has nothing meaningful to latch onto - and the result is a lot of random noise with the AGC turning up the volume as far as possible in the hope of picking up some very faint signal. With many modern radios, there is additional circuitry to turn the volume down to zero when there is no carrier detected - and in that case you don't get any noise between channels. But that kind of sophistication has not always been present - and cheaper/older radios don't have it. SteveBaker (talk) 13:37, 23 December 2008 (UTC)[reply]

Ion slowing down in plasma

What electrical effect, if any, would cause an ion to slow down in a sparse cloud of ions i.e. outer space? I want to figure out how well the Earth would hold a charge. In case anyone's wondering, I am not a geocidal maniac planing on causing the Earth to spiral into the sun by putting electron guns on the top of a launch loop to build up a charge of over 8*1017 Coulombs and letting the Abraham-Lorentz force take care of the rest. That would just be silly. Why would you even bring it up? — DanielLC 06:47, 23 December 2008 (UTC)[reply]

Short and not-in-depth answer: You have to take into account the magnetic fields in space - homogeneous magnetic change only the direction of the velocity vector (except for the Abraham-Lorentz force of course), but inhomogeneous fields also cause a change in kinetic energy. Icek (talk) 15:09, 23 December 2008 (UTC)[reply]
Hmm - magnetic fields never change the total kinetic energy a charged particle, regardless of the inhomogeneity. I think what the OP is looking for is Coulomb scattering. An ion traveling through a "cloud of ions" (see plasma (physics)) interacts with the background ions via the coulomb interaction, thus transferring some of its kinetic energy to the background ions. This leads to an exponential decay of momentum and energy of the incident ion. I won't comment on the rest of the plans you don't have. --Bmk (talk) 05:35, 24 December 2008 (UTC)[reply]

What happens to sexual desire after penectomy?

Are there any reliable studies about sexual desire in men after a penectomy, particularly if so much has been removed that orgasm is impossible (or at least nearly impossible)? Without being able to orgasm do they go crazy from not being able to orgasm or lose their sexual desire completely? If not reliable studies have been conducted, anecdotal evidence is fine. 67.184.14.87 (talk) 06:54, 23 December 2008 (UTC)[reply]

I don't know but I strongly discourage original research. 95.112.186.8 (talk) 07:56, 23 December 2008 (UTC)[reply]
Many things[32]... Julia Rossi (talk) 09:54, 23 December 2008 (UTC)[reply]

nutrition facts

If a jar of pickles contains 10 pickles and each pickle contains 500 mg of sodium, if one eats all the pickles and drinks the juice does one consume 5000mg sodium. In other words is the juice factored in when computing sodium in such things as pickles,olives, or canned tuna for that matter.206.251.18.119 (talk) 11:50, 23 December 2008 (UTC)[reply]

Probably not in the case of pickles or olives. The nutrition 'panel' on the side of the jar tells you only the content of one "serving" - which is likely to be 1 pickle - not 1 pickle and 3 tablespoons of liquid (which is probably more 'brine' - used as a preservative - than 'juice' from the pickles)...so I suspect they leave out the information about the content of the liquid. That's probably NOT the case for canned tuna because it's much harder to separate liquid from solid)...but it's hard to know for sure. One has to be VERY careful reading those nutrition panels because very often their estimate of 'serving size' is very small indeed! SteveBaker (talk) 13:26, 23 December 2008 (UTC)[reply]

Genetic Misogyny and sex war?

Could the counter-instinctual mistreatment (to Western eyes) of women and girls in the tribal areas of Pakistan, Afghanistan and the Middle east have some genetic component, and would it ever be acceptable to test this theory on a large scale with DNA analysis? Also could a sex war ever break out, given that women even in secular insurgency groups such as the Tamil Tigers can be a ruthless fighting force? —Preceding unsigned comment added by 80.2.194.166 (talk) 13:24, 23 December 2008 (UTC) 80.2.194.166 (talk) 13:29, 23 December 2008 (UTC) Trevor Loughlin[reply]

Not a genetic component, no. However, despite the claims of those who throw acid on women that "the Koran tells us to do it", it says no such thing. It says that both men and women should "dress modestly", but it's a long way from that to "you should throw acid on any woman who shows her face in public". It's strictly a cultural belief, which they use their religion to justify. As for a gender war with actual shooting, I'd say women would lose that war unless they outnumbered the men considerably, as they would have to tend the children, as well, and wouldn't have weapons or military training. Striking women who deny men sex and food is one possibility, but only after the possibility of violence against the strikers has been removed from the table. StuRat (talk) 14:37, 23 December 2008 (UTC)[reply]
Was "striking" an adjective, and if so which sense of "striking," or a verb in that last sentence? Edison (talk) 17:32, 23 December 2008 (UTC)[reply]
I meant it as "going on strike". However, such striking would be more effective if the women were strikingly beautiful, and less effective if the men were striking them down. StuRat (talk) 19:34, 23 December 2008 (UTC)[reply]
A "sex war", or even widespread dissatisfaction, can only break out if (1) the women believe they are being mistreated and (2) the women are not willing to tolerate the mistreatment. I don't think either criterion is fulfilled as of now; a person's beliefs are heavily influenced by her upbringing, and the Muslims are no exception. --Bowlhover (talk) 18:06, 23 December 2008 (UTC)[reply]
Have a search for the pink vigilantes in India to see how women can overcome such obstacles. There's many other less extreme examples in India also where women are getting more control of their fate. Dmcq (talk) 22:05, 23 December 2008 (UTC)[reply]
The article here is Sampat Pal Devi for the Pink Saree Gang, and there's the late Phoolan Devi, dacoit leader. The less extreme measures include microfinance for women. Julia Rossi (talk) 22:57, 23 December 2008 (UTC)[reply]

Not so bright question...

How do I compare the brightness of a TV projector, given as lumens, with an LCD TV, given as cd/m2 ? Obviously, the first step is to figure out the area of the projected image and do some math and conversions. But the second step is to figure out what portion of the lumens produced by the bulb actually makes it to the screen. How do I do that ? StuRat (talk) 14:29, 23 December 2008 (UTC)[reply]

Assuming the thing is focussed and aimed correctly, you can assume that all of the lumens produced by the projector make it to the screen (after all - where else would they go?) - but the complication is in how efficient the screen is. There is a trade-off between viewing angle and brightness to be made in the actual material of the screen. Some screens are covered with tiny retro-reflectors that push most of the light back out towards the projector - giving the screen a very narrow visibility angle - but a heck of a lot of brightness within that angle. Others are closer to perfect lambertian reflectors and they spread the light out equally in all directions - so you can see the image equally well from any place in the room - but it's going to be rather dim. Similar considerations apply with your LCD TV - many LCD's have very tight viewing constraints - others don't - so the number of candelas being emitted by each square meter of the screen can be identical between two devices - and yet one appear much brighter than the other. Remember - a one milliwatt laser can take out your eye - a 10 watt light bulb is scarcely bright enough to read by! The light bulb is visible through 360 degrees - the laser through some tiny fraction of a degree. SteveBaker (talk) 14:48, 23 December 2008 (UTC)[reply]
Surely there isn't a 100% efficiency at the projector, and much of the light generated becomes heat. Some might also be absorbed or scattered by dust particle in the air between the projector and screen. StuRat (talk) 14:58, 23 December 2008 (UTC)[reply]
The scattering due to dust is negligable and the light that is absorbed inside the projector ought not to have been counted in the lumens value you got from the manufacturer (they ought to be stating the brightness on the output side of the final lens - not the brightness of the light source) - so it's only light absorbed by the air (which is pretty transparent over the distance a projector is likely to be used!) or in the screen itself (which ought to be designed to reflect a large percentage of the incoming light - not absorb it as heat). I think you can ignore those concerns. The screen, however, is a critical part of the equation here. SteveBaker (talk) 17:07, 23 December 2008 (UTC)[reply]
Sure they should give the actual output lumens of the device, but it's in each manufacturer's interest to be deceptive and instead list the output of the bulb, as that will make it appear that they have a brighter image. So, unless you have a source that confirms that they do the right thing, I'd tend to doubt that they would. StuRat (talk) 17:32, 23 December 2008 (UTC)[reply]

Also, what's the range of brightnesses for TVs by type (LCD, plasma, DLP, projector), etc. ? StuRat (talk) 14:58, 23 December 2008 (UTC)[reply]

Also, what is the range of brightness between between the white and black areas of various types of screen? For LCD and CRT screens, it varies with the width of the black area. Text, which has narrow lines, is dark grey rather than black. Wider black areas are a darker grey, approaching full black. It would much improve readability if text could be made jet black. I once had a black-and-white CRT computer monitor - the contrast for lettering was much better than on a color LCD or CRT monitor. – GlowWorm —Preceding unsigned comment added by 98.17.46.132 (talk) 15:55, 23 December 2008 (UTC)[reply]
The range of brightness (usually called "The Contrast Ratio") is indeed an important concern for overall image quality. If the image is nice and bright - but the contrast ratio is crap - the colors will be very 'pastel' and the black areas will be grey...not a very appealing image. For bright, vibrant colors and deep blacks you need a high constrast ratio AND a bright image. This is where you see a big difference between LCD projectors (where a bright light is shone through an LCD panel to form the image) versus the DLP projectors where the light shines onto about a million tiny mirrors that can be moved such as to either reflect light out through the lens to the screen - or off to the side somewhere. For a long time, DLP's had vastly better contrast ratios than LCD projectors - but I've been out of that business for a couple of years now and things may have changed since then.
The issue of displaying thin vertical lines is to do with the rate at which the display can go from dark to light along the scanline - and that's 'bandwidth' - and 'modulation depth' - which are generally rolled into one "resolution" figure in the spec sheets for the display - but the measurement of it is something of a black art and comparing two monitors on the basis of this number is a bit more subjective. If you can get a set of test patterns displayed on the thing (like a TV 'test card') you should be able to directly compare resolutions. Horizontal lines are a different problem and relate more directly to the monitor's raw screen resolution and have nothing to do with bandwidth. However, StuRat isn't asking about any of those things.
SteveBaker (talk) 17:07, 23 December 2008 (UTC)[reply]

Male genitilia

This question has been removed. Per the reference desk guidelines, the reference desk is not an appropriate place to request medical, legal or other professional advice, including any kind of medical diagnosis, prognosis, or treatment recommendations. For such advice, please see a qualified professional. If you don't believe this is such a request, please explain what you meant to ask, either here or on the Reference Desk's talk page.
This question has been removed. Per the reference desk guidelines, the reference desk is not an appropriate place to request medical, legal or other professional advice, including any kind of medical diagnosis or prognosis, or treatment recommendations. For such advice, please see a qualified professional. If you don't believe this is such a request, please explain what you meant to ask, either here or on the Reference Desk's talk page. --~~~~
If you have concerns about anything on your body, please seek advice from a medical professional and not random people on the interwebz. --Jayron32.talk.contribs 18:10, 23 December 2008 (UTC)[reply]

A tool for measuring cubic volume

Hi. I would like to measure the cubic volume of some irregular shaped products, specifically folded clothing (think of a folded up pair of jeans with a belt, or a folded t-shirt with a hanger). Does anyone know of a commercially available device that is suitable for this task? Thanks for your help. sparkl!sm hey! 16:27, 23 December 2008 (UTC)[reply]

I don't know of any machine for doing this - but you hardly need one. Archemedes principle applies here.
For most irregular shapes - take a large, utterly full, container of water - push the item completely below the surface (water will of course overflow the sides of the container) - then remove the object and measure the amount of water that it takes to completely refill the container (using a kitchen measuring jug for example). The volume of water you have to add to restore the level is the volume of the item. Sadly, for folded up clothing, this is a little tricky since the object is absorbant - and the 'volume' of something like cloth is a little hard to define because it's full of microscopic holes. Probably - you're interested in the volume of the irregular shape surrounding the cloth - ignoring the spaces between the folds and between the fibres of the cloth - in which case you're going to have to seal the item in a waterproof plastic bag having compressed the clothing to the desired degree. The difficulty isn't so much in the measuring as knowing exactly WHAT you're trying to measure! For items that you really can't immerse in water like that (eg because they float or compress too much under the water pressure - or if you really DID want to take account of the space between the folds of cloth, etc) then you could replace the water in the experiment above with some other fluid - perhaps a heavier-than-air gas like SF6...you can put that stuff into something like a large aquarium tank and use an air-filled balloon as a float to tell you the level of the gas. Put a mark on the balloon and a scale up the side of the tank. Partly fill the tank with SF6 - note where the balloon floats. Drop in the item of clothing and see how much the balloon rises. Multiply the area of the bottom of the tank by the amount the balloon rises to get the volume of the item. SteveBaker (talk) 16:50, 23 December 2008 (UTC)[reply]
Funny, we did think of rigging up a displacement theory solution, but it isn't really practical.
You are correct, the actual volume I am interested in is the volume taken up by the garments when they are stacked - so the space in the folds of the cloth etc is also included in the calculation. You are also correct in suggesting that there will be a degree of compression of the garments, but I can perhaps factor this into the measurement. What I am looking for is something like this, a tool that can be used accurately on an industrial level. That one isn't suitable though, the garments exceed 18 inches in some cases. sparkl!sm hey! 17:19, 23 December 2008 (UTC)[reply]
I'm guessing you want to know how much storage space is required per item. In this case, as mentioned above, the clothing can be compressed to varying degrees, but you most likely want to know how much space it takes up without being compressed at all, if you want to avoid wrinkles. I suggest taking a storage unit of known volume, like a box, and pack in as many identical items as fit comfortably. Then divide the container's volume by the number of items to determine the volume per item. StuRat (talk) 17:22, 23 December 2008 (UTC)[reply]
(ec)There might be some standard test for the volume of clothing. We could devise several different methods, which would yield different volumes. Water displacement in a plastic bag would yield a different volume from SF6 displacement because the gas could fill much more of the space between the fibers of the garment, yielding a smaller volume. Another way would be to place the garment in a container and fill it to the top with beans, marbles, rice, styrofoam peanuts, or any other small defined particles which would not soak into the fabric as a liquid or gas would, noting the volume of the filler when the garment is removed. The smaller the filler particles, the smaller the measured volume. Another approach would be to take a box of a certain size, like 1 cubic foot, and see how many of the garments fit in the one box. The variation here would be how much force you apply if any to pack them in. This is the "volume" that would be of interest to a shipper or manufacturer. Edison (talk) 17:26, 23 December 2008 (UTC)[reply]

The idea of filling a standard size box is a really good one, particularly as the garments are in a warehouse, and will be order picked into a standard plastic tray. However, the measurement method needs to be fairly accurate as well as simple, and there is a problem when the cubic volume of the garment is not a factor of the cubic volume of the tray. For example, the tray is 1.5 cubic feet, and I can fit 1 folded coat into the tray. The coat fills just over half of the tray, meaning that I cannot fit 2 coats into the tray. What is the cubic volume of the coat? Well, it is less than 1.5 cubic feet, but more than 0.75 cubic feet - this is not an accurate enough method for what is required. sparkl!sm hey! 17:43, 23 December 2008 (UTC)[reply]

If your use case is stacking the coats in that 1.5 cubic foot box, then it would be accurate enough for your purposes. If you're looking to fit the coats in a 10 cubic foot box, however, then it wouldn't be. For what it's worth, the volume-in-a-box-divided-by-number-in-a-box technique can be made more accurate by increasing the size of a box. A 0.8 ft3 coat measured with a 1.5 ft3 box gives you a size in the 0.75-1.5 ft3 range, but measured with a 15 ft3 box yields ~0.79-0.83 ft3 (between 18 and 19 coats per box). -- 128.104.112.113 (talk) 17:58, 23 December 2008 (UTC)[reply]
Exactly. If 10 garments fit in the box, the accuracy is within 10%. If 100 fit, the accuracy is within 1%. Here we're getting below the variation you would get from trial to trial. Also note that larger sizes of the same garment will, of course, take up more room. If you want the average for all sizes, ensure that you have a representative sample (with more of the most common sizes) in the trial box. StuRat (talk) 19:26, 23 December 2008 (UTC)[reply]
The tool in the link you gave us looks like it's measuring the height and width of the object as it passes under that archway. If I were building something like that I'd have a camera looking down on the object while the object moves through a scanning laser in some well-defined manner. Thats something a person with the right knowledge could rig up spectacularly easily with a PC (or even something like an Arduin microprocessor board), a laser pointer, a barcode-scanner lens and a web-cam. The precision of the device would depend on the speed of the measurements you require and resolution of the camera - both of which should be easy to control. If your garments are already passing along a conveyor of some kind - it would be really simple to make a machine to do it.
You may wonder how I know this: Well, a couple of years ago, I actually built something to scan 3D objects (You can read about it on my personal Wiki: Here). It used a Lego(!) turntable, a $25 web cam, a $10 laser pointer and a lens bought on eBay in a pack of miscellaneous lenses for $5. My objective was to make 3D models by sampling the position of the reflected laser light on the surface of the object and slowly rotating the object to get a "radial point cloud" (to use a technical term) which could then be turned into polygons and rendered as a 3D image. But using the 'point cloud' to calculate the volume from the 3D model is pretty simple - and for folded clothing - a linear conveyor belt with TWO cameras and TWO lasers (one red, one green) would do a better job. There are restrictions about things like measuring deep concavities - and the machine works best in the dark - but you could probably handle that in most industrial situations either by enclosing that section of the conveyor belt or by using much brighter lasers. The cost of the parts to make the thing would be under $100 - plus the cost of a computer to drive do the actual work - but if you only need one of these machines then the software and development costs would be your issue here. The image processing software needed to extract the coordinate information from the image on the camera is a little tricky - especially for something like clothing which might have a lot of different surface colors - if you want to discuss it in more detail - my email address can be found via my User: page. SteveBaker (talk) 20:48, 23 December 2008 (UTC)[reply]
That's pretty impressive, I must admit!
To put things into context here: this is a BIG warehouse where I need to measure (clothing) products accurately. When I say big, I mean there are 15000 different products stored there, and each week there are 1000+ new products added. Each new product must be measured. Storage in the warehouse is not a problem (the stock is palletised in large storage boxes). However, when the stock is picked for despatch to store, it is picked as individual garments into plastic trays of 1 ft3. Any combination of garment mix could be picked into the plastic tray i.e. there might be 1 t-shirt, 1 coat and 6 pairs of jeans per tray. In order to maximise the shipping operation, there must be minimal gaps left in the tray once the product is picked into it. This is achieved by accurately measuring the products when they first arrive at the warehouse (goods in). Presently, the method is to measure each product with a ruler, and treat each pack as a cuboid. This is wildly inaccurate for many different reasons - the net result is that my lorries are stacked with half-full trays and I am "shipping air". This costs a LOT of money in an operation of this scale. What I am looking for is a tool that will easily, accurately and consistently measure the cubic volume of this product mix - I am surprised there isn't a commercially available tool to do this with - SteveBaker, there is a gap in the market here, you could make millions!!
Thanks for your responses guys! sparkl!sm hey! 21:32, 23 December 2008 (UTC)[reply]
You said "the stock is palletised in large storage boxes". Don't you know the number of items in each box ? Is the stock in each item identical ? Do you know the volume of each box (or can you get it from H×W×L) ? If so, it seems like all that is needed is some very simple math to determine the volume per item. If you need a program to determine which items can be packed together, based on the volumes of each, to fit into a given volume, that could be done, too. Beware that you don't want to get too terribly efficient at packing, though, or you'll pack the clothes so closely they will get wrinkled. Also, are there combos that shouldn't be packed together, like fragile clothes and clothes with hooks ? StuRat (talk) 01:01, 24 December 2008 (UTC)[reply]
Interesting. The problem that you have is – at least tangentially – related to a classic optimization problem in mathematics: the aptly-named bin packing problem. That problem deals with objects of known size which must be packed into a minimum number of standard bins. TenOfAllTrades(talk) 01:13, 24 December 2008 (UTC)[reply]
I wonder - isn't cloth of sufficiently reliable density to at least ESTIMATE the volume by weighing it? If your boxes are literally half full - wouldn't you get at least somewhere close by weighing it? SteveBaker (talk) 02:06, 24 December 2008 (UTC)[reply]
Since your target volume is the 1 ft3 tray you can define your clothes as occupying a portion thereof rather simply. Tape a tapemeasure each, to one side and crosswise to the bottom of your tray. Then you get a couple of plexi glass strips. Fold a sample garment. Put one strip on top of the garment, note the measurement, put one on the side of your garment (if it doesn't fill the bottom entirely. Note measurement. Repeat with the 3rd dimension if it isn't the tray's length. Since the way you fold the garment will influence the volume occupied by the garment (OR: crupmpled up laundry takes up more space than folded.) you should try various ways of folding it, unless it's pre-folded and your staff doesn't do any folding. There are a limited number of ways to fold your garments, so you'll end up with a limited number of volumes measured for each garment. (Make sure your staff knows what folding method to use when.) You could probably rig some thingamyjig to do the measuring for you, but I assume, since you only have to do it once when the garment is entered into the inventory, you wouldn't save that much manpower vs. doing it manually. The measurements are also more useful to you than the volume, because, even if 2 leather coats would fit your tray by volume, you might not be able to fold them in a way to occupy the space the way you'd need to. Thus volume is only one parameter you'd have to consider. (Your socks might have enough volume to fit in the L shaped space left between the coats and the Jeans, but I doubt your clients would appreciate if you'd deliver them crunched in there.)76.97.245.5 (talk) 02:57, 24 December 2008 (UTC)[reply]

Non-lethal weapon against gangs

What non-lethal weapons are there against a street gang attack? Mr.K. (talk) 17:47, 23 December 2008 (UTC)[reply]

See Less-lethal weapons for more information, there's a whole list of links you can follow from there. --Jayron32.talk.contribs 18:06, 23 December 2008 (UTC)[reply]
There is nothing there that I could order through the internet. :( --Mr.K. (talk) 18:20, 23 December 2008 (UTC)[reply]
Well, you didn't ask about that, did you? There is an interesting article about something the Israelis have been working on: Skunk. Perhaps you could get ahold of some scent at a hunting supply store, or google on "skunk scent" and pick a purveyor. "Deer scent" isn't much nicer-smelling. --Milkbreath (talk) 18:50, 23 December 2008 (UTC)[reply]
If it's an entire gang - you'd be very unlikely to take them all out non-lethally without also doing yourself some kind of injury. Those whom you don't incapacitate are going to be VERY upset with you and the consequences may well end up being a whole lot worse than if you'd done nothing. So this may not turn out to be such a great idea. A cellphone with a GPS and a one-button 911 dialler has to be your best defense. For just one attacker - a Tazer is probably the answer - and even against a gang, waving a Tazer around threateningly with your back to a wall may be enough to hold them off until the cops show up. A multi-shot tazer might even keep back a handful of them. Sadly, they too may have thought of this so there are never any guarantees. SteveBaker (talk) 21:15, 23 December 2008 (UTC)[reply]
I personally doubt street gangs would be intimidated by a tazer. It's not quite the same level of threat as an actual gun—it's not quite as much as an investment to take one for the team in such a situation. Frankly, you're better of not trying to violently confront street gangs, if you care about your own health. --98.217.8.46 (talk) 02:57, 24 December 2008 (UTC)[reply]
A few stink bombs may be effective if they aren't very determined. It is a non-aggressive way of getting them to go away. Dmcq (talk) 21:55, 23 December 2008 (UTC)[reply]
Are tear gas grenades obtainable? Though of course you would need a protective mask. Exxolon (talk) 22:43, 23 December 2008 (UTC)[reply]
Air bazooka would have the advantage of not being immediately recognizable as a "weapon". A Water cannon would probably also be available online somewhere. When dealing with a gang, though I'd also like to strongly advise against any vigilante action. They are very likely to sneak up on you next time and not to repay you "in kind" either. Call the cops, that's what they are there for. 76.97.245.5 (talk) 03:09, 24 December 2008 (UTC)[reply]

Unpaid medical treatment

This question has been removed. Per the reference desk guidelines, the reference desk is not an appropriate place to request medical, legal or other professional advice, including any kind of medical diagnosis or prognosis, or treatment recommendations. For such advice, please see a qualified professional. --Coneslayer (talk) 20:14, 23 December 2008 (UTC)[reply]

AeroGarden gizmo.

My wife has been afflicted by the TV adverts for a gadget called 'AeroGarden' (http://www.aerogrow.com) and wanted one for Xmas. This is a gadget that lets you grow stuff like lettuce and baby tomatoes indoors, hydroponically. Leaving it to the last minute - I wandered into the local store that sells them and was shocked...SHOCKED to see that these things cost $140...plus supplies...plus other crap. Figuring that a lettuce costs about $1 and the machine takes several weeks to grow one - I figure we'd save about $12 a year on food - so this gadget might maybe pay for itself in 10 years...oh - but you need 'supplies' and it eats 60W of electricity...so probably more like 20 years.

But not being one to disappoint - I've been wondering if I can make one...tomorrow (I have the day off and nothing better to do than to sit and chat to you guys!).

It looks like the thing consists of a container for the plant with water and nutrients underneath - and a pair of CFL lamps with a timer to do day/night cycles...plus some stuff to tell you when to add water and nutrient tablets - which can only be some other kind of timer hooked up to an LED. I'm thinking that a couple of nested plastic popcorn serving bowls with some light fittings and such plus a regular 24 hour timer would get me something functionally identical for about $10 plus lightbulbs...which at least stands a chance of reducing the payback period to something close to the 'novelty-wearing-off' period which historically has been under 6 months for this kind of gadget. (cf Breadmaker, Icecream Maker, Pizza cooker).

Does anyone here actually have one of these machines? Or some insight as to how the plant and the water make contact? Our hydroponics article offers loads of suggestions - mostly that there needs to be some kind of substrate to bring the water up to the roots by capillary action...I'm guessing that stuff like that could be found in a gardening store.

Any information that would increase the probability of it actually growing something would help. Failing that I'm just going to have to buy lettuces of steadily increasing size at the store and sneak them into the container at dead of night in order to convey the impression that whatever I make actually works! SteveBaker (talk) 21:05, 23 December 2008 (UTC)[reply]

I'm under the impression (I don't have one) that they actually use aeroponics rather than hydroponics. Same general concept, different technical implication. (Might be relevant if trying to build one yourself.) -- 128.104.112.113 (talk) 22:04, 23 December 2008 (UTC)[reply]
My first piece of advice is to grown an herb like basil, which picked fresh will add flavor not found in dried seasoning. There are a dozen varieties of basil so you'll feel like a gourmet in selecting the right blend for your pizza or spaghetti. Herbs can be harvested at any point in their life cycle and tend to be low maintenance. My second piece of advice is spend the $140 and spare yourself some grief. --Digrpat (talk) 22:12, 23 December 2008 (UTC)[reply]
Although grief may set in later when it's gathering dust. Julia Rossi (talk) 22:50, 23 December 2008 (UTC)[reply]
"Hydroponics" and "aeroponics" in ebay list over 2,700 items for growing plants indoors in artificial medium. There is equipment, lights, books, chemicals, and more. "Marijuana" also lists stuff of that kind – for educational purposes only, of course. – GlowWorm
To answer SteveBaker's question of repaying costs, a sizable portion of the hydroponic/aeroponic/indoor gardening market is people who want to grow wacky tobacky in their basement, where it is not easily detectable by authorities. There's probably a small market for people who genuinely want to grow some tomatoes and summer squash in January, but for the most part the entire industry exists to support Mary Jane production. And a crop of said weed will likely pay for the entire operation in the very first growing season. So yeah, go ahead and get the set-up for those greenhouse tomatoes in February, but also understand that you aren't really the target market for this equipment. People buying this stuff are the same people who are buying "tobacco pipes" and "rolling papers"... --Jayron32.talk.contribs 00:35, 24 December 2008 (UTC)[reply]
You've listed the financial aspects, so let me say outright that this gizmo isn't about saving money. Why else grow food at home then ? Here's some reasons:
1) If you have small kids it's a good lesson on where food comes from (and far less likely to make your kids run away from home than if you start slaughtering your own livestock). Why not just grow it in a garden then ? Well, you could, but isn't that where the neighbor's cat likes to tinkle ?
2) It enables you to control what goes into the food, so you really know it's "organic", as opposed to trusting some combination agricultural company and chemical company that claims their food is organic (even though you've seen their food glowing in the dark).
3) It's fun. OK, you're not likely to to die from excitement, but it's still a bit of a thrill to grow something yourself.
4) You have to give some piece of crap gift to everyone to honor the Holy Day of Commercialism, so why not give this ? StuRat (talk) 00:51, 24 December 2008 (UTC)[reply]
Oh, no, I think its a great idea. I mean, we do LOTS of things just for fun, and this one seems like quite a cool adventure. I wasn't disparaging the idea of gardening, just noting that SteveBaker's frustration over the cost was likely misplaced, as he's not really the target market for this product. But still, I think this is an AWESOME gift, for the record... --Jayron32.talk.contribs 01:00, 24 December 2008 (UTC)[reply]

Sail all wet?

Why do submarines have sails? It looks to me like it would add drag and weaken the structure. Clarityfiend (talk) 21:22, 23 December 2008 (UTC)[reply]

According to Conning tower (which is part of the 'sail'): "A conning tower is a raised platform on a ship or submarine, often armored, from which an officer can con the vessel; i.e., give directions to the helmsman. It is usually located as high on the ship as practical, to give the conning team good visibility.". The 'sail' is there to get the conning tower up as high as possible. SteveBaker (talk) 21:27, 23 December 2008 (UTC)[reply]
Wheres on a sailing ship giving directions to the helmsman from the conning tower makes lots of sense, on a sub I suppose it's more to do with keeping your feet dry. (The bridge being downstairs, inside the sub) If you wouldn't have the sail you'd have to raise the entire top of the sub out of the water to go "topside" for a look-see. In calm sea that wouldn't be that big a deal, but give it a bit of a chop and you'd be real happy to be a bit farther up. Apart from making the sub as easy to spot as a boat from afar, if you'd raise the whole thing up above the waves, it would add all sort of engineering challenges as to the shape of the hull where to put the ballast and pump outlets etc. 76.97.245.5 (talk) 22:27, 23 December 2008 (UTC)[reply]
Looking up that article led me to the question: Why was a French submarine called "Casabianca (Q183)" in 1935? — Sebastian 22:52, 23 December 2008 (UTC)[reply]
In addition to giving the commander a dry place to stand, it provides a handy place to put communications antennas, radar equipment, and other stuff that you want to have high up and clear of the water when the boat is surfaced (or nearly surfaced). It also makes it easier for crew to board and exit the craft in less-than-perfectly calm weather. (If you opened a hatch that was flush with the top of the rest of the hull in any sort of wave action then you'd very quickly get a sub full of water.) TenOfAllTrades(talk) 00:27, 24 December 2008 (UTC)[reply]

Red Panda, Giant Panda thumb.

The Red Panda articles says "[The Red Panda and the Giant Panda] are only very distantly related by remote common ancestry from the Early Tertiary Period." Is the specialised thumb (extra digit formed formed at the wrist), which they both have, derived from their common ancestry, or is it a case of convergent evolution? Jooler (talk) 22:39, 23 December 2008 (UTC)[reply]

Convergent evolution indeed, according to this paper. Nice! --Dr Dima (talk) 00:21, 24 December 2008 (UTC)[reply]
My understanding is that the Red Panda is closely related to racoons, while the Giant Panda is closer related to the bears. --Jayron32.talk.contribs 00:29, 24 December 2008 (UTC)[reply]
Correct. According to Flynn et. al. (2005) Syst. Biol. 54(2) pp. 317–337 (see figure 3 therein if you have full access), Red Panda's closest living relatives are Mephitidae (skunks) and Procyonidae (raccoons, coatis, kinkajous, and the like) of Musteloidea (weasel) superfamily; while the Giant Panda is closely related to bears and hence is placed in Ursidae (bear) family, superfamily Ursoidea. Happy Holidays! --Dr Dima (talk) 02:02, 24 December 2008 (UTC)[reply]

Happy Holidays!

Just saying, as per Dr Dima, happy holidays to all from here in brown Australia. o<]:-) Julia Rossi (talk) 04:12, 24 December 2008 (UTC)[reply]